Вы находитесь на странице: 1из 272

SAN BEDA COLLEGE ALABANG SCHOOL OF LAW

CIVIL PROCEDURE SPL2

INDIVIDUAL OUTPUT
COMPILATION OF CASE BRIEFS
RULES 1- 65 OF THE RULES OF
COURT

SUBMITTED BY: SUBMITTED TO:


Stephanie Pagdanganan DEAN RICO PAOLO
QUICHO
2018400116
CIVPRO SPL2
CASE MATRIX

CASE FACTS ISSUE RULING ANALYSIS

Rule 2 — Cause of Action

Samson v. Sps. Spouses Gabor own a Whether or not the No. In this case, Section 2, Rule 2 of the
Gabor parcel of land in Rizal. complaint states a aside from the fact Rules of Court provides that
They sold the land to cause of action. the respondent a cause of action is the act
petitioner, who executed a spouses had or omission by which a party
deed of sale to Ramos. mortgaged the violates the right of another.
Respondent filed an action property to
for legal redemption with respondent bank,
the RTC. Petitioner and there is no other
Ramos then executed an allegation of act or
agreement revoking the omission on part of
transfer. Respondent respondent bank in
Spouses Gabor are violation of the right
registered owners of a of petitioner.
parcel of land in Rizal
Province.

Uniwide Holdings Uniwide entered into a Whether or not a No. The restriction When construing with
v. Cruz franchise agreeemnt with case based on should be strictly Section 5, Rule 2 (which
Cruz. It was agreed that it several causes of construed as relating states the general rule
would be five years and it action is solely to the agreement where personal actions
would be that way to dismissible on for which the exclusive may be commenced and
establish and operate a ground of venue stipulation is tried where the plaintiff or
Uniwide Family Store. improper venue? embodied. Expanding defendant resides, at the
Article 10.2 of the the scope of such election of the plaintiff, the
Agreement called for the limitation on a exception being that
franchisee to pay a monthly contracting party will parties may, before filing
service fee. In case of create unwarranted of action, validly agree in
delay, it was provided that restrictions which the writing on an exclusive
there would be charged an parties might find venue) where there is a
interest. Article 27.5 unintended or worse, joinder of causes of action
provided that the arbitrary and between the same parties
franchisee consented to the oppressive. one of which does not
exclusive jurisdiction of the arise out of the contract
courts of Quezon. where the exclusive
venue was stipulated
upon, the complaint, as in
the one at bar, may be
brought before other
venues provided that such
other causes of action
falls within the jurisdiction
of the court and the venue
lies therein. In this case,
the deeds, unlike the
Agreement, bear no
exclusive venue
stipulation with respect to
the causes of action
thereunder.
Lourdes Suites v. Two executive contracts Whether or not the No. The courts are not On the one hand, failure
Binaro were executed with MeTC committed precluded from to state cause of action
respondent Binaro for room grave abuse of dismissing a case for refers to insufficiency of
accomodations for two discretion in lack of cause of action. the pleading, and is a
groups of students, as dismissing with In the case of ground for dismissal
petitioner owns a hotel. prejudice based Macaslang v. Zamora, under Rule 16 of the
Although respondent paid on lack of cause the Court provided that Rules of Court. On the
the total contract price, it is of action. failure to state cause of other hand, lack of cause
claimed that there was an action and lack of cause of action refers to a
unpaid balance for of action are really situation where the
damages to furniture. different from each evidence does not prove
other. the cause of action
alleged in the pleading.

Rule 3, Sections 1 to 3 — Parties to Civil Actions

Iron and Steel Iron and Steel Authority Yes.In this case, ISA Rule 3, Section 3 of the
Authority v. Court was created with the power Whether or not the instituted the Rules of Court expressly
of Appeals to initiate expropriation of Republic of the expropriation recognize the role of
land required for basic iron Philippines is proceedings in its representative parties,
and steel facilities. The entitled to be capacity as an agent or that a party authorized by
National Steel Corporation substituted for ISA representative of the statute may sue or be
had an expansion program, in view of the Republic pursuant to its sued without joining the
and thus the president expiration of ISA’s authority under PD 272. party for whose benefit
issued a proclamation term. From these premises, the action is presented or
withdrawing from sale or the expiration of ISA’s defended.
settlement a large track of statutory term did not
public land in Iligan. Since justify dismissal of the
certain parts of such public eminent domain
land were occupied by non- proceedings.
operational fertilizer plants
owned by respondent, an
LOI was issued directing
the National Steel
Corporation to negotiate
with respondent for
compensation of
occupancy rights.

Sps. Oco v. Sabas Limbaring subdivided No. Absent any clear Interest within the meaning
Limbaring, his lot and then executed Whether or not proof that a trust was of the Rules means material
Deeds of Sale in favor of Respondent was a created, he cannot be interest or an interest in
Jennifer Limbaring and Sarah real party in deemed a real party in issue to be affected by the
Jane Limbaring. Percita Oco, interest in the suit interest. decree or judgment of the
daughter of Sabas, filed a to rescind the case, as distinguished from
case of perjury and Deeds of mere curiosity about the
falsification of documents Reconvenyance. question involved. One
against her Respondent having no material interest
Uncle Victor Limbaring, who to protect cannot invoke the
was the father of Jennifer and jurisdiction of the court as
Sarah Jane. During pre-trial, the plaintiff in an action.
the parties agreed that the When the plaintiff is not the
two parcels of land should be real party in interest, the
reconveyed to petitioner, who case is dismissible on the
was to pay respondent ground of lack of cause of
expenses incurred to transfer action
the title to her name.

Golangco v. Fung Respondent issued an Whether or not the Yes. The OSG is to The Administrative Code of
office memorandum Court of Appeals represent “the 1987 mandates the OSG to
maliciously imputing correctly ruled on Government in the represent “the Government
against petitioner that he the petition for Supreme Court and the in the Supreme Court and
had committed bribery. certiorari of the Court of Appeals in all the Court of Appeals in all
petitioner? criminal proceedings;
This was sent to criminal proceedings; represent the Government
petitioner’s superiors in represent the and its officers in the
POEA and this casued Government and its Supreme Court, the Court of
prejudice to petitioner. officers in the Supreme Appeals, and all other courts
Court, the Court of or tribunals in all civil actions
Appeals, and all other and special proceedings in
courts or tribunals in all which the Government or
civil actions and special any officer thereof in his
proceedings in which official capacity is a party.
the Government or any
officer thereof in his
official capacity is a
party, according to the
Administrative code. In
this case, the People of
the Philippines were
indispensable parties,
and petitioner not
joining the People as a
party in his action for
certiorari in the Court of
Appeals was fatal and
enough cause for
summary rejection of
his petition.
Equitable PCI Atonio Tiu wanted to Whether or not the Yes. In this case, since Rule 3, Section 2 of the
Bank, Inc. v. Heirs secure loans obtained by complaint filed by the mortgaged property Rules of Court provides
of Tiu Ching from Petitioner Respondents, is presumed conjugal, that as a general rule,
Equitable Bank, so a real without impleading Matilde is obliged every action must be
estate mortgage in favor of Matilde who must principally under the prosecuted or defended in
petitioner was issued. Tui also be Antonio’s AREM. Therefore, she the name of the real party
also executed an an heir and who was is the real party in in interest. Section 3 of
Amendement to the Real principally obliged interest and hence, the the same Rule provides
Estate Mortgage which under the AREM, action must be that the beneficiary shall
increased the amount is dismissible for prosecuted in her name be included in the title of
secured by the mortgage. lack of cause of as she stands to be the case and shall be
This was signed by his wife action. benefited or injured in deemed to be the real
which said, “with my marital the action. party in interest.
consent”. Tiu died, and the
loan remained unsettled.

Rule 3, Sections 4 to 6 — Parties to Civil Actions

Department of An administrative order Whether or not the Yes. In this case, the As a general rule, a state
Health v. Phil was issued by the DOH. case should be DOH, being an may not be sued.
Pharmawealth, This gave out guidelines dismissed for unincorporated agency However, if it consents,
Inc., and procedures for the being a suit of the government, can either expressly or
accreditation of against the State. validly invoke the impliedly, then it may be
government suppliers for defense of immunity the subject of a suit. An
pharmaceutical products from suit because it has unincorporated
for sale or distribution to not consented to be government agency
the public. This was then sued. The doctrine of without any separate
amended by other state immunity extends juridical personality of its
administrative orders. also to complaints filed own enjoys immunity from
These orders provided that against state officials for suit because it is invested
the accreditation may be acts done in the with an inherent power of
recalled, suspended, or discharge and sovereignty.
revoked after due performance of their
deliberation, hearing, and duties.
notice by the DOH
Accreditation Committee.
The DOH then issued a
Memorandum hearing and
notice by the DOH
accreditation Committee.
Next, a list and category of
sanctions to be imposed on
accredited government
suppliers of
pharmaceuticals. The DOH
then issues a
memorandum which invited
representatives of 24
accredited drug companies
to a meeting where they
were directed to submit
within 10 days their
respective explanations on
the adverse findings
covering their products.
Instead of submitting a
written explanation, PPI
sent a late letter stating that
it referred the matter to
their lawyers. Usec Galon
then informed PPI that it’s
accreditation had been
suspended.

Air It was discovered by Whether or not the Yes. the CA thereby In exercising the right of
Transportation Spouses Ramos that a ATO could be correctly appreciated eminent domain, the State
Office v. Sps. portion of their land was sued without the the juridical character of exercises its jus imperii,
Ramos being used as part of the State’s consent. the ATO as an agency as distinguished from its
runway of Loakan airport, of the Government not proprietary rights, or jus
operated by petitioner Air performing purely gestionis; yet, even in that
Transportation Office governmental or area, where private
(ATO). The respondents, sovereign function, but property had been taken
Spouses Ramos, then was instead involved in in expropriation without
agreed to sell the land to the management of just compensation being
ATO, but ATO did not pay. Loakan Airport, an paid, the defense of
activity that was not immunity from suit could
exclusive prerogative of not be set up by the State
the State in its against an action for
sovereign capacity. payment by the owners.
Furthermore, the Immunity has not been
doctrine of sovereign upheld in favor of the
immunity cannot be proprietary functions
successfully invoked to whose function was not in
defeat a valid claim for pursuit of a necessary
compensation arising function of government,
from the taking without but was essentially a
just compensation and business.
without proper
expropriation
proceedings being first
resorted to of the
plaintiffs’ property.

Rule 3, Sections 7 to 12 — Parties to Civil Actions

Lotte Phils. v. Petitioner hired Whether or not 7J Yes. No final ruling on What is an indespensible
dela Cruz respondents to work in a is an this matter can be had party? It is important to
confectionery facility. indispensable without impleading 7J, answer this first before we
Petitioner then entered into party and should whose inclusion is delve into the discussion.
a contract with have been necessary for the An indispensable party is
Maintenance and Janitorial impleaded in effective and complete a party in interest without
Services to provide respondents’ resolution of the case whom no final
manpower for needed petition in the and in order to accord determination can be had
maintenance, utility, Court of Appeals. all parties with due of an action. The joinder
janitorial, and other process. of indispensable parties is
services. Respondent then mandatory. The absence
was fired. of an indispensable party
renders all subsequent
actions of the court null
and void. In this case, 7J
is an indispensable party.
It is a party in interest
because it will be affected
by the outcome of the
case.
Chua v. Torres An RBC check in favor of No. The Supreme Court Section 5, Rule 7 of the
Caltex Service Center was Whether or not a held that the RTC erred in Rules of Court makes no
issued by Chua in payment co-plaintiff dismissing the instant distinctions that would
of purchase of oil. But the impleaded only as complaint. expressly exempt a
check was dishonored by necessary party, necessary party from
drawee bank. Beltran did who has no claim executing the certification
not bother to check who for relief, should against forum shopping.
had issued the check and also make a Nevertheless, a misjoined
instead instituted an action certification party plaintiff has no
against petitioner Christine against forum business participating in
Chua for BP 22. The shopping. the case as a plaintiff in
policce officers went to her the first place, and it
residence, in the auto would make little sense to
repair shop of her brother, require the misjoined
and even her uni. party in complying with all
the requirement expected
of plaintiff. At the same
time, Section 11, Rule 3 of
the Rules of Court
provides that neither
misjoinder nor non-joinder
of parties is ground for
dismissal of action.
Clearly, misjoinder of
parties is not fatal to the
complaint. The rule
prohibits dismissal of a
suit on the ground of non-
joinder or misjoinder of
parties.
Office of the City Respondents claim to be No. It is an uncontested The State is neither a
Mayor of the absolute owners of land Whether or not the fact that the subject land necessary nor
Paranaque v. in Paranaque. The land State is an was formed from the indispensable party to an
Ebio was due to an accretion of indispensable alluvial deposits that have action where no positive
Cut-cut creek. It was party to gradually settled along the act shall be required from
banks of Cut-cut creek.
asserted by respondents respondents’ it or where no obligation
that the original occupant action for shall be imposed. The law
was their great grandfather prohibitory that governs over the
Jose Vitalez. Jose gave the injunction. accreted portion is Article
land to his son, and his son 84 of the Spanish Law of
continuously and Waters of 1866 and
exclusively occupied and Article 457 of the New
possessed such lot. Civil Code which provide
Respondent married the that alluvial deposits
son’s daughter, and the along banks of a creek do
couple made their house not form part of the public
on the lot. Then the son domain as the alluvial
executed a transfer of property automatically
rights in favor of Ebio. So belongs to the owner of
respondents were the estate to which it may
surprised when several have been added.
public officers proceeded to Moreover, the action for
cut 8 coconut trees planted prohibition seeks to enjoin
on such lot. the city government of
Paranaque from
proceeding with its
implementation of the
road construction project.

Bulawan v. Bulawan alleges that she Whether or not No. . In this case, during Section 7, Rule 3 of the
Aquender is the owner of a certain lot, respondent is an the proceedings before Rules of Court states that
and that she had bought indispensable the trial court, the an indispensable party is
the property from the party. answers of Yap and a party in interest without
Yaptengco brothers, who in Register of Deeds whom no final
turn claim that they had should have prompted determination can be had
inherited the property from the court to inquire of an action. The absence
Yap Chin Chun. Yap further whether there of an indispensable party
claimed ownership also were other renders all subsequent
over the same property. indispensable parties. It actions of the court null
Yap says that in another would be the height of and void for want of
civil case, the trial court inequity to allow authority.
had already declared the respondent’s title to be
subdivision survey of the nullified without being
Yaptengco brothers given the opportunity to
simulated. This means that present any evidence in
Yap Chin Chun is still the support of his
rightful owner. The trial ostensible ownership of
court ruled for Bulawan, property.
This was then affirmed by
the Court of Appeals. When
the decision became final
and executory, the trial
court issued a writ of
execution. This writ was
then questioned, as
Aquende claims that he
was unaware of any
litigation on his property

National Power National Power Corporation Whether or not No. The NPC, therefore, It was held that saying
Corporation v. received a notice of NPC still owned or ceased to operate that that NPC was liable for
Provincial franchise tax delinquency operated the business in Bataan by local franchise tax is a
Government of from the provincial business subject operation of law. nullity. EPIRA created the
Bataan government of Bataan. to local franchise National Transmission
Respondents then sent tax. Corporation (TRANSCO)
again notices of tax due. and transferred to it
The NPC then replied that NPC’s electrical
it had ceased to be liable transmission function. The
for payment of such tax NPC therefore ceased to
ever since the enactment of operate that business in
EPIRA. Ignoring NPC’s Bataan by operation of
view, a warrant of levy was law. Since local franchise
issued on 14 real tax is imposed on the
properties causing their privilege of operating a
sale at public auction. franchise, it is clear that
such tax is not a liability of
the NPC. Further, the
EPIRA created the Power
Sector Assets and
Liabilities Management
Corporation (PSALM
Corp) and transferred to it
all of NPC’s “generation
assets”. It also transferred
all existing liabilities of the
NPC to PSALM Corp,
presumably including the
unpaid liability for local
franchise tax. Since the
subject properties belong
to PSALM Corp and
TRANSCO, they are
certainly indispensable
parties to the case that
must be necessarily
included before it may
properly go forward.

Rule 3, Sections 13 to 19 — Parties to Civil Actions

Chiang Kai Shek Private respondent Oh was Whether or not a Yes. In this case, Under Rule 3, Section 1 of
School v. Court of dismissed from his school that has although the school has the Rules of Court, it
Appeals teaching position in Chian not been not been incorporated, states quite plainly that
Kai Shek school, after incorporated may petitioner cannot now only natural or juridical
having taught for a period be sued. invoke its own non- persons may be parties in
of 33 years, for no reason. compliance with the law a civil action. Act No.
to immunize it from 2706, as amended,
private respondent’s imposes the obligation
complaint. upon schools recognized
by the government to
incorporate under the
Corporation Law within 90
days from such
recognition.
Gochan v. Young The daughter of Gochan, Whether or not Yes. Cecilia then Section 3, Rule 3 of the
Alice, and the mother of Spouses Uy have remains a stockholder Rules of Court only permit
respondents, had inherited the personality to of the corporation in an executor or administrator
50 shares of stock in the file suit before the view of the nullity of the to represent or bring suits on
corporation Gochan Realty SEC. Contract of Sale. behalf of the deceased, but
this does not prohibit the
from her father. When Alice Although she was no heirs from representing the
died, she left 50 shares to longer registered as a deceased.
her husband, John Young stockholder in the
Sr. The RTC of Cebu records as of filing of
adjudicated 6/14 of these case, the admitted
shares to her children. allegations in the
John Sr requested Gochan complaint made her still
to partition the shares of his a bona fide stockholder
late wife. This would be of Gochan Realty as
done by canceling the between the parties.
stock certificates in his
name. Then, new
certificates would be issued
in the name of
respondents. Gochan
Realty refused, citing the
right of first refusal given to
the remaining stockholders.

Judge Sumaljag A complaint for the nullity of Whether or not the No. In this case, Section 16, Rule 3 likewise
v. Sps. Diosdidit deed of sale of real deceased may be although counsel for expressly provides that "the
property was executed substituted by Josefa did in fact notify heirs of the deceased may
between Josefa and petitioner. the court of the fact of be allowed to be substituted
spouses Diosdidit and her death, he was not for the deceased, without
requiring the appointment of
Literato. It was alleged that able to properly give the an executor or
this deed of sale was court the name and administrator . . .".
spurious. Josefa was the address of the legal
sister of Menendez, and representatives. The
they were two of six heirs person, petitioner
who inherited the property herein, that counsel
in equal parts. gave as substitute was
Respondents filed an not one of those
amended answer with mentioned under
counterclaim, impleading Section 16 of Rule 3.
Petitioner Sumaljag with
Josefa on the allegation
that petitioner, at the
instance of Josefa,
occupied the lots without
respondents’ authority.

O. Ventanilla Alfredo S Tan and private Whether or not the Yes. The CA was correct The reason given by
Enterprises v. Tan responded Adelina Tan were the Court of in ruling that there is no petitioner, that its former
leased to properties by Appeals was extraordinary counsel had died before the
petitioner. The properties in correct. circumstance in this case CA Decision was
question were in Cabanatuan that would merit a recall of promulgated, hence, it was
City. The Tans, however, did the entry of judgment to not properly notified of the
not follow the terms of the reopen the case. judgment, is too tenuous to
lease. Petitioner then filed a be given serious
complaint against the Tans for consideration. In Mojar, et
termination of contract. A al. v. Agro Commercial
case was then filed in the Security Service Agency,
RTC Inc.,[17] the Court explained
that it is the party's duty to
inform the court of its
counsel's demise, and
failure to apprise the court of
such fact shall be
considered negligence on
the part of said party.

Sarsaba v. Fe Respondent represented by What is the legal No. In the case before Us, When a party to a pending
vda. De Te her attorney-in-fact, Faustino effect of death it appears that action dies and the claim is
Castañeda, filed with the of the plaintiff during respondent's counsel did not extinguished, the Rules
RTC, a Complaint for the pendency of the not make any of Court require a
recovery of motor vehicle, case? manifestation before the substitution of the deceased.
damages with prayerfor the RTC as to her death. In In such cases, a counsel is
delivery of the truck pendente fact, he had actively obliged to inform the court of
lite against petitioner, participated in the the death of his client and
Sereno,Lavarez and the proceedings. Neither had give the name and address
NLRC of Davao City. he shown any proof that of the latter's legal
Respondent alleged, among he had been retained by representative. The rule on
others, that: (1) she is the wife respondent's legal substitution of parties is
of the late Pedro Te, the representative or any governed by Section 16, 46
registered owner of the truck, one who succeeded her. Rule 3 of the 1997Rules of
as evidenced by the Official Civil Procedure, as
Receipt and Certificate of amended. The rule on
Registration. Petitioner substitution by heirs is not a
alleges that that there was no matter of jurisdiction, but a
showing that the heirs have requirement of due process.
filed an intestate estate The rule on substitution
proceedings of the estate of was crafted to protect every
Pedro Te, or that respondent party's right to due process.
was duly authorized by her It was designed to ensure
co-heirs to file the case; and that the deceased party
that the truck was already would continue to be
sold to Gasing on March 11, properly represented in the
1986 by one Jesus Matias, suit through his heirs or the
who bought the same from duly appointed legal
the Spouses Te. Corollarily, representative of his estate.
Gasing was already the lawful It is only when there is a
owner of the truck when it denial of due process, as
was levied on execution and, when the deceased is not
later on, sold at public represented by any legal
auction. On October 17, 2005, representative or heir, that
petitioner filed an Omnibus the court nullifies the trial
Motion to Dismiss the Case proceedings and the
on the following grounds: (1) resulting judgment therein.
lack of jurisdiction over one of
the principal defendants; and
(2) to discharge respondent's
attorney-in-fact for lack of
legal personality to sue. It
appeared that the respondent,
Fe Vda. de Te, died on April
12, 2005. Respondent,
through her lawyer, argues
that respondent's death did
not render functus officio her
right to sue since her
attorney-in-fact, Faustino
Castañeda, hadlong testified
on the complaint on March
13, 1998 for and on her behalf
and,accordingly, submitted
documentary exhibits in
support of the complaint.
Dagadag v. Respondents were sent a Who may appeal The CSC and the mayor The established rule is that a
Tongnawa memorandum by Dagdag, from the Decision of of Tanudan are real real party in interest is one
the mayor of the municipality the Court of parties in interest in this who would be benefited or
of Tanudan, asking them why Appeals? case and, therefore, can injured by the judgment, or
they should not be contest the decision of the one entitled to the avails of
administratively santioned for CA. the suit. The word "interest,"
acts unbecoming of public as contemplated by the
servants and failure to Rules, means material
perform their duties. They interest or an interest in
were given 72 hours. Michael issue and to be affected by
Tongnawa and Antonio the judgment, as
Gammod, respondents, are distinguished from mere
the municipal engineer and interest in the question
municipal planning and involved or a mere incidental
development coordinator.The interest. Stated differently,
Municipal Grievance the rule refers to a real or
Committee found respondents present substantial interest
liable for insubordination, non- as distinguished from a
performance of duties and mere expectancy, or a
absences without official future, contingent,
leaves. Respondents were subordinate, or
then suspended for two consequential interest. As a
months. general rule, one who has
no right or interest to protect
cannot invoke the
jurisdiction of the court as
party-plaintiff in an action.

Rule 3, Sections 18 to 22 — Parties to Civil Actions

Torres v. Rodellas Respondent left her family Whether or not the Yes. While it is not Because pf A.O No. 18,
and went to Saudi Arabia. counsel of Edwino mentioned when Edwino’s s.1987, Rules of Court apply
Edwino moved to has the personality exact date of death is in in a suppletory character
Respondent’s house in to file Motion for Motion for whenever practicable. Sec.
Occidental Mondoro. Edwino Reconsideration Reconsideration filed 16, Rule 3, Rules of Court, is
occupied the portion vacated before the Office of the applicable here. Substitution
by respondent’s sister. He President, considering of a deceased party by heirs
claims that respondent sold counsel told Office of the is allowed in actions that
him the house, and he has an President of Edwino’s survive the death of a party
affidavit of death in a delayed thereto, particularly, causes
relinquishment/sale of right. manner, sec. 16, Rule 3 of action that affect primarily
Edwino also filed an MSA with merely provides for and princippally property
the DENR in his own name disciplinary action against and properety rights, injuries
for the property. counsel in case of such to the person being merely
Respondent’s MSA failure to comply with its incidental. In this case, the
application was turned down duty. In said section, one action outlives the death of
for Edwino. Respondent’s son can find no mention that Edwino as this case involves
returned. He learned of counsel would be without the parties’s respective
Edwino’s claim and filed with legal personality to appear rights to acquire property
the community environment for the benefit of the client which is the MSA grant.
and natural resources office or its heirs.
(CENRO) against Edwino’s
MSA on the ground that the
affidavit of relinquishment was
forged as respondent was
abroad
Re: Query of Mr. Good Shepherd Foundation is Can the Courts No. The Good Shepherd Article III, Sec 11 of the
Roger C. an organization which works grant to Good Foundation, not being a 1987 Constitution is clear in
Prioreschi Re for and with indigent persons, Shepherd who natural litigant, cannot be it’s intent , where the bases
exemption from to quote “the poorest among works for indigent exempt. or the exemption from legal
legal and filing the poor, to the newly born and underprivileged and filing fees is the free
fees of the Good and abandoned babies, to people, from access clause. The clear
Shepherd children who never saw the payment of legal intent of the language of the
Foundation, Inc smile of their mother, to old fees? provisions of the Rules of
people who cannot afford a Court show that only a
few pesos to pay for common natural party litigant is
prescriptions, to broken indigent.
families who returned to a
normal life. The administrator
of this asked the Supreme
Court if he could be exempt
from the payment of legal
fees.

Rule 4 — Venue of Actions

Marcos-Araneta The Benedicto group refused Is an affidavit Yes. the signature of any Regarding the certificate of
v. Court of to reconvey 65% enough for forum of the principal petitioners non-forum shopping, the
Appeals stockholdings of FEMII and shopping? or principal parties, as general rule is that all the
UEC as part of their oblidation Francisca is in this case, petitioners or plaintiffs in a
to hold those shares and their would constitute a case should sign it.
fruits in trust and for the substantial compliance However, time and again it
benefit of petitioner. Thus, with the rule on is stressed that the rules on
petitioner instituted two verification and forum shopping, which were
complaints which were later certification of non-forum designed to promote the
consolidated. Respondent shopping. It cannot be orderly administration of
and his daughter tried to have overemphasized that justice, do not interdict
the cases dismissed, which Francisca herself was a substantial compliance with
the RTC did. Petitioner then principal party. its provisions under
filed an amended complaint. justifiable circumstances.
Julita Benedicto took over
after her husband
Sps. Ang v. Sps. Was the dismissal Yes. The petitioners The rules on the venue of
Ang A $300,000 loan was of the complaint on assert, the said complaint personal actions are fixed
obtained by respondents from the ground that for collection of sum of for the convenience of the
petitioners, as well as a venue was proper? money may be filed in the plaintiffs and their witnesses.
promissory note executed for court of the place where If one were to invoke
the latter, where respondents Atty. Aceron resides, Section 3, Rule 3 of the
promised to pay petitioners which is the RTC of Rules of Court, one would
with interest of 10% per year Quezon City. insist that Atty. Aceron,
upon demand. But being their attorney-in-fact,
respondents failed to pay. is deemed a real party in
The loan already amounted to interest in the case below
720,000 plus interest, and can prosecute the same
petitioners, residents of LA, before the RTC.
executed an SPA to Atty
Aceron to file an action in
court against respondents.

Universal Robina There was a contract of sale WON the trial court No. A court may not A court may not dismiss an
Corporation v. between Universal Robina may dismiss motu dismiss an action motu action motu proprio on the
Lim Corporation and Albert Lim. proprio petitioner’s proprio on the ground of ground of improper venue as
Petitioner sold to Lim grocery complaint on the improper venue as it is not it is not one of the grounds
products amounting to 808k. ground of improper one of the grounds wherein the court may
After tendering partial venue? wherein the court may dismiss an action motu pro
payments, Lim did not settle dismiss an action motu prio on the basis of the
his obligation despite proprio on the basis of the pleadings. Sec. 2. Rule 4 of
demands. pleadings. the Rules of Court provide
that in venue of personal
actions. All other actions
may be commenced and
tried where the plaintiff or
any of the principal plaintiffs
resides, or where the
defendant or any of the
principal defendants resides,
or in the case of a non
resident defendant where he
may be found, at the
election of the plaintiff. Sec.
4 Rule 4 of the same Rules
provide that this Rule shall
not apply. Where the parties
have validly agreed in
writing before the filing of the
action on the exclusive
venue thereof. In personal
actions, the plaintiff may
commence an action either
in the place of his or her
residence or the place
where the defendant
resides. The parties may
agree to a specific venue
which could be in a place
where neither of them
resides. In addition, it is
stated under Sec. 1, Rule 9
of the Rules of court that
improper venue not
impleaded in the motion to
dismiss or in the answer is
deemed waived.The court
may only dismiss an action
motu proprio in case of lack
of jurisdiction over the
subject matter, litis
pendentia, res judicata and
prescription.

Saludo v. AMEX is a corporation doing WON venue was Yes. The appellate court The rule on venue, like other
American business in The Philippines improperly laid committed reversible error procedural rules, is designed
Express which engages in providing in finding that petitioner to insure a just and orderly
International, credit and other credit Saludo was not a resident administration of justice, or
Inc., facilities. It is situated in of Southern Leyte at the the impartial and
Makati. Plaintiff is a member time of the filing of his evenhanded determination
of the House of complaint of every action and
Representatives who lived in proceeding., so while
Leyte. Petitioners credit card petitioner’s complaint for
from that company was damages against
dishonored as well as his respondents before the court
daughter. He was then is a personal action
charged with late payment, governed by Section 2, Rule
and his cards were cancelled. 4 of the Rules of Courts, the
Petitioner claimed moral choice of venue for personal
damages as a result of actions cognizable by the
AMEX’s act which was RTC is given to plaintiff but
committed in gross and not to plaintiff's whim
evident bad faith. In their because the matter is
answer, respondents regulated by the Rules of
specifically denied the Court.
allegations in the complaint.
Further, they raised the
affirmative defenses of lack of
cause of action and improper
venue. On the latter,
respondents averred that the
complaint should be
dismissed on the ground that
venue was improperly laid
because none of the parties
was a resident of Leyte
San Luis v. San San Luis contracted three Was venue was Yes. The petition for Section 1, Rule 73 of the
Luis marriages. The first marriage, properly laid? letters of administration of Rules of Court. In election
there were six children, the the estate of Felicisimo cases, "residence" and
second, one son, the third he should be filed in the "domicile" are treated as
had no son. Respondent Regional Trial Court of the synonymous terms, that is,
wanted a dissolution of their province "in which he the fixed permanent
conjugal partnership assets resides at the time of his residence to which when
and filed a petition for letters death. There is a absent, one has the
of administration before the distinction between intention of returning.
RTC. The heirs from the first "residence" for purposes However, for purposes of
wife filed a motion to dismiss, of election laws and fixing venue under the Rules
but the trial court denied. "residence" for purposes of Court, the "residence" of a
Stating at the time of San of fixing the venue of person is his personal,
Luis’ death, since he was the actions. actual or physical habitation,
governor of Laguna, it should or actual residence or place
have been filed there. of abode, which may not
necessarily be his legal
residence or domicile
provided he resides therein
with continuity and
consistency.

United Overseas A mortgage agreement was Whether or not In the present case, there Section 1, Rule 4 of the
Bank v. Rosemor entered between petitioner Rosemoor is only one proceeding 1997 Rules of Civil
Mining and and respondent, wherein committed forum- sought to be nullified and Procedure, provides that
Development respondent mortgaged 6 shopping in filing that is the extra-judicial actions affecting title to or
parcels of land in Nueva the two cases mortgage foreclosure possession of real property,
Ecija. Extrajudicial foreclosure against the bank sale. And there is only or interest therein, shall be
of the properties was made, one initial transaction commenced and tried in the
and petitioner was the highest which served as the basis proper court which has
bidder. of the foreclosure sale jurisdiction over the area
and that is the mortgage wherein the real property
contract. involved, or a portion
thereof, is situated.The
venue of the action for the
nullification of the
foreclosure sale is properly
laid with the Malolos RTC
Rule 5 — Uniform Procedure in Trial Courts

Lucas v. Fabros Elenita did not appear the WON the motion for Here, the order of As a rule, a motion for
Preliminary Conference, and reconsideration filed dismissal issued by reconsideration is a
so Judge Fabros dismissed by Elenita is a respondent judge due to prohibited pleading under
the case. However, Elenita prohibited pleading failure of a party to appear the Rule on Summary
filed a motion for as contemplated during the preliminary Procedure. This rule,
reconsideration and such was under the Revised conference was obviously however, applies where the
granted. Lucas, who was the Rules on Summary not a judgment on the judgment sought to be
respondent in that case, filed Proceeding? merits of the case. reconsidered is one
another case against the rendered on the merits. As
judge for gross ignorance, held by the Court in an
stating Section 19 of the earlier case involving Sec.
Rules of Summary Procedure, 19 (c) of the Revised Rules
as a motion for on Summary Procedure:
reconsideration is prohibited. "The motion prohibited by
Respondent explained, this Section is that which
however, that it was granted seeks reconsideration of the
in the interest of justice. judgment rendered by the
court after trial on the
merits of the case."
Bongato v. Petitioner was accused of WON the Court of Yes. The MTCC should Pursuant to Section 36 of
Malvar, unlawfully entering a parcel of Appeals gravely have squarely ruled on BP 129, the Court on June
land belonging to the abused its the issue of jurisdiction. 16, 1983, promulgated the
spouses. Petitioner then filed discretion in ruling Rule on Summary
a motion for extension of time, that the Motion to Procedure in Special Cases,
which was denied by the Dismiss was a a motion to dismiss or quash
MTCC, as it was proscribed prohibited pleading is a prohibited pleading.
under the Rule on Summary Under the 1991 Revised
Procedure. Petitioner filed an Rule on Summary
answer (with a new counsel) Procedure, however, a
which the MTCC disregarded, motion to dismiss on the
as it was filed beyond the ten- ground of lack of jurisdiction
day reglementary period. over the subject matter is an
Petitioner filed a motion to exception to the rule on
dismiss with another counsel prohibited pleadings.
which the MTCC denied.
Thereafter, the MTCC
rendered a decision ordering
petitioner to vacate the land in
question, and to pay rentals,
attorneys fees, and the costs
of the suit.
Pascual v. Lorenzo Manaois was WON Respondent Yes. Verily, judgment Section 6 of the Rule allows
Jovellanos accused of forcible entry, but failed to apply the should have been the trial court to render
the complaint was dismissed Rules on Summary rendered based on the judgment, even motu
as it was without prejudice for Procedure? allegations of the proprio, upon failure of the
being insufficient in some Complaint and the defendant to file an answer
material allegations. Petitioner evidence presented within the reglementary
then corrected this, but therein, inasmuch as the period. Moreover, under
instead of answer. Manaois defendant failed to file his Section 10 of the Rule,
filed a motion to strike out as answer after the lapse of respondent was duty-bound
he claims the complaint is ten (10) days from the to render his decision within
false. No answer was service of the summons. thirty (30) days from receipt
submitted, and the motion to of the last affidavits and
strike out was granted. position papers, or the
Complainant accused judge expiration of the period for
of neglect of duty. filing them. This
notwithstanding, he has not
yet ruled on the Motion for
Summary Judgment dated
December 15, 1999, filed in
accordance with Section 6 of
the Rule on Summary
Procedure.
Rule 6 — Kinds of Pleadings
Victorina v. Filed for a complaint for sum WON the CA erred Yes. The basic requirement under
Brewmaster of money against petitioner, in its decision the rules of procedure is that
International Respondent is a marketing a complaint must make a
company who sells beer. plain, concise, and direct
Accordingly, petitioner got a statement of the ultimate
credit of 182. 502 pesos. As facts on which the plaintiff
proof, there are sales relies for his claim. Ultimate
invoices. It says that despite facts: substantial facts which
demands, they did not pay. either directly form the basis
Petitioner responds, denying of the plaintiff's primary right
any knowledge of the and duty or directly make up
obligation. the wrongful acts or
omissions of the defendant.
They refer to the principal,
determinative, constitutive
facts upon the existence of
which the cause of action
rests. Thus, the CA erred.
Mongao v. Pryce Respondent agreed to sell to Whether or not the No. The records reveal Sec. 1, Rule 19 of the ROC
Properties the latter for the price of 5 pleadings of the that respondent provides that where an
million a parcel of land in respondent corporation did not file any answer fails to tender an
Cotabato. Respondent paid corporation did not formal complaint for issue, or otherwise admits
the 550,000 earnest money, tender an issue? consignation the material allegations of
but after considerable delay, the adverse party's pleading,
offered to pay the balance, the court may, on motion of
but such was rejected. that party, direct judgment
Respondent did not pay the on such pleading.
check to petitioner Mongao Respondent’s answer
despite written and oral admitted that there was a
demands. perfected contract of sale
between it and petitioner
and that respondent refused
to tender payment of the
purchase price solely to
petitioner. Now, if an
answer does in fact
specifically deny the material
averments of the complaint
in the manner indicated by
said Section 10 of Rule 8,
and/or asserts affirmative
defenses in accordance with
Sections 4 and 5 of Rule 6, a
judgment on the pleadings
would naturally not be
proper.

Buncayao v. Fort Petitioner is one of two Whether or not Yes. The only claim that A compulsory counterclaim
Ilocandia entrepeneurs who added respondent's remained was the claim is any claim for money or
Properties, improvements on the counterclaim is for damages against any relief, which a defending
foreshore area of a beach compulsory? respondent. party may have against an
where a hotel was opposing party, which at the
constructed. The other time of suit arises out of, or
entrepeneurs began setting is necessarily connected
up their own stalls in the area, with, the same transaction or
and formed a Beach Resport occurrence that is the
Owner’s Association. Six subject matter of the
parcels of land in the beach plaintiff's complaint. It is
were transferred to the compulsory in the sense that
Philippine Tourism Authority. it is within the jurisdiction of
Petitioner and other D’Sietro the court, does not require
members then applied for a for its adjudication the
foreshore lease with the presence of third parties
CENRO and was granted a over whom the court cannot
provisional permit. DENR acquire jurisdiction, and will
then denied the lease be barred in the future if not
applications. set up in the answer to the
complaint in the same case.
Any other counterclaim is
permissive.
Philtranco On Felix Paras way home toWhether or not Paras No. The Supreme Court Under the Section 12 Rule 6
Service Manila from Bicol, he boarded acan recover damages does not disturb the of Rules of Court— A third-
Enterprises v. CA vehicle owned and operated bymoral damages in this unanimous findings by the party complaint is a claim
Inland Trailways inc. The bussuit based on quasi- CA and the RTC on the that a defending party may,
was bumped by another vehicledelict negligence of Philtranco with leave of court, file
owned and opearated by and its driver being the against a person not a party
Philtranco. The said accident direct cause of the to the action, called the
brought damage to the vehicles, physical injuries of Paras third-party defendant, for
and caused physiacal injuries to and the material damage contribution, indemnity,
the passengers. Paras was of Inland. subrogation or any other
taken to the hospital, where he relief, in respect of his
was diagnosed with several opponent's claim. In
injuries such as hematoma. Balbastro v. Court of
PAras went to two operations Appeals, Section 12 of Rule
affecting the fractured portions 6 of the Revised Rules of
of his body. Court authorizes a
defendant to bring into a
lawsuit any person "not a
party to the action . . . for
contribution, indemnity,
subrogation or any other
relief in respect of his
opponent's claim." The
impleader of new parties
under this rule is proper only
when a right to relief exists
under the applicable
substantive law. The
requisites for a third-party
action are, firstly, that the
party to be impleaded must
not yet be a party to the
action; secondly, that the
claim against the third-party
defendant must belong to
the original
defendant; thirdly, the claim
of the original defendant
against the third-party
defendant must be based
upon the plaintiff's claim
against the original
defendant; and, fourthly, the
defendant is attempting to
transfer to the third-party
defendant the liability
asserted against him by the
original plaintiff.
Section 3, Rule 7 of the Rules of Civil Procedure provides, “Every pleading must be signed by the party or counsel representing him,
stating in either case his address which should not be a post office box. The signature of counsel constitutes a certificate by him that
he has read the pleading.
Sameer Overseas Respondent was hired under a Whether or not The award of the three- In Serrano
Placement one year employment contract Cabiles month equivalent of v. Gallant Maritime Services,
Agency Inc with Sameer Overseas was entitled to the respondent’s salary Inc. and Marlow Navigation
Placement Agency Inc. She unexpired portion of should be increased to the Co., Inc., this court ruled that
went to Taiwan to work in Wacol her salary due to amount equivalent to the the clause “or for three (3)
as quality control. There, she illegal dismissal.? unexpired term of the months for every year of the
was asked to work as a cutter. employment unexpired term, whichever is
A certain Mr Huwang then contract.Therefore, the less” is unconstitutional for
informed her without prior notice said pleadings cannot be violating the equal protection
that she was terminated and considered unsigned and clause and substantive due
that “she should immediately without any legal effect. process. Sameer contends
report to their office to get her that both the June 6, 2001
salary and passport.” She was Petition and the July 5, 2001
asked to “prepare for immediate Motion for Reconsideration
repatriation.” She earned only a filed by ASBT before the Court
total of 9k NT because Wacoal of Appeals were signed by
deducted to cover her plane Mildred Santos, as corporate
ticket expense back to Manila. president, who is not a
member of the Bar. As such,
Sameer argues that both the
Petition and the Motion for
Reconsideration should be
considered unsigned
pleadings which produce no
legal effect, pursuant to the
last paragraph of Section 3,
Rule 7 of rules of court.
Obviously, the rule allows the
pleadings to be signed by
either the party to the case or
the counsel representing that
party

Anderson v. Ho An ejectment case was filedWhether or not the Yes. The Supreme Court The need to abide by the
with the MeTC. This case wasrules on certification held that there is No Rules of Court and the
then dismissed. Later, it wasagainst forum justifiable reason exists in procedural requirements it
appealed to the RTC which alsoshopping may be this case as to relax the imposes has been constantly
dismissed the same. Thus,relaxed in this case. rule on certification underscored by this Court.
petitioner filed a motion for against forum shopping. One of these procedural
reconsideration. This was then requirements is the certificate
denied by the RTC. Wanting to of non-forum shopping which,
file with the CA a petitioner for time and again, has been
review under Rule 42 of the declared as basic necessary
Rules of Court, Atty Oliva, and mandatory for procedural
petitioner’s counsel. Filed a orderliness. In Vda. De
motion for extension of time of Formoso v. Philippine National
15 days. This was granted by Bank the Court reiterated the
the CA. Later, said counsel guidelines respecting non-
sought another extension of 15 compliance with or submission
days as the petition had been of a defective certificate of
finalized and sent to Anderson non-forum shopping, the
in hawaii for her to read as well relevant portions of which are
as sign. as follows: As to certification
against forum shopping, non-
compliance therewith or a
defect therein, is generally not
curable by its subsequent
submission or correction
thereof, unless there is a need
to relax the Rule on the ground
of ‘substantial compliance or
presence of ‘special
circumstances or compelling
reasons’.

Fuentebella v. Respondent engaged the Was the certification Yes.


Castro funeral services of Rolling false because there An omission in the certificate
Hills Memorial Park for the was already a prior of non-forum shopping about
interment of the remains of case before the
her dead husband, Freddie MTCC any event that would not
Castro. As the burial was
constitute res judicata and litis
taking place and the casket of
her deceased husband was to pendentia, as in the present
be lowered, it was discovered
that the dimensions of the case, is not fatal as to merit
vault did not fit with the the dismissal and nullification
measurements of the casket.
Because of this, the casket of the entire proceedings
was lifted and placed under
considering that the evils
the heat of the sun for one
hour in front of all the sought to be prevented by the
mourners. To her humiliation,
the employees of petitioner said certificate are not present.
measured the casket by using Hence, in any event, the trial
a spade.
court correctly held that the
submission of a false
certification shall constitute
indirect contempt of court,
without prejudice to the
corresponding administrative
and criminal sanctions.

Cagayan Valley v. Whether Cagayan’s Yes. The president can The following officials or
Commissioner of Petitioner says that it granted president can sign sign the verification and employees of the company
Internal Revenue 20% sales discounts to qualified the subject certification. The Court can sign the verification and
verification and has recognized the certification without need of
senior citizens on purchases of certification sans authority of some a board resolution: (1) the
the approval of its corporate officers to sign Chairperson of the Board of
medicine. This was in Board of Directors. the verification and Directors, (2) the President
certification against forum of a corporation, (3) the
accordance with RA 7432. This shopping General Manager or Acting
was treated as deductions from General Manager, (4)
Personnel Officer, and (5)
the gross sales in order to arrive an Employment Specialist in
a labor case. The rationale
at the net sales instead of applied in the foregoing
cases is to justify the
treating them as tax credit as
authority of corporate
provided. They then filed a officers or representatives of
the corporation to sign the
claim with the BIR for tax verification or certificate
against forum shopping,
refund/ tax credit. BIR did not being “in a position to verify
act on the tax credit so Cagayan the truthfulness and
correctness of the
filed a petition for review before allegations in the petition.
the CTA.

Chua v. Chua was president of theWhether or not Petitioners were unable to If the forum shopping is not
Metropolitan company Chua and Filiden. Thesuccessively filing state in the Certificate of considered willful and
Bank and Trust company got a 4 million pesoCivil Case No. CV-01- Non-Forum Shopping the deliberate, the subsequent
Company loan from metrobank, secured0207 and Civil Case existence of Civil Case case shall be dismissed
by a real estate mortgage.No. CV-05-0402 No. CV-01-0207 pending without prejudice, on the
However, the value of theamounts to forum before RTC-Branch 258. ground of either litis
collateral was more than theshopping. pendentia or res judicata.
loan, so Chua was given an However, if the forum
open credit line for future loans. shopping is willful and
They were unalbe to pay their deliberate, both (or all, if...
obligations, so metrobank there are more than two)
entered into a debt settlement actions shall be dismissed
agreement with them, whereby with prejudice..[43]
the loan was restructured.
Finally, it was demanded that
the company pay their liabilities.
When petitioners were unable to
pay, Metrobank tried to extra-
juducually forcelose the REM
constituted on subject
properties.

Metrobank v. A loan was obtained byWhether or not THE No. The petition is not writ of possession may issue
Santos respondent from Metrobank ofCOURT OF covered by Section 5, either (1) within the one year
the amoung of two millionAPPEALS Rule 7 of the Rules and a redemption period, upon the
pesos. To secure payment, aCOMMITTED A certification against forum filing of a bond, or (2) after the
real estate mortgage over aMISAPPREHENSION shopping is not required. lapse of the redemption
condominium unit in Makati andOF FACTS period, without need of a
all it’s improvements were bond. In order to obtain a writ
issued. When the loan was not of possession, the purchaser
paid despite demand, in a foreclosure sale must file
Metrobank instituted an a petition, in the form of an ex
extrajudicial foreclosure parte motion, in the
proceedings against the real registration or cadastral
estate morgage, where proceedings of the registered
Metrobank was the highest property. The reason why this
bidder. So Metrobank registered pleading, although
the certificate of sale. The denominated as a petition, is
redemption period lapsed actually considered a motion is
without De Koning redeeming best explained in Sps. Arquiza
the property. Metrobank v. CA, where it is stated that
demanded that possession over the certification against forum
the condominium unit be turned shopping is required only in a
in. complaint or other initiatory
pleading. The ex parte petition
for the issuance of a writ of
possession filed by the
respondent is not an initiatory
pleading.

Rule 8 — Manner of making allegations in a Pleading

Vda. de Daffon v. Petitioner and her husband had Whether or not the Yes. The allegations in In this case, the Court of
CA one child, Joselito, who married Court of Appeals the complaint in the case Appeals was correct. The
respondent and they had six erred at bar are sufficient to Supreme Court ruled that in
children. Joselito dies after his establish respondent's the determination of whether
father. Respondent, with her six right to the estate of a complaint fails to state a
children, instituted an action for Amado. By stating their cause of action, only the
partition against petitioner over relationship to the statements in the complaint
the properties left by petitioner’s deceased, they may be properly considered.
husband which formed part of established their line of The complaint needs only to
his conjugal partnership. succession as the basis allege the ultimate facts on
for their claim. Their rights which the plaintiffs rely for
to succeed as heirs were their claim.
transmitted from the
moment of death of the
decedent.
Titan Spouses David acquired a lot in Whether or not No. It is true that the reply While Section 8, Rule 8 is
Construction v. White Plains, Quezon City, there is a Failure to filed by Manuel alleging mandatory, it is a discovery
David, registered under the name of Deny the that the special power of procedure and must be
“MARTHA S. DAVID, of legal Genuineness and attorney is a forgery was reasonably construed to
age, Filipino, married to Manuel Due Execution of not made under oath. attain its purpose, and in a
A. David". Spouses then the SPA However, the complaint, way as not to effect a denial
separated and no longer which was verified by of substantial justice. The
communicated. Manuel then Manuel under oath, interpretation should be one
discovered that Martha had alleged that the sale of the which assists the parties in
previously sold the property to subject property executed obtaining a speedy,
Titan Construction Corporation by his wife, Martha, in inexpensive, and most
for 1m through a deed of sale in favor of Titan was without important, a just
the name of Titan. Manuel filed his knowledge, consent, determination of the
a Complaint for Annulment of and approval, express or disputed issues..
Contract and Reconveyance implied; and that there is
against Titan before the nothing on the face of the
RTC.Manuel alleges that the deed of sale that would
sale executed by Martha in show that he gave his
favor of Titan was without his consent thereto.
knowledge and consent.

Consolidated A complaint for recovery of sum Whether or not the Yes. In this case, both the There can be no dispute to
Bank v. Del of money against respondents CA erred when it court a quo and the Court the fact that the allegations
Monte Motor was filed by petitioner, who found that of Appeals erred in ruling in the ANSWER of both
Works impleades the spouse of respondents denied that respondents were defendants, they denied
Morales in order to bind their the material able to specifically deny generally and specifically
conjugal partnership of gains. allegations of the allegations in under oath the genuineness
Petitioner says that it extended petitioner’s petitioner's complaint in and due execution of the
a loan to respondents in the complaint? the manner specifically promissory note and by way
amount of 1 million pesos. required by the rules. of special and affirmative
Under the promissory note, defenses herein states that
respondents bound themselves he (MORALES) never
to pay petitioner the full amount signed the promissory note
of the loan through twenty five attached to the complain in
monthly installments of 40,000 a his personal and/or
month. Respondents defaulted individual capacity.
on their monthly installments Moreover, what appears in
despite demand. the record was an admission
of paragraphs 1 & 2 but they
deny generally and
specifically the rest of the
allegations. It would be
considered that there is a
sufficient compliance of the
requirement of the law for
specific denial.
Luistro v. CA First Gas Power Corporation Whether the No. The complaint falls Section 5, Rule 8 of the 1997
entered into a Substation complaint alleges short of the requirement Rules of Civil Procedure states
interconnection agreement with fraud with that fraud must be stated that “In all averments of fraud
Meralco and NPC, where First particularity as with particularity. or mistake, the circumstances
Gas would finance, construct, required under constituting fraud or mistake
and commission a 230 kilovolt Section 5, Rule 8 of must be stated with
electric power transmission line. the 1997 Rules of particularity. Malice, intent,
Under the SIA, First Gas had to Civil Procedure knowledge or other condition
acquire easement of right of of the mind of a person may
way over certain lands. Frist be averred generally.” Again,
Gas entered into a Contract of the complaint falls short of the
Easement of Right of Way with requirement that fraud must be
Luistro, an owner of a parcel of stated with particularity. The
land in Batangas where First complaint merely states “That
Gas was granted perpetual sometime in the year of 1997,
easement. However, Luisitro’s the consolidator-facilitator of
counsel wrote a letter to Frist the Defendants FGPC and
Gas asking for a temporary Balfour by means of fraud and
stoppage of all kinds of work machinations of words were
within the vicinity of petitioner’s able to convince[] the plaintiff
residential hoouse pending to enter into ‘CONTRACT OF
settlement of petitioner’s EASEMENT OF RIGHT OF
grievance that the house and WAY’ wherein the latter
other improvements lay granted in favor of the
underneath the transmission defendant FGPC the right to
wire/line being constructed and erect [its] Tower No. 98 on the
would endanger the life and land of the plaintiff situated at
health of the persons in the Barangay Maigsing Dahilig,
vicinity. Lemery 4209 Batangas
including the right to Install
Transmission Lines over a
portion of the same property
for a consideration therein
stated, a xerox copy of said
contract is hereto attached as
[] ANNEXES “A” up to “A-4” of
the complaint” AND; “That the
said contract, (Annexes “A” up
to “A-4”) was entered into by
the plaintiff under the
“MISREPRESENTATION,
PROMISES, FALSE AND
FRAUDULENT
ASSURANCES AND TRICKS”
of the defendants”

Rule 9 — Effect of Failure to Plead

Indiana Petitioner, a school, posted anWhether or not the Yes. Certiorari was the In the case of Lina v. Court
Aerospace v. advertisement in the Manilaorder of default is only plain, speedy and of Appeals discussed the
Commission on Bulletin. Engineering (TPRAME)valid adequate remedy in the remedies available to a
Higher Education inquired whether petitioner had ordinary course of law, defendant declared in
acquired university status. Dr because the default Order default, as follows: (1) a
Vera referred the letter to had improvidently been motion to set aside the order
CHED, and CHED to it’s issued. of default under Section
regional director in Cebu, 3(b), Rule 9 of the Rules of
requesting the office to conduct Court, if the default was
an investigation. As a discovered before judgment
consequence of said Report, could be rendered; (2) a
respondent's Legal Affairs motion for new trial under
Service was requested to take Section 1(a) of Rule 37, if
legal action against petitioner. the default was discovered
Respondent directed petitioner after judgment but while
to desist from using the term appeal is still available; (3) a
University, including the use of petition for relief under Rule
the same in any of its alleged 38, if judgment has become
branches. final and executory; and
(4) an appeal from the
judgment under Section 1,
Rule 41, even if no petition
to set aside the order of
default has been resorted to.
Monzon v. Spouses Relova and PerezWhether or not Yes. The RTC applied the Section 3, Rule 9
Spouses Relova allege that Monzon issued aMonzon is declared in effects of a default order
“(a) Effect of order of
promisory note in their favordefault upon petitioner under
default. — A party in default
worth P600k and secured by a Section 3, Rule 9 of the
shall be entitled to notice of
lot in Tagaytay city. A deed of Rules of Court which
subsequent proceedings but
absolute sale over the parcel of states the “If the
not to take part in the trial.”
land was later executed in favor defending party fails to
of the Perez spouses. The answer within the time Failure to file a responsive
same thing happened with allowed therefor, the court pleading within the
spouses Relova where a shall, upon motion of the reglementary period, and not
promisory note in the amount of claiming party with notice failure to appear at the
200,000 was issued and to the defending party, hearing, is the sole ground for
secured by a lot. A deed of and proof of such failure, an order of default, except
conditional sale over the parcel declare the defending the failure to appear at a pre-
of land was later issued in favor party in default. trial conference wherein the
of spouses Relova. Monzon effects of a default on the part
was indevted to the Coastal of the defendant are
Lending Corporation, who followed, that is, the plaintiff
extrajudically foreclosed the shall be allowed to present
proeprty of Monzon. The evidence ex parte and a
winning bidder in this judgment based thereon may
extrajudicial forecloseure was be rendered against the
Addio properties. Of the amount defendant.
paid by Addio, there was a
residue of roughlyP1.6M
(indebtedness of Monzon was
only around P3.4M while Addio
paid P5M for the property that‘s
why there‘s an excess). This
residue is in the custody of Atty.
Luna as Branch Clerk of Court.
Monzon argues that she has
already fulfilled her obligation to
the spouses via dacion en pago
evidenced by the Deed of
Conditional Sale and the Deed
of Absolute Sale.
Rule 10 — Amended and Supplemental Pleadings

Sps. Lambino v. Petitioner, a lawyer, and his wife Whether or not the No. Before they filed their As a general rule, leave will be
Presiding Judge secured a housing loan from supplemental original complaint, granted to file a supplemental
private respondent BPI. The complaint was petitioners were already complaint which alleges any
interest rate was 19% pe annum proper? aware of the deductions material fact which happened or
payable in 180 monthly came within plaintiff's knowledge
installments. A mortgage loan since the original complaint was
agreement was executed by filed, such being the office of a
petitioners over their property as supplemental complaint. The
security. However, petitioners failed purpose of the rule is that the
to pay the monthly amortizations entire controversy might be
from January 15, 1995 to May 15, settled in one action; to avoid
1995. Private respondent filed a unnecessary litigation; prevent
petition for the extrajudicial delay, unnecessary repetition of
foreclosure of the MLA with the Ex- effort; unwarranted expense of
Officio Sheriff of the RTC of litigants; to broaden the scope of
Valenzuela City and sought to have the issues in an action owing to
the property sold to satisfy the the light thrown on it by facts,
balance of petitioners' loan events and occurrences which
account. The public auction was have accrued after the filing of the
set at 10:00 a.m. on July 11, 1995. original pleading; to bring into
Petitioners filed a complaint for record the facts enlarging or
annulment of the MLA and the charging the kind of relief to which
extrajudicial foreclosure sale with a plaintiff is entitled.
prayer for a Temporary Restraining
Order (TRO) before the RTC of
Valenzuela City.

Philippine Ports After the expiration of the lease whether the CA No. The CA did not err in The Court has emphasized
Authority v. contract of Veterans Shipping erred in ruling that finding that the RTC the import of Section 3, Rule
Gothong Corporation over the Marine the RTC committed committed grave abuse of 10 of the 1997 Rules of Civil
Slip Way in the North Harbor grave abuse of discretion in issuing the Procedure in Valenzuela v.
then petitioner WG&A discretion when it Order dated March 22, Court of Appeals, which
requested respondent PPA for it denied the 2002 denying the states that the Section 3,
to be allowed to lease and admission of the admission of respondent's Rule 10 of the 1997 Rules of
operate the said facility. second amended second amended Civil Procedure amended
Thereafter, then President complaint. complaint. the former rule in such
Estrada issued a memorandum manner that the phrase "or
addressed to the Secretary of that the cause of action or
the Department of defense is substantially
Transportation and altered" was stricken-off and
Communication (DOTC) and the not retained in the new
General Manager of PPA. rules. The clear import of
such amendment in
Section 3, Rule 10 is that
under the new rules, "the
amendment may (now)
substantially alter the
cause of action or
defense."

Wallem A shipment was made of IndianWhether the filing of No. The order of the lower The settled rule is that the
Philippines Soya Bean meal to Manila. Theamended complaint court dismissing the filing of an amended
Shipping, Inc. v. vessel was owned by Consti-against petitioner amended complaint pleading does not retroact to
S.R Farms, Feed with respondent as it’sshould retroact to the against the said defendant the date of the filing of the
ship agent. The shipmentdate of fling of original on ground of prescription original; hence, the statute
arrived in Manila, and wascomplaint was affirmed by this of limitation runs until the
discharged. All throughout the Court. submission of the
entire period of unloading, good amendment. It is true that,
and dair weather condition as an exception, this Court
prevailed. However, there was a has held that an amendment
shortage of the shipment. which merely supplements
and amplifies facts originally
alleged in the complaint
relates back to the date of
the commencement of the
action and is not barred by
the statute of limitations
which expired after the
service of the original
complaint. The exception,
however, would not apply to
the party impleaded for the
first time in the amended
complaint. The rule on the
non-applicability of the
curative and retroactive
effect of an amended
complaint, insofar as newly
impleaded defendants are
concerned, has been
established as early as in
the case of Aetna Insurance
Co. v. Luzon Stevedoring
Corporation. In the said
case, the defendant Barber
Lines Far East Service was
impleaded for the first time
in the amended complaint
which was filed after the
one-year period of
prescription.
Spouses Dionisio Whether or not the No. Here, both the original An amended complaint that
Gorgonio Cruz owned
v. Linsangan Dionisios' and the amended changes the plaintiff's cause
agricultural lands in San
amendment of their complaint required of action is technically a new
Rafael, Bulacan. The
complaint effectively Wilfredo to defend his complaint. Consequently,
tenant, Romualdo,
changed their cause possession based on the the action is deemed filed on
cultivated the lands. When
of action from one allegation that he had the date of the filing of such
Romualdo died, his widow
of ejectment to one stayed on the land after amended pleading, not on
was given permission by
of recovery of Emiliana left out of the the date of the filing of its
Cruz to stay on the property
possession owner's mere tolerance original version. Thus, the
provided that she would
and that the latter had statute of limitation resumes
vacate it upon demand.
demanded that he leave. its run until it is arrested by
Petitioners bought property
the filing of the amended
from Cruz. They trued to get
pleading. The Court
Romualdo’s widow out. acknowledges, however,
that an amendment which
does not alter the cause of
action but merely
supplements or amplifies the
facts previously alleged,
does not affect the
reckoning date of filing
based on the original
complaint. The cause of
action, unchanged, is not
barred by the statute of
limitations that expired after
the filing of the original
complaint. To determine if
an amendment introduces a
different cause of action, the
test is whether such
amendment now requires
the defendant to answer for
a liability or obligation which
is completely different from
that stated in the original
complaint.
Rule 11 — When to File Responsive Pleadings

Sps. Barraza v. Respondent owned a Was the Order of Yes. Respondent judge There is nothing in the
Campos, Jr. lechon business, however respondent judge granted petitioners a Rules which provide,
petitioner used the trade declaring 15-day extension to file directly or indirectly, that
name in the CFO. petitioners in their answer, or up to the interruption of the
default for failure Nov. 18. Instead of filing running of the period
to file their answer their answer, petitioners within which to file an
within the filed a motion to dismiss answer when a motion to
reglementary complaint on Nov. 17, dismiss the complaint is
period provided by or one (1) day before filed and pending before
law correct? the expiration of the the court, refers only to
period as extended by the original period of
the court. This is clearly fifteen (15) days and not
allowed under Sec. 1, to the extension of time to
Rule 16. file the answer as granted
by the court. It may be
true that under Section 4
of Rule 16, if the motion to
dismiss is denied or if the
termination thereof is
deferred, the movant shall
file his answer within the
time prescribed by Rule
11, computed from the
time he received notice of
the denial or deferment,
unless the court provides
a different period. This
Section 1 of Rule 11 in
relation to Section 4 of
Rule 16 allows the
defendant to file his
answer not only within the
original fifteen (15) days
period but also within "a
different period (as) fixed
by the court. "
David v. A complaint for accounting, Did the RTC No. Thus, petitioner Petitioners voluntary
Guttierez-Fruelda reconveyance, and commit grave waived any defect in the appearance was
damages with prayer for abuse of service of summons by equivalent to service of
preliminary attachment discretion in publication or even summons. It has cured
against petitioner. It was denying want of process any alleged defect in the
claimed that petitioner petitioner's motion because for the RTC to service of summons.
fraudulently exceeded his to lift order of validly act on his Petitioner's motions were
special power of attorney to default? motions, it necessarily not motions to dismiss the
put land in his name and acquired jurisdiction complaint on the ground
did not remit and account over his person. of lack of jurisdiction over
any money from the Unfortunately, his claim his person. On the
transaction involving their that he has meritorious contrary, the motions
lands. defenses is invoked the RTC's
unsubstantiated. He did jurisdiction while seeking
not even state what the following affirmative
evidence he intends to reliefs: to grant extension,
present if his motion is deny the motion to
granted. declare petitioner in
default and lift the order of
default.
Rule 13 — Filing and Service of Pleadings, Judgments and Other Papers

Republic v. Whether or not the Yes. The present Section 4, Rule 65 of the
Caguioa A petitioner for TRO and procedural rules petition was filed within Rules of Court is clear on
mandatory injunction was were complied the reglementary this point — "In case a
filed against the Finance with? period. Contrary to the motion for
Secretary, seeking to nullify private respondents' reconsideration or new
section 6 of RA 9334. position, the 60-day trial is timely filed, whether
period within which to such motion is required or
file the petition not, the sixty (60) day
for certiorari is counted period shall be counted
from the Republic's from notice of the denial
receipt of the July 5, of said motion."
2006 order denying the
latter's motion for
reconsideration.

UP v. Sison A construction agreement Whether the SC The service of the denial "If any party has appeared
of the motion for by counsel, service upon
was entered into with should give due reconsideration upon Atty. him shall be made upon his
petitioner and Stern course to UP’s Nolasco of the UPLB counsel or one of them,
Builders Corp for a building petition for review Legal Office was invalid unless service upon the
in UPLB. UP did not pay all in the name of and ineffectual because party himself is ordered by
three buildings and the last equity in order to he was admittedly not the the court. Where one
counsel of record of UP. counsel appears for several
one was disallowed by reverse or modify The rule is that it is on the parties, he shall only be
COA. Stern Builders sued the adverse counsel and not the client entitled to one copy of any
UP to collect the unpaid judgment against that the service should be paper served upon him by
billing and to recover it despite its made. the opposite side.”
damages. finality?

Rule 14 — Summons, Sections 1 to 14

Cathay Metal A joint venture was entered Whether petitioner cannot use The Rules of
Corp. v. Laguna into with respondent and respondent was respondent's failure to Court provides that
West Multi- farmer beneficiaries in validly served with amend its Articles of notices should be sent to
Purpose CAvite. While respondent summons or Incorporation to reflect the enumerated officers.
Cooperative was negotiating, petitioner notice of the its new address as an The Cooperative
entered into an irrevocable hearing? excuse from sending or Code provisions on
exclusive right to buy attempting to send to notices cannot replace the
contracts with the same respondent copies of rules on summons under
farmers, where the farmer the petition and the the Rules of Court. Rule
beneficiaries committed summons. 14, Section 11 of the
themselves to sell to Rules of Court provides
petitioner their agricultural an exclusive enumeration
properties upon of the persons authorized
conversion. Respondent to receive summons for
caused the annotation of its juridical entities. These
adverse claim on the persons are the juridical
beneficiaries certificates of entity's president,
title. Contracts of sale of managing partner,
the properties were general manager,
executed. TCTs were also corporate secretary,
issued. A consolidated treasurer, or in-house
petition for cancellation of counsel. The enumeration
the adverse claims were in Section 11 of Rule 14 is
filed. The postman issued a exclusive. Service of
certification which said that summons upon persons
the reason for the return other than those officers
was that the cooperative enumerated in Section 11
was inexistent. Therefore, is invalid. Even substantial
petition was tried to be filed compliance is not
personally by petitioner. sufficient service of
This failed for the same summons. This provision
reason. The RTC declared of the rule does not limit
that the service, by service to the officers'
registered mail, was places of residence or
effected offices. If summons may
not be served upon these
persons personally at their
residences or offices,
summons may be served
upon any of the officers
wherever they may be
found.
Rapid Realty and A declaration of nullity of Whether Yes. It is settled that if Prescinding from the
Development subdivision plans was filed respondents, in there is no valid service of foregoing, it is thus clear
Corp. v. Villa by petitioner. Summons filing summons, the court can that: (1) Special
were attempted, but the the first Motion to still acquire jurisdiction appearance operates as
first one, which was a Lift the Order of over the person of the an exception to the
defendant by virtue of the
personal service, failed. Default, voluntarily latter's voluntary
general rule on voluntary
Zapanta, the court process submitted appearance pursuant to appearance;
server, then resorted to themselves to the Sec. 20, Rule 14 of the (2) Accordingly, objections
substituted service, serving jurisdiction of the Rules of Court. to the jurisdiction of the
to the househelp who did court? court over the person of
know acknowledge receipt. the defendant must
be explicitly made; and
(3) Failure to do
so constitutes voluntary
submission to the
jurisdiction of the court,
especially in instances
where a pleading or
motion seeking affirmative
relief is filed and
submitted to the court for
resolution.
Green Star Universal Robina Corp Whether or not the No. It is a well- NURC maintains that the
Express, Inc. v. owned a van, which got summons was established rule that the RTC did not acquire
Nissin-Universal into a vehicular accident properly served on rules on service of jurisdiction over it as the
Robina Corp., which resulted to the death NURC, vesting the summons upon a summons was received
of the van’s driver. A case trial court with domestic private by its cost accountant,
was filed for reckless jurisdiction juridical entity must be Tinio. It argues that under
imprudence resulting in strictly complied with. Section 11, Rule 14 of
homicide, and petitioner Otherwise, the court the 1997 Rules of Court,
sent a demand letter to cannot be said to have which provides the rule on
respondent. The criminal acquired jurisdiction service of summons upon
case was dismissed. over the person of the a juridical entity, in cases
defendant. where the defendant is a
domestic corporation like
NURC, summons may be
served only through its
officers. Here, Tinio, a
member of NURC's
accounting staff, received
the summons on January
22, 2004.

Rule 15 — Motions
Dela Reyes v. A favorable decision was Is a hearing At any rate, the trial As a general rule, all
Ramnani rendered for respondent in needed for the court gave petitioner an written motions should be
a civil case. A write of motion? opportunity to oppose set for hearing under
execution was then issued the subject motion as in Section 4, Rule 15 of
by the RTC. The branch fact he filed a the Rules of Court
sheriff then conducted a Comment/Opposition on excepted from this rule
public bidding. Respondent March 1, 2004 before are non-litigious motions
was the highest bidder and the trial court. Petitioner or motions which may be
a write of possession was cannot, therefore, acted upon by the court
issued. The respondent validly claim that he was without prejudicing the
filed an order to the sheriff denied his day in court rights of the adverse
to execute the final party. As already
certificate of sale in her discussed, respondent is
favor. This was opposed by entitled to the issuance of
petitioner. the final certificate of sale
as a matter of right and
petitioner is powerless to
oppose the same. Hence,
the subject motion falls
under the class of non-
litigious motions.
Preysler v. Manila A complaint for forcible Whether the RTC No. In this case, the “The ordinary motion day
Soutcoast Dev. entry was filed by petitioner was correct in petitioner's Omnibus is Friday. Hence, the
Corp against the respondent. dismissing Motion was set for notice should be served
petitioner's hearing on 12 by Tuesday at the latest,
Omnibus Motion November 2004. Thus, in order that the
for allegedly failing to comply with the requirement of the three
to comply with the notice requirement, days may be complied
three-day notice respondent should have with. If notice be given by
requirement? received the notice of ordinary mail, it should be
the hearing at least actually received by
three days before 12 Tuesday, or if not claimed
November 2004, which from the post office, the
is 9 November 2004. date of the first notice of
the postmaster should be
at least five (5) days
before Tuesday.” Section
4 of Rule 15 provides that
"[e]very written motion
required to be heard and
the notice of the hearing
thereof shall be served in
such a manner as to
ensure its receipt by the
other party at least three
(3) days before the date
of the hearing, unless the
court for good cause sets
the hearing on shorter
notice." Thus, the date of
the hearing should be at
least three days after
receipt of the notice of
hearing by the other
parties.
PNB v. Deang A dacion en pago Whether the CA Yes. In requesting for a The intention to delay was
Marketing Corp arrangement in an was correct in 30-day extension or rather obvious.
agreement forged by declaring until June 11, 2006 to Petitioner's Motion for
petitioners transformed petitioner in file answer, petitioner Extension of Time to File
respondent’ outstanding default? apparently reckoned the Answer was laden with
loan obligations into a date from which the glaring lapses. It filed the
seven year term loan. extension would start Motion for Extension,
Respondents filed before on May 12, 2006, which however, via a private
the RTC a Complaint. was not the last day of courier (without any
Summons was served on the 15-day period explanation for availing for
petitioner. Respondents sought to be such mode) on May 14,
filed a Motion to Declare extended, it being May 2006, which was received
[Petitioner] in Default, 5, 2006. By by the RTC on May 15,
which they set for hearing. computation, petitioner 2006 or ten days late. A
On even date, the trial actually sought more motion for extension of
court received petitioner's than 30 days, contrary time to file a pleading
Motion for Extension of to the period of must be filed before the
Time [30 days up to June extension it purportedly expiration of the period
11, 2006] to File Answer. requested. The sought to be
Eight days prior to the counting of the period extended. The court's
slated hearing of was erroneous, even if discretion to grant a
respondents' Motion to one uses the material motion for extension is
Declare [Petitioner] in dates alleged by conditioned upon such
Default, the trial court petitioner. No motion's timeliness, the
issued an Order denying satisfactory reason was passing of which renders
said motion and granting adduced to justify the the court powerless to
petitioner's Motion for tardiness of the Answer entertain or grant it. Since
Extension of Time to File and no compelling the motion for extension
Answer. To the trial court's reason was given to was filed after the lapse of
Order respondents filed a justify its admission. the prescribed period,
Motion for there was no more period
Reconsideration. In the to extend. Petitioner was
meantime, petitioner filed not candid enough to aver
its Answer to the Complaint in the Motion for
on May 25, 2006. Extension that the period
had lapsed, as it still toyed
with the idea that it could
get away with it. The
allegations therein were
crafted as if the said
motion was timely filed.
Rule 16 — Motion to Dismiss

Conejos v. According to petitioners, Whether the CA The dismissal of the Under the Rules, the only
Boconegra respondents were served erred acted with complaint on the ground grounds the court could
with summons and legal grave abuse of of lack of jurisdiction take cognizance of, even
processes through Atty discretion? over the person of the if not pleaded in the
Piazarri, whose office respondents after they motion to dismiss or
address is at 2830 Juan had voluntarily answer, are: (a) lack of
Luna St, Tondo Manila. appeared before the jurisdiction over the
Atty Piazarro wrote a trial court clearly subject matter; (b)
demand letter on behalf of constitutes grave abuse existence of another
respondents to vacate the of discretion amounting action pending between
land, stating that to lack of jurisdiction or the same parties for the
respondents owned it. in excess of jurisdiction same cause; and (c) bar
on the part of the RTC. by prior judgment or by
Quite apart from their statute of limitations.
voluntary appearance,
respondents'
Supplemental Motion to
Dismiss and Second
Supplemental Motion to
Dismiss were clearly in
violation of Rule 15,
Section 8 in relation to
Rule 9, Section 1 of the
Rules
Panganiban v. A sublease and dealer Whether the CA No. The requirement that The requirement that a
Pilipinas Shell agreement was entered erred in affirming a motion to dismiss motion to dismiss should
Corp., between Panganiban and RTC-Makati's should be filed within the be filed within the time for
Shell, where Shell leased dismissal of Civil time for filing the answer filing the answer is not
to Panganiban a gasoline Case No. 95-1010 is not absolute. absolute. Even after an
station for a period which on motion of shell answer has been filed, a
would terminate at the on the ground defendant can still file a
same time as Shell’s lease. of litis motion to dismiss on the
As Shell was not the owner pendentia which following grounds: (1) lack
of the lot, and was leasing was filed long of jurisdiction, (2) litis
it under a lease agreement, after Shell had pendentia (3) lack of
it notified Panganiban that filed its answer? cause of action, and (4)
the agreement was expiring discovery during trial of
and that Panganiban evidence that would
should wind up her constitute a ground for
business affairs, but as she dismissal. Litis
had believed it would pendentia is also one of
expire later, she still the grounds that authorize
occupied the lots and paid a court to dismiss a
rentals, although Shell did case motu proprio. In this
not accept. case, the bona
fide existence of litis
pendentia is beyond
dispute.
Universal Robina URC and Lim entered into Whether the trial No. Implicit from Sec. 1, Thus, a court may not
Corp. v. Lim a contract of sale where court may Rule 9 of the Rules of dismiss an action motu
URC sold to Lim grocery dismiss motu Court is that improper proprio on the ground of
products totaling proprio petitioner's venue not impleaded in improper venue as it is not
P808,059.88. After complaint on the the motion to dismiss or one of the grounds
tendering partial payments, ground of in the answer is wherein the court may
respondent refused to improper venue? deemed waived. dismiss an action motu
settle his obligation despite proprio on the basis of the
petitioner's repeated pleadings. In Dacoycoy v.
demands. IAC, this Court held that a
trial court may not motu
proprio dismiss a
complaint on the ground
of improper venue. The
court may only dismiss an
action motu proprio in
case of lack of jurisdiction
over the subject
matter, litis pendentia, res
judicata and prescription.
In the instant case,
respondent, despite
proper service of
summons, failed to file an
answer and was thus
declared in default by the
trial court. Verily, having
been declared in default,
he lost his standing in
court and his right to
adduce evidence and
present his defense,
including his right to
question the propriety of
the venue of the action.

Galindo v. Heirs Galindo possessed a lot of Whether the RTC Yes. Section 1, Rule 3 of An unauthorized
of Roxas land and paid taxes committed a grave the Rules of complaint does not
thereon. When he died, his abuse of its Court provides that only produce any legal
heirs sold their rights and discretion? persons or juridical
persons or entities effect. Section 1, Rule 3 of
interests over the land to the Rules of
authorized by law may be
Roxas. Urbano, one of the parties in a civil action. Court provides that only
heirs of Galindo, was a
persons or juridical
minor then, so he did not
persons or entities
sign the document
(Documento de authorized by law may be
Compromiso) but he parties in a civil
executed an affidavit action. Section 4, Rule
signifying his conformity to 8 of the said Rules further
the sale executed by his provides that facts
siblings; (4) Urbano later showing the capacity of a
ratified the sale by party to sue or be sued or
executing an affidavit the authority of a party to
signifying he was sue or be sued in a
agreeable thereto. The CFI representative capacity
ordered the Galindo heirs must be averred in the
to execute a deed of complaint. In order to
absolute sale in favor of the maintain an action in a
heirs of Roxas. When court of justice, the
Urbano died intestate, his
plaintiff must have an
heirs executed an
actual legal existence,
extrajudicial settlement of
estate where they that is, he or she or it
adjudicated unto must be a person in law
themselves as owners the and possessed of a legal
undivided rights and entity as either a natural
interests of Urbano over or an artificial person, and
the lot. no suit can lawfully be
prosecuted in the name of
that person. The party
bringing suit has the
burden of proving the
sufficiency of the
representative character
that he claims. If a
complaint is filed by one
who claims to represent a
party as plaintiff but who,
in fact, is not authorized to
do so, such complaint is
not deemed filed and the
court does not acquire
jurisdiction over the
complaint.
Chu v. Sps. 5 registered lands were Was the case Yes. The petitioners A compromise
Cunanan sold to Cunanan for P5m barred by res were not at liberty to agreement is a contract
by Sps Chu. It was judicata although split their demand to whereby the parties, by
stipulated that the the compromise enforce or rescind making reciprocal
ownership of the lots were agreement did not the deed of sale with concessions, avoid a
still with the Chus. Such expressly include assumption of litigation or put an end to
would be transferred to Benelda Estate as mortgage and to one already commenced.
Cunan only upon complete a party and prosecute piecemeal or The intent of the parties to
payment of the total although present only a portion of settle all their claims
consideration in the terms the compromise the grounds upon which against each other is
of the deed of sale, A agreement made a special relief was expressed. Apparently,
special power of attorny no reference to sought under the deed the petitioners were guilty
was issued authorizing the lots now of sale with assumption of splitting their single
Cunanan to borrow P5, registered in of mortgage, and then cause of action to enforce
161, 090 from any banking Benelda Estate's to leave the rest to be or rescind the deed of
institution. The proceeds of name? presented in another sale with assumption of
the net balance would then suit; otherwise, there mortgage.
be delivered. But Cunanan would be no end to
transderred the title of the litigation. Their splitting
five lands to her name violated the policy
without the knowledge of against multiplicity of
the spouses Chu, and then suits, whose primary
later to spouses Carlos. objective was to avoid
unduly burdening the
dockets of the courts.
Their contravention of
the policy merited the
dismissal of Civil Case.

Manila Bankers Yes. Section 48 serves The self-regulating feature


Life Insurance A life insurance policy was Whether the a noble purpose, as it of Section 48 lies in the
Corp. v. Aban taken out by Sotero. In it, courts below were regulates the actions of fact that both the insurer
her niece respondent was correct in applying both the insurer and the and the insured are given
beneficiary. Petitioner Sec. 48 of the insured. Under the the assurance that any
issued the insurance policy. Insurance Code? provision, an insurer is dishonest scheme to
Sotero died. However, given two years — from obtain life insurance
upon investigation, the effectivity of a life would be exposed, and
petitioner found that Sotero insurance contract and attempts at unduly
did not personally apply for while the insured is denying a claim would be
insurance, and it was alive — to discover or struck down. Life
respondent who filed the prove that the policy is insurance policies that
insurance application. void ab initio or is pass the statutory two-
rescindible by reason of year period are essentially
the fraudulent treated as legitimate and
concealment or beyond question, and the
misrepresentation of the individuals who wield
insured or his agent. them are made secure by
After the two-year the thought that they will
period lapses, or when be paid promptly upon
the insured dies within claim. Section 48
the period, the insurer contributes to the stability
must make good on the of the insurance industry,
policy, even though the and prevents a situation
policy was obtained by where the insurer
fraud, concealment, or knowingly continues to
misrepresentation. This accept annual premium
is not to say that payments on life
insurance fraud must be insurance, only to later on
rewarded, but that deny a claim on the policy
insurers who recklessly on specious claims of
and indiscriminately fraudulent concealment
solicit and obtain and misrepresentation,
business must be such as what obtains in
penalized, for such the instant case.
recklessness and lack
of discrimination
ultimately work to the
detriment of bona
fide takers of insurance
and the public in
general.
Vitangcol v. New A land in Calamba, LAguna, Whether or not the Yes. The trial court acted The test of sufficiency of
Vista Properties, was owned by both Maria and Amended within its discretion in facts alleged in the
Inc., Clemente Alipit. They then Complaint of considering said SPA complaint constituting a
executed a special power of Vitangcol states a relative to the motion to cause of action lies on
attorney making Milagros A cause of action? dismiss the amended whether or not the court,
De Guzman their attorney-in- complaint, as Vitangcol admitting the facts alleged,
fact to sell their property. De duly submitted that piece could render a valid verdict
Guzman then executed a of document in court in in accordance with the
deed of absolute sale to New the course of the June 7, prayer of the complaint. And
Vista Properties. When New 2004 hearing on the to sustain a motion to
Vista leanred that the parcel motion to dismis The trial dismiss for lack of cause of
of land was claimed by court, however, erred in action, it must be shown that
petitioners Vitangcol by a ruling that, taking said the claim for relief in the
deed of sale between Maria SPA into account, the complaint does not exist,
and Vitangcol. amended complaint stated rather than that a claim has
no cause of action. been defectively stated, or is
Indeed, upon a ambiguous, indefinite, or
consideration of the uncertain.
amended complaint, its
annexes, with the June
18, 1989 SPA thus
submitted, the Court is
inclined, in the main, to
agree with the appellate
court that the amended
complaint sufficiently
states a cause of action.
In a motion to dismiss for
failure to state a cause of
action, the focus is on the
sufficiency, not the
veracity, of the material
allegations.
Cua v. Wallem Cua sought payment of 2 Whether or not Cua’s claim for payment A review of the pleadings
Phils. Shipping million pesos as reparation for Cua's claim for of damages has not submitted by the
Inc damage to his shipment of payment of prescribed due to the respondents discloses that
soybean against respondent. damages against admitted fact that there they failed to specifically
the respondents was indeed an extension deny Cua's allegation of an
has prescribed? of their agreement. Cua agreement extending the
timely filed his claim period to file an action to
before the trial court and November 12, 1990.
the CA failed to Wallem's motion to dismiss
appreciate the admissions simply referred to the fact
made by the respondents that Cua's complaint was
in their pleadings that filed more than one year
negate a finding of from the arrival of the
prescription of Cua's vessel, but it did not contain
claim. It was ruled that the a denial of the extension.
action has not prescribed Since the COGSA is the
because jurisprudence applicable law, the
recognized the validity of respondents' discussion to
agreements extending support their claim of
such period pursuant to prescription under Article
COGSA. The allegation of 366 of the Code of
an agreement extending Commerce would, therefore,
the period to file an action not constitute a refutation of
in Cua's complaint is a Cua's allegation of
material averment that, extension.
under Section 11, Rule 8
of the Rules of Court,
must be specifically
denied by the
respondents; otherwise,
the allegation is deemed
admitted.
Swedish Match v. Petitioner had three Whether or not the Yes. The Court of Appeals Respondents' attempt to
CA, subsidiary corporations in The appellate court erred procedurally in motu prove the alleged verbal
Philippines. Phimco and Phils erred in reversing propio dismissing the acceptance of their US$36
are some of them. Petitioner the trial court's complaint in its entirety for million bid becomes futile in
decided to sell them, and decision? “lack of a valid cause of the face of the overwhelming
several corporations tried to action: without the benefit evidence on record that
get Phimco, one of them of a full blown trial and on there was in the first place
being respondent. A case was the mere motion to no meeting of the minds with
filed against petitioners to sell dismiss. respect to the price.
the Phimco, but petitioner
alleged that there is no cause
of action as there was no
perfected contract between
them.
California and A cargo of soyben meal Whether or not the Yes. The trial court Where a preliminary hearing
Hawaiian Sugar arrived in manila via the trial court committed committed grave abuse of appears to suffice, there is
Co. v. C.F Sharp vessel MV Sugarl Islander. grave abuse of discretion when it denied no reason to go on to trial.In
& Co., Such was consigned to discretion when it a preliminary hearing on a the present case, the trial
several consignees, one of denied petitioners' simple issue of fact that court did not categorically
them being the Metro Manila Motion for a could have possibly resolve petitioners' Motion to
Feed Millers Association. The Preliminary settled the entire case. Dismiss, but merely deferred
cargo from the vessel to Hearing? One reason why dockets resolution thereof. Section 6,
barges were discharged on of trial courts are clogged Rule 16 of the 1997 Rules,
Nov 30. Respondent says that is the unreasonable specifically provides that a
when the cago was weighed, refusal to use a process preliminary hearing on the
a shortage was discovered. or procedure, like a affirmative defenses may be
Petitioners refused to settle. motion to dismiss, which allowed only when no
is designed to abbreviate motion to dismiss has been
the resolution of a case. filed. Section 6, however,
must be viewed in the light
of Section 3 of the same
Rule, which requires courts
to resolve a motion to
dismiss and prohibits them
from deferring its resolution
on the ground of
indubitability. The ground
invoked may still be pleaded
as an affirmative defense
even if the defendant's
Motion to Dismiss has been
filed but not definitely
resolved, or if it has been
deferred as it could be under
the pre-1997 Rules..
Sps. Rasdas v. A parcel of land in Isabela is Whether or not the Yes. The Supreme Court It was held that it is the
Estenor the subject of this case. complaint is barred observed that the causes second kind of res judicata,
Respondent filed a complaint by res judicata? of action in the two cases conclusiveness of
for recovery of owndership of involved were so glaringly judgment," that barred the
the land, asserting that he similar that it had to affirm instant complaint. Any right,
was the owner. The RTC the dismissal of the fact, or matter in issue
ruled in favor of petitioners. second case by virtue of directly adjudicated or
Respondent then appealed the "bar of former necessarily involved in the
the RTC decision before the judgment rule." determination of an action
Court of Appeals, which before a competent court in
reversed the judgement of the which judgment is rendered
RTC. Petitioners were on the merits is conclusively
ordered to vacate the land. settled by the judgment
therein and cannot again be
litigated between the parties
and their privies whether or
not the claim, demand,
purpose, or subject matter of
the two actions is the same.
Tan v. Tan Petitioner and respondent Whether or not the Yes. We upheld the It as decided that it was
were married in Manila annulment decided judgment of nullity of the executory when the husband
Cathedral. They were blessed by the trial court marriage even if it was failed to appeal during the
with two sons. Justin was attained finality? based solely on evidence reglementary period. The
born in Canada and Russel presented by therein husband claimed that the
was born in The Philippines. private respondent. The decision of the trial court
Petitioner then filed a case for alleged negligence of was null and void for
annulment under Article 36. counsel resulting in violation of his right to due
The trial court issued a partial petitioner's loss of the process. He argued he was
judgement approving the right to appeal is not a denied due process when,
compromise and declared the ground for vacating the after failing to appear on two
marriage void. On 4 trial court's judgments. scheduled hearings, the trial
November 2004, petitioner court deemed him to have
filed a motion to dismiss and waived his right to present
a motion for reconsideration. evidence and rendered
She claimed she was no judgment based solely on
longer interested in the suit. the evidence presented by
Petitioner stated that the private respondent.
circumstances in her life had
led her to the conclusion that
withdrawing the petition was
for the best interest of the
children. She prayed that an
order be issued vacating all
prior orders and leaving the
parties at the status quo ante
the filing of the suit.
Aldersgate A case was filed against Whether or not the Yes. As this case Moreover, the Court finds no
College, Inc. v. respondents before the SEC. Regional Trial Court involves an intra- justification for the dismissal
Gauuan When the SEX was erred in dismissing corporate dispute, the of the case based on the
reorganized pursuant to RA the case? motion to dismiss is mere issuance of a board
8799, the case was undeniably a prohibited resolution by the incumbent
transferred to the RTC of pleading. members of the Board of
Nueva Vizcaya for further Trustees of petitioner
proceedings. The case was corporation recommending
sought to be dismissed on the its dismissal, especially
complaint or the issuance of a considering the various
summary judgement. The issues raised by the parties
RTC denied the motion on the before the court a quo.
ground that "there are several Hence, the RTC should not
issues raised which would still have entertained, let alone
need the presentation of have granted the subject
evidence to determine the motion to dismiss.
rights of the parties."
Rule 17 — Dismissal of Actions

Quintos v. Petitioners and respondents Whether or not the No. . The Supreme Court This is not to say, however,
Nicolas are siblings. Their parents respondents' held that Art. 494, as that the action for partition
were owners of the subject counterclaim for cited, is an exception to will never be barred by res
property, whcih was located in partition is already Rule 17, Sec. 3 of the judicata. There can still be
Quezon city. When the barred by laches or Rules of Court to the res judicata in partition
parents died, they left to their res judicata? effect that even if the cases concerning the same
ten children the ownership order of dismissal for parties and the same subject
over the subject property. failure to prosecute is matter once the respective
Respondent siblings brought silent on whether or not it shares of the co-owners
an action against petition is with prejudice, it shall have been determined with
be deemed to be without finality by a competent court
prejudice with jurisdiction or if the
court determines that
partition is improper for co-
ownership does not or no
longer exists.
Samson v. Fiel- Benedicta and Marcial Whether or not the Yes. The RTC dismissed The remedy to obtain
Macaraig Samson obtained a loan of case was dismissed the case with prejudice for reversal or modification of
10k pesos from Far East with prejudice? failure to prosecute for an the judgment on the merits
Bank and Trust Co. The loan unreasonable length of is appeal.The availability of
was secured by a real estate time, pursuant to Section the right to appeal precludes
morgage over four parcels of 3, Rule 17 of the Rules of recourse to the special civil
land in Marikina. When Court. The RTC Order action for certiorari. The
petitioners did not comply with dated 20 June 2003 was a RTC Order subject of the
the terms of the loan, FEBTC final judgment which petition was a final judgment
filed for extra judicial disposed of the case on which disposed of the case
foreclosure of the real estate the merits. This was even on the merits; hence, it was
mortgage at the RTC. These clarified in the subsequent a subject for an ordinary
were sold to FEBTC. RTC Order of 22 appeal, not a petition for
December 2003 which certiorari.
denied petitioners' motion
for reconsideration
wherein the lower court
stated that: "Therefore,
the dismissal was 'with
prejudice' or a dismissal
that had the effect of
adjudication upon the
merits in accordance with
Section 3, Rule 17 of the
Rules of Court."
Eloisa A real estate mortgage over Whether or not the Yes. Petitioners Section 3, Rule 17 of the
Merchandising properties located in Makati dismissal is proper? themselves did nothing to 1997 Rules of Civil
Inc. v. Trebel were executed by petitioner get the case moving for Procedure stated that the
International Inc., on behalf of respondent. A nine months failure on the part of the
continuing suretyship was plaintiff, without any
executed in favor of BDO to justifiable cause, to comply
secure the credit with any order of the court or
accomondation. BDO the Rules, or to prosecute
inisiated foreclosure his action for an
proceedings. Petitioners filed unreasonable length of time,
a complaint for annulment of may result in the dismissal
real estate mortgage. BDO of the complaint either motu
filed a motion to dismiss on proprio or on motion by the
the ground of lack of cause of defendant. However, despite
action, which can be the trial court's leniency and
determined from the facts admonition, petitioners
alleged in the complaint. continued to exhibit laxity
and inattention in attending
to their case.
Heirs of Favis, Sr. Dr. Favis was married to Whether or not the No. The Court of Appeals The appellate court
v. Gonzales Captolina Aguilar. They had appellate court may ignored the facts of the committed egregious error in
seven children. When dismiss the order of case that clearly dismissing the complaint.
Captilona died, Dr Favis took dismissal of the demonstrated the refusal The facts of the case show
a common law wife. They had complaint for failure by the respondents to that compromise was never
one child. Finally, he married to allege therein compromise. Instead it an option insofar as the
his common law wife, and he that earnest efforts ordered the dismissal of respondents were
acknowledged their son as towards a petitioner's complaint on concerned. The impossibility
legitimate. When Dr Favis compromise have the ground that it did not of compromise instead of
died, he executed a deed of allege what in fact was litigation was shown not
fonation transferring and been made? shown during the trial. alone by the absence of a
conveying properties in favor The error of the Court of motion to dismiss but on the
of his grandchildren with the Appeals is patent. respondents' insistence on
common law wife. the validity of the donation in
their favor of the subject
properties Respondents not
only confined their
arguments within the pre-
trial order; after losing their
case, their appeal was
based on the proposition
that it was error for the trial
court to have relied on the
ground of vitiated consent
on the part of Dr. Favis.
Ching v. Cheng, Antonio Ching owned P Wing Whether or not the No. When respondents Rule 17 of the Rules of Civil
Enterprises. Ching alleged "two-dismissal rule” filed the second case, Procedure governs
that he was the only child of is applicable in this they were merely re-filing dismissals of actions at the
Ramon with his common law case? the same claim that had instance of the plaintiff.
wife. The wife disputed this. been previously dismissed Hence, the "two-dismissal
She states that even if on the basis of lack of rule" under Rule 17, Section
Ramon’s certificate alleges jurisdiction. When they 1 of the Rules of Civil
that he was Antonio’s moved to dismiss the Procedure will not apply if
illegitimage child, he was just second case, the motion the prior dismissal was done
adopted. Joseph and Jaime, to dismiss can be at the instance of the
on the other hand, claim to be considered as the first defendant. The dismissal of
his illegitimage children with dismissal at the plaintiff's the first case was done at
the maid. LAter, Ching was instance. The trial court the instance of the
murdered. Ramon induced dismissed the first case by defendant under Rule 16,
the maid and her children to granting the motion to Section 1 (b) of the Rules of
sign an agreement that dismiss filed by the Civil Procedure.
waived the estate. defendants.
Rule 18 — Pre Trial
Paranaque Kings Catalina Santos entered into Whether or not the Yes. the RTC was able to The trial court has the
Enterprises v. a contract of lease with Chua. CA correctly upheld explain to the satisfaction discretion on whether to
Santos, This lease was over land in the RTC's denial of of the Court that the grant or deny a motion to
Paranaque. Chua was given petitioner's Motion postponement of the pre- postpone and/or reschedule
the first option to buy. Chua to Cancel Pre-Trial? trial. the pre-trial conference in
then caused the construction accordance with the
of a 6 door commercial circumstances obtaining in
complex on the leased the case. This must be so as
premise. However, by reason it is the trial court which is
of business reverses, he was able to witness firsthand the
constrained to assign his events as they unfold during
rights to Bing. Bing assumed the trial of a case.
all oblications under the lease Postponements, while
contract with Santos. Lee permissible, must not be
executed a deed of countenanced except for
assignment over the leased clearly meritorious grounds
premises. Petitioner then filed and in light of the attendant
a complaint before the RTC circumstances.
against Santos. Respondents
moved to dismiss.
Alarcon v. CA Roberto Alarcon filed a Whether or not No. All of the matters From the foregoing, the
complaint for the annulment there was intrinsic taken up during the pre- admissions were clearly
of a deed of sale agaisnt or collateral fraud trial, including the made during the pre-trial
respondents. Alacron says attendant, which stipulation of facts and the conference and, therefore,
that before he went to Brunei would justify the admissions made by the conclusive upon the parties
to work, he left with his father setting aside of the parties are required to be making it. The purpose of
a special power of attorney to partial decision? recorded in a pre-trial entering into a stipulation of
administer, mortgage, or sell order. Juani cannot now facts or admissions of facts
his properties in Bulacan. claim that he was denied is to expedite trial and to
Using the SPA, a cportion of his day in court when relieve the parties and the
the land was sold to judgment was rendered court, as well, of the costs of
respondents. on the basis of their proving facts which will not
(Juani, Baluyot and Sulit) be disputed on trial and the
counsels' admission truth of which can be
ascertained by reasonable
inquiry. The parties
themselves are required to
attend or their representative
with written authority from
them in order to arrive at a
possible amicable
settlement, to submit to
alternative modes of dispute
resolution, and to enter into
stipulations or admissions of
facts and documents.
Tiu v. Middleton A complaint for recovery of Whether or not a Yes. The court proceeded It was emphasized that pre-
possession of real property judge can exclude a with the trial without trial and its governing rules
was filed against petitioner. witness whose modifying the Order. In are not technicalities which
Before the commencement of name and synopsis the same vein, the parties may ignore. As
the trial, the court a quo sent of testimony were respondents did not earlier stated, pre-trial is
a notice of Pre-trial not included in the challenge it before the essential in the simplification
Conference stating in part: pre-trial brief? trial. Neither did they and the speedy disposition
"The parties are warned that invoke the power of the of disputes. In light of the
witnesses whose names and trial court to compel the objectives of a pre-trial and
addresses are not submitted petitioner to submit the the role of the trial court
at the pre-trial may not be names of his witnesses therein, it is evident that
allowed to testify at the and summaries of their judges have the discretion to
trial . . . ." In his Pre-trial Brief, testimonies. By their exclude witnesses and other
petitioner averred that he silence, respondents pieces of evidence not listed
would be presenting six acquiesced to the Pre-trial in the pre-trial brief, provided
witnesses, but he did not Order allowing the the parties are given prior
name them. presentation of petitioner's notice to this effect. Indeed,
unnamed witnesses. the court and the parties
must pay attention not only
to the pre-trial briefs, but
also to the pre-trial order.
Respondents' failure to
challenge the pre-trial brief
submitted by petitioner
before the trial nor invoke
the power of the trial court to
compel the petitioner to
submit the names of his
witnesses and summaries of
their testimonies is fatal to
their case. The provision in
the Pre-trial Order allowing
petitioner to present six
witnesses "shall control the
subsequent course of
action."
Saguid v. CA, Gina was married. But then Whether or not the Yes. Under Section 6, Rule 18 of
she petitioner and separated trial court erred in the 1997 Rules of Civil
from her husband. They allowing private Procedure, the failure of the
cohabited as husband and respondent to defendant to file a pre-trial
wife. They had no children. present evidence ex brief shall have the same
When their relationship went parte? effect as failure to appear at
bitter, she went to work as an the pre- trial, i.e., the plaintiff
entertainer in Japan. The may present his evidence ex
couple then separated. parte and the court shall
Respondent filed a complaint render judgment on the
for Partition and Recovery of basis thereof.
Personal Property with
Receivership against the
petitioner with the Regional
Trial Court of Boac,
Marinduque. She alleged that
from her salary of $1,500.00 a
month as entertainer in
Japan, she was able to
contribute P70,000.00 in the
completion of their unfinished
house. She prayed that she
be declared the sole owner of
these personal properties and
that the amount of
P70,000.00, representing her
contribution to the
construction of their house, be
reimbursed to her. petitioner
claimed that the expenses for
the construction of their house
were defrayed solely from his
income as a captain of their
fishing vessel.
Tolentino v. Respondents allege that they Whether or not No. There was no lack of From the foregoing, the
Laurel are registered owners of a Petitioners were due process claimed by failure of a party to appear at
parcel of land in Quezon. denied their day in petitioners. On the the pre-trial has adverse
However, petitioners have court? contrary, petitioners were consequences. If the absent
been in actual possession of given more than ample party is the plaintiff, then his
the Western portion of the opportunity to be heard case shall be dismissed. If it
said property which they had through counsel. is the defendant who fails to
tried to develop into appear, then the plaintiff is
fishponds. allowed to present his
evidence ex parte and the
court shall render judgment
on the basis thereof. Thus,
the plaintiff is given the
privilege to present his
evidence without objection
from the defendant, the
likelihood being that the
court will decide in favor of
the plaintiff, the defendant
having forfeited the
opportunity to rebut or
present its own evidence.
Corpuz v. Corpuz Azucena Corpus had both a Whether or not the Yes. The spouses' petition Section 5 of Rule 18
mastercard and VISA of excuse of the for certiorari was thus provides that the dismissal
Citibank, which she paid in full petitioners is a valid properly dismissed by the of an action due to the
on December 7, 1998. While cause under Rule appellate court. plaintiff's failure to appear at
she was in Italy, she dined at 18 of the Rules of Procedural infirmities the pre-trial shall be with
a restaurant. When she tried Court. aside, this Court took a prejudice, unless otherwise
to settle her bill, she considered look at the ordered by the court. In this
presented her VISA which the spouses' excuse to justify case, the trial court deemed
restaurant did not honor, but their non-appearance at the plaintiffs — herein
then she presented her the pre-trial but found spouses as non-suited and
MasterCard which was nothing exceptional to ordered the dismissal of
honored. On the same day warrant a reversal of the their Complaint.
she incurred a bill with a store lower courts' disposition
but both her VISA and thereof.
Mastercard were not honored.
Azucena’s husband inquired
why his wife’s credit cards
were not honored, and
Citibank explained that her
check-payments had not yet
been cleared at the time.
When she returned, she wrote
letters to Citibank.Citibank did
not respond to the letter,
however, drawing Azucena to
write Citibank for the
cancellation of the cards. Only
after Azucena's counsel
informed Citibank of imminent
legal remedies on her part did
Citibank indulge Azucena with
a written explanation why her
credit cards were not honored
in Italy.
Kent v. Micarez Petitioner’s parents were Whether the RTC Yes. The Court found the
Since mediation is part of
looking for a residential lot for erred in dismissing sanction too severe to be
Pre-Trial, the trial court
petitioner and her spouse. As Civil Case No. 13- imposed on the petitioner
shall impose the
she was married to an 2007 due to the where the records of the
appropriate sanction
American citizen, she failure of petitioner's case is devoid of evidence
including but not limited
arranged to pay for the duly authorized of willful or flagrant
to censure, reprimand,
purchase price of the representative, disregard of the rules on
contempt and such other
residential lot and register in Manalang, and her mediation proceedings.
sanctions as are provided
the names of Spouses counsel to attend There is no clear
under the Rules of
Micarez, under an implied the mediation demonstration that the
Court for failure to appear
trust. A deed of absolute sale proceedings under absence of petitioner's for pre-trial, in case any or
was executed between the provisions representative during both of the parties absent
Spouses Micarez and the of A.M. No. 01-10- mediation proceedings on himself/themselves, or for
owner. Petitioner sent the 5-SC-PHILJA and March 1, 2008 was abusive conduct during
money which was used for 1997 Rules on Civil intended to perpetuate mediation proceedings.
the payment of the lot. Later, Procedure. delay in the litigation of
she learned that the Spouses the case. Neither is it
Micarez sold the lot to indicative of lack of
Dionesio. interest on the part of
petitioner to enter into a
possible amicable
settlement of the case.|||
Aguilar v. Three complaints for sum of Whether or not the Yes. In the case at bar, The case of Philippine
LIghtbringers money were separately filed MCTC properly the petitioners failed to American Life & General
Credit by respondent Lightbringers allowed respondent attend the pre-trial Insurance Company v.
Cooperative, Credit Cooperative against to present evidence conference set on August Joseph Enario discussed
petitioners. Petitioners were ex parte? 25, 2009. They did not the difference between non-
members of the cooperative even give any excuse for appearance of a defendant
who borrowed funds. their non-appearance, in a pre-trial conference and
Petitioners say there was a manifestly ignoring the the declaration of a
discrepancy between the importance of the pre-trial defendant in default in the
principal amount of the loans stage. Thus, the MCTC present Rules of Civil
which show that they never properly issued the Order, Procedure. If the absent
borrowed. They also claim allowing respondent to party is the plaintiff, then his
that no interest could be had present evidence ex case shall be dismissed. If it
because there was no written parte. is the defendant who fails to
agreement as to it’s appear, then the plaintiff is
imposition. allowed to present his
evidence ex parte and the
court shall render judgment
on the basis thereof. Thus,
the plaintiff is given the
privilege to present his
evidence without objection
from the defendant, the
likelihood being that the
court will decide in favor of
the plaintiff, the defendant
having forfeited the
opportunity to rebut or
present his own evidence.
Sps. Salvador v. There was a dispute wherein Whether or not the As a result of their Pre-trial is an answer to the
Sps. Rabaja petitioner spouses as the Court of Appeals inattentiveness, Spouses clarion call for the speedy
sellers, the sellers’ agent, and erred in not finding Salvador could no longer disposition of cases.
the buyers, respondents, that the lower court present any evidence in Although it was discretionary
were fighting over a parcel of gravely abused its their favor. Spouses under the 1940 Rules of
land. What happened was discretion in Rabaja, as plaintiffs, were Court, it was made
that respondent heard that declaring petitioners properly allowed by the mandatory under the 1964
petitioners were looking for in default and in RTC to present Rules and the subsequent
buyers, so respondent made depriving them of evidence ex parte against amendments in 1997. "The
the initial payment to the the opportunity to Spouses Salvador as importance of pre-trial in civil
agent in front of the seller. cross-examine defendants. Considering actions cannot be
The seller had a special respondents Sps. that Gonzales as co- overemphasized."
power of attorney. The next Rabaja. defendant was able to
day, there was a contract to attend the pre-trial
sell. Respondents made conference, she was
several payments. However, allowed to present her
petitioners said they did not evidence. The RTC could
receive any payment, and only render judgment
respondents received a notice based on the evidence
to vacate. presented during the trial.

Rule 19 — Intervention

Pagtalunan v. A case was filed with the Whether or not the Yes. In the present case, Intervention is not a matter
Tamayo, Court of the First Instance of petitioners have a the State in the exercise of right but may be permitted
Bulacan by the respondent, right to intervene in of its sovereign power of by the courts when the
who is the Republic of The the expropriation eminent domain has applicant shows facts which
Philippines, for expropriation proceedings decided to expropriate the satisfy the requirements of
of a parcel of land. The CFI instituted by the subject property for public the law authorizing
gave a writ of posession State? use as a permanent site intervention. Under Section
placing the Republic in for the Bulacan Area Shop 2, Rule 12 of the Revised
possession of the land, of the Department of Rules of Court, what
Public Works and qualifies a person to
Highways. On the other intervene is his possession
hand, petitioners have not of a legal interest in the
been issued matter in litigation, or in the
an emancipation success of either of the
patent. |Petitioners cannot parties, or an interest
now successfully argue against both, or when he is
that Celso Pagtalunan is so situated as to be
legally entitled to a portion adversely affected by a
of the proceeds from the distribution or other
expropriation proceedings disposition of property in the
corresponding to the custody of the court or an
value of the landholding. officer thereof.
Pulgar v. RT of 34 tax declarations on the Whether or not the No. The cessation of the Jurisdiction over an
Mauban building and machinery in RTC erred in principal litigation on intervention is governed by
Mauban Plant were issued dismissing Pulgar's jurisdictional grounds at jurisdiction over the main
and assessedfor a certain motion for that means that Pulgar action. Accordingly, an
amount. Later, the QPL tried intervention as a had, as a matter of intervention presupposes
to pay first quarter installment consequence of the course, lost his right to the pendency of a suit in a
but such was rejected. Thus, dismissal of the intervene. court of competent
QPL filed a complaint for main case. jurisdiction.
consignation and damages
before the RTC against the
province of Quezon.

Rule 20 — Calendar of Cases

Ang Kek Chen v. Whether or not the It is not. When the A violation or disregard of
Bello Petitioner Ang was charged issue of alleged respondent judge the Court's circular on how
before the then Manila City, grave abuse of conducted the raffle of the the raffle of cases should be
with the crimes of discretion in three criminal cases in conducted is not to be
"Maltreatment," "Threats," and violation of Circular question, apparently in countenanced. A party has
"Slight Physical Injuries," No. 7 of this Court, violation of the Court's the right to be heard by an
committed on December regarding the Circular No. 7, he did not impartial and unbiased
1977. manner of raffle of only arouse the suspicion tribunal. LThe raffle of cases
cases, not denied or that he had some ulterior is of vital importance to the
explained by public motive for doing so, but administration of justice
respondent, is a he violated the cardinal because it is intended to
trivial one.||. rule that all judicial insure impartial adjudication
processes must be done of cases. By raffling the
above board. cases public suspicion
regarding assignment of
cases to predetermined
judges is obviated.
Rule 21 — Subpoena

Universal Rubber Petitioner was sued by Whether or not the Yes. The argument that Based on jurisprudence, in
Products, Inc. v. two respondent issuance of the the petitioner should first order to entitle a party to the
CA corporations who "subpoena duces be found guilty of unfair issuance of a
claim unfair tecum" is proper in competition before an "subpoena duces tecum", it
competition. Private a suit for unfair accounting for purposes must appear, by clear and
respondents made a competition? of ascertaining the unequivocal proof, that the
request to the amount of damages book or document sought to
respondent Judge to recoverable can proceed, be produced contains
issue a stands without merit. The evidence relevant and
subpoena duces complaint for unfair material to the issue before
tecum against the competition is basically a the court, and that the
treasurer of herein suit for "injunction and precise book, paper or
petitioner. Acting damages. Hence, the document containing such
favorably on that collection of the evidence has been so
request, said complainant for the designated or described that
respondent Judge accounting of petitioner's it may be identified. A
issued a gross sales as damages "subpoena duces tecum"
subpoena duces per R.A. 166, appears once issued by the court
tecum,Petitioner filed most relevant. For Us, to may be quashed upon
a motion in the court determine the amount of motion if the issuance
below praying that the damages allowable after thereof is unreasonable and
subpoena duces the final determination of oppressive, or the relevancy
tecum dated be the unfair labor case of the books, documents or
quashed on the would not only render things does not appear, or if
grounds that: (1) the nugatory the rights of the persons in whose behalf
said subpoena is both complainant under Sec. the subpoena is issued fails
unreasonable and 23 of R.A. 166, but would to advance the reasonable
oppressive as the be a repetitious process cost of production thereof.
books and documents causing only unnecessary
called for are delay.
numerous and
voluminous; (2) there
is no good cause
shown for the
issuance thereof; and
(3) the books and
documents are not
relevant to the case
pending below. Acting
on the said motion and
on the opposition
thereto, respondent
Judge denied the
motion to quash the
subpoena duces
tecum. ||

People v. Montejo A petition was alleged by the Whether or not It was argued that under the
Yes. Upon refusal to
City Fiscal of Zamboanga Respondent Judge Rules of Court, a witness is
give bail, the court must
against Felix Wee Sit for commit any error or not bound to attend a
commit him to prison
double homicide and serious abuse his discretion hearing if held outside the
until he complies or is
physical injuries through in refusing to issue province he resides unless
legally discharged.
reckless imprudence. It was an order of arrest or the distance be less than 50
claimed that Ernesto to cite said witness kilometers from his
Salvador, a resident of Rizal, for contempt? residence to the place of
was a material and important trial. Such contention did not
witness. command the assent of this
Court. Thus: "It is loathe to
clip what undoubtedly is the
inherent power of the Court
to compel the attendance of
persons to testify in a case
pending therein. Section 9 of
Rule 23 is thus interpreted to
apply solely to civil cases.
Genorga v. Dr Gil Genorga, the former Whether or not a Yes. What was done by It was argued that under the
Quintain, municipal health officer of court of first Judge Quitain was, Rules of Court, a witness is
Masbate had a warrant of instance hearing a therefore, within his not bound to attend a
arrest against him as he did criminal case may discretion. hearing if held outside the
not appear as medico-legal compel by province he resides unless
witness in a pending murder subpoena the the distance be less than 50
case. Accused says that he attendance of a kilometers from his
sent a letter to respondent witness in his sala residence to the place of
judge asking if he would be in Zamboanga City, trial. Such contention did not
reimbursed but got nor reply. when the known command the assent of this
Thus, he suffered humiliation address of such Court. It would follow then
from being arrested. witness is at that respondent Judge
Montalban, Rizal? should decide said motion
without taking into
consideration Section 9 of
Rule 23.
Rule 22 — Computation of Time

Luz v. National The petitioner was charged Whether or not the Yes. The Court The extension granted by
Amnesty with violation of illegal petitioner timely Resolves, for the the Court of Appeals should
Commission possession of firearms in the filed his second guidance of the be tacked to the original
Regional Trial Court of Makati motion for extension Bench and the period and commences
City. of time to file his Bar, to declare that immediately after the
petition for review? Section 1, Rule 22 expiration of such period.
speaks only of “the Under the Resolution of this
last day of the Court in A.M. No. 00-2-14-
period” so that SC, the CA has no
when a party discretion to reckon the
seeks an commencement of the
extension and the extension it granted from a
same is granted, date later than the expiration
the due date of such period, regardless of
ceases to be the the fact that said due date is
last day and a Saturday, Sunday, or a
hence, the legal holiday.
provision no longer
applies. Any
extension of time
to file the required
pleading should
therefore be
counted from the
expiration of the
period regardless
of the fact that said
due date is a
Saturday, Sunday
or legal holiday.

Neypes v. CA An action for annulment of Whether or not the Yes,Henceforth, this The new rule aims to
judgement and titles of land petition for appeal "fresh period rule" shall regiment or make the appeal
were filed before the RTC of was timely filed? also apply to Rule 40 period uniform, to be
Oriental Mindoro against the governing appeals from counted from receipt of the
Bureau of Forest the Municipal Trial Courts order denying the motion for
Development. In the course of to the Regional Trial new trial, motion for
the proceedings, the parties Courts; Rule 42 on reconsideration (whether full
filed various motions with the petitions for review from or partial) or any final order
trial court. Among these were: the Regional Trial Courts or resolution. The Court thus
(1) the motion filed by to the Court of Appeals; hold that petitioners
petitioners to declare the Rule 43 on appeals from seasonably filed their notice
respondent heirs, the Bureau quasi-judicial agencies to of appeal within the fresh
of Lands and the Bureau of the Court of Appeals and period of 15 days, counted
Forest Development in default Rule 45 governing from July 22, 1998, the date
and (2) the motions to dismiss appeals by certiorari to of the receipt of Motion for
filed by the respondent heirs the Supreme Court. Reconsideration.
and the Land Bank of the
Philippines, respectively.
Rule 23 — Deposition pending action
Vda. de Respondents were charged Whether or not Rule No. In the case at bench, In criminal cases, the
Manguerra v. with Estafa through 23 of the Rules of in issue is the examination accused may be afforded
Risos Falsification of public Court applies to the of a prosecution witness, the opportunity to cross-
document. The Criminal case instant case? who, according to the examine the witnesses
arose from the falsification of petitioners, was too sick to pursuant to his constitutional
a deed of real estate travel and appear before right to confront the
mortgage allegedly committed the trial court. witnesses face to face. It
by respondents where they also gives the parties and
made it appear that their counsel the chance to
Concepcion, the owner of the propound such questions as
mortgaged property known as they deem material and
the Gorordo property, affixed necessary to support their
her signature to the position or to test the
document. Hence, the credibility of said
criminal case. Respondents witnesses. Lastly, this rule
filed a Motion for Suspension enables the judge to
of the Proceedings in Criminal observe the witnesses'
Case on the ground of demeanor. As exceptions,
prejudicial question. The Rules 23 to 28 of the Rules
RTC granted the aforesaid of Court provide for the
motion. Concepcion's motion different modes of discovery
for reconsideration was that may be resorted to by a
denied on June 5, 2000 party to an action. These
rules are adopted either to
perpetuate the testimonies
of witnesses or as modes of
discovery.
Rule 30 — Trial

Lee v. Romillo, Private respondent filed a In the case of The National


Whether or not No default is involved
Jr., complaint for accounting with Housing Authority v.Court of
the delegation of herein. As held in the
damages. There was a Appeals, the Court
the reception of case of Laluan v.
contract for arrastre services categorically stated that:
the evidence ex- Manalo, no provision of
in Manila between respondent "The contention that the
parte to the lower law or principle of public
and William Lines. In the year Trial Court cannot delegate
court's legal policy prohibits a Court
1973, plaintiff Mendioro the reception of evidence to
researcher is from authorizing its
conducted arrastre operations illegal.||| Clerk of Court to receive its Clerk of Court, citing the
as per arrastre contract with the evidence of a party case of Lim Tanhu v.
William Lines, Inc, that all the litigant.|| Ramolete is not well taken.
income from the arrastre Suffice it to say, for
operations during the said purposes of this suit, that the
period were received and held said case referred to
in trust for Mendioro by reception of evidence by a
defendants Lee, Tan and Clerk of Court after
Chiongbian who were also declaration of defendant's
tasked with the disbursement default
of the said income for
operational expenses.
Defendants Lee, Tan and
Chiongbian were duty-bound
to turn over to plaintiff
Mendioro the income they
held in trust for him; and they
failed to do so despite
Mendioro's repeated
demands.|||

Rule 31 — Consolidation or Severance

Metropolitan A complaint was filed against Whether or not The Trial Court held the Exceptions to the general
Bank and Trust the Marcoses for ill-gotten holding of a Separate Trials to be rule are permitted only when
Company v. wealth. Later, the Republic separate trial would improper. The Court held there are extraordinary
Sandoval moved for the amendment of deny Metro bank that Sandiganbayan grounds for conducting
the complaint to implead due process, committed grave abuse of separate trials on different
Asian Bank as an additional because Asian its discretion in ordering a issues raised in the same
defendant. This was granted. Bank was entitled to separate trial as to Asian case, or when separate trials
contest the Bank (Metrobank) on the of the issues will avoid
evidence of the ground that the issue prejudice, or when separate
Republic against against Asian Bank was trials of the issues will
the original distinct and separate from further convenience, or
defendants prior to that against the original when separate trials of the
Asian Bank's defendants. Thereby, the issues will promote justice,
inclusion as an Sandiganbayan veered or when separate trials of
additional away from the general the issues will give a fair trial
defendant.||| rule of having all the to all parties. Otherwise, the
issues in every case tried general rule must apply.
at one time, unreasonably
shunting aside the dictum
in Corrigan, supra, that a
"single trial will generally
lessen the delay,
expense, and
inconvenience to the
parties and the courts."
Rule 33 — Demurrer to Evidence

Casent Realty Two promissory notes were Did CA err in No. The evidence Rule 33, Section 1 of the
Development issued by petitioner to Rare excluding the contemplated by the 1997 Rules of Civil
Corp. v. reality, which were assigned petitioner’s rule on demurrer is that Procedure: “the defendant
Philbanking Corp to respondent by a Deed of affirmative which pertains to the may move for dismissal on
Assignment. Petitioner did not defenses in its merits of the case, the ground that upon the
pay, despite demand. facts and the law the plaintiff
answer in excluding technical has shown no right to relief.
resolving a aspects such as The evidence contemplated
demurrer to capacity to sue. by the rule on demurrer is
evidence. that which pertains to the
merits of the case, excluding
technical aspects such as
capacity to sue. The “facts”
referred to in Section 8
should include all the
means sanctioned by the
Rules of Court in
ascertaining matters in
judicial proceedings.
Petitioner, by way of
defense, should have
presented evidence to
show that the Dacion
includes the promissory
notes.
Uy v. Chua Respondent had an illegal Was the trial court it was incumbent upon Demurrer to Evidence is
relationship with Irene and correct in granting RTC to make thegoverned by Rule 33 of the
sired two children. The birth the demurrer to determination. It did notRules of Court, Section 1
certificates of the children evidence? take into consideration provides that: “After the
were filed out with the any of the evidence plaintiff has completed the
presented by petitioner. presentation of his evidence,
names or Irene’s father and the defendant may move for
her mother’s maiden name. dismissal on the ground that
Respondent financially upon the facts and the law
supported petitioner the plaintiff has shown no
right to relief. If his motion is
denied, he shall have the
right to present evidence. If
the motion is granted but on
appeal the order of dismissal
is reversed he shall be
deemed to have waived the
right to present evidence.”
Demurrer is an aid or
instrument for the
expeditious termination of an
action, similar to a motion to
dismiss, which the court or
tribunal may either grant or
deny. Necessarily, the last
line of Section 1, Rule 33
should not apply herein and
respondent should still be
allowed to present evidence
before the RTC for Special
Proceedings No. 12562-
CEB.
Rule 34 — Judgment on the Pleadings

Wood Technology WTC got a loan from Whether or not the We agree with both the “In a proper case for
Corp. v. Equitable respondent, signed by appellate court trial and the appellate judgment on the
Bank Cordova and Young, which made a mistake courts that this matter pleadings, there is no
bound themselves as when it allowed proferred as a defense ostensible issue at all
surities. When demand was RTCs judgement could be resolved because of the failure of
made by respondent, on the pleadings judiciously by plain the defending party’s
petitioners did not pay. resort to the stipulations answer to raise an issue.
They claim that only one in the promissory note On the other hand, in the
demand letter was made. which was already case of a summary
Plus, the loan had not before the trial court. judgment, issues
matured yet, as the apparently exist.”
maturity date was blank. The judgment rendered
by the trial court is valid
as a summary judgment.
A judgment on the
pleadings is proper when
an answer fails to tender
an issue, or otherwise
admits the material
allegation of the adverse
party’s pleadings.
Municipality of The NPC was found liable If the application a review of the records A motion for judgment on
Tiwi v. Betito for unpaid real estate of the rule of reveal that respondent the pleadings admits the
taxes. This included judgment on the and petitioners set-up truth of all material and
geothermal plants in the pleadings and/or multiple levels of claims relevant allegations of the
Municipality of Tiwi with summary and defenses, opposing party. The
respectively with some judgment must rest on those
substations in Albay (which judgment is failing to tender an issue allegations taken together
were sold in an auction baseless, while others requiring the with such other allegations
sale). In the sale, the improper and presentation of evidence as are admitted in the
Municipality obtained unwarranted for resolution. pleadings. It is proper when
ownership over the an answer fails to tender an
properties. An agreement issue, or otherwise admits
was entered into between the material allegation of the
BPC and Albay where NPC adverse party’s pleading. If it
agreed to settle it’s tax appears that not all the
liabilities.The mayor material allegations of the
requested to remit the complaint were admitted in
the answer for some of them
rightful tax share of Tiwi were either denied or
and its barangays. The disputed, then the defendant
governor stated that the has set up certain special
request cannot be granted. defenses. Those, if proven,
So NPC requested would have the effect of
clarification from the OP. nullifying plaintiff's main
cause of action, judgment on
the pleadings cannot be
rendered.
Tan v. Dela Vega A complaint for quieting of Whether or not a In the case at bar, we In a proper case for
title and the declaration of judgment on the find that the trial court judgment on the
nullity of free patent was pleadings is erred in rendering pleadings, there is no
filed by respondents proper in the judgment on the ostensible issue at all
against the heirs of instant case. pleadings because the because of the failure of
Mencias and the Secretary pleadings filed by the the defending party’s
of DENR. Petitioner was parties generated answer to raise an issue.
later impleaded as ostensible issues that
purchasers of the disputed necessitate the
lot in Rizal. presentation of
evidence in order to
determine the validity of
the titles from which the
respondent’s title was
derived
Adolfo v. Adolfo A petitioner for judicial Whether or not the Instead of resolving Judgment on the pleadings
separation of property was CA erred in petitioner's motion for is proper "where an answer
filed in the RTC by Adolfo deciding the case judgment on the fails to tender an issue, or
against his wife. Allegedly, on a question of pleadings, the trial court otherwise admits the
the parties were married substance not in should have denied it or material allegations of the
and they had one child. accord with law, held it in abeyance. adverse party's pleading."
The acquired a lot in Rule 26 of the Summary judgment, on the
Mandaue during marriage. 1997 Rules, and other hand, will be granted
The parties then separated applicable "if the pleadings, supporting
because of irreconcilable jurisprudence. affidavits, depositions, and
admissions on file, show
differences. Petitioner then that, except as to the
suggested separation of amount of damages, there is
conjugal property, but no genuine issue as to any
respondent refused. material fact and that the
Respondent contended that moving party is entitled to a
she is the sole owner of the judgment as a matter of
subject property which she law." If the answer does in
inherited from her mother; fact specifically deny the
that petitioner is a lazy bum, material averments of the
gambler, drunkard, wife complaint and/or asserts
abuser, and neglectful father; affirmative defenses, a
that respondent found all judgment on the pleadings
means to support the family. would naturally be improper.
She caused to be built on a In a case for judgment on
portion of her mother's land a the pleadings, the Answer is
house even while petitioner such that no issue is raised
was bumming around; that at all. The essential question
one day, petitioner destroyed in such a case is whether
the roof of the house that was there are issues generated
then being built; that petitioner by the pleadings. The trial
subsequently abandoned her court however disregarded
and their child in 1968, and the fact that its decision was
transferred to Davao City then the subject of a
where he took a mistress and pending appeal. One of the
begot four children by her. issues raised in the appeal
is precisely whether the
subject property is conjugal,
or a paraphernal asset of the
respondent.
Rule 35 — Summary Judgment
Calubaquib v. A landholding in Cagayan Whether or not a Yes. For this case, the The guidelines for summary
Republic was declared by President propriety of trial court proceeded to judgement was ignored by
Manuel L Quezon as a rendering a render summary judgment the trial court. Summary
military reservation site, under summary with neither of the parties judgment is proper when,
Proclamation No. 80. It is judgment is proper filing a motion therefor it upon motion of the plaintiff
subject to private rights, if any was improper for the trial or the defendant, the court
there be. Supposedly,
in this case. court to have persisted in finds that the answer filed by
petitioners unlawfully entered rendering summary the defendant does not
the military reservation and judgment. tender a genuine issue as to
demands were made for them any material fact and that
to leave. Petitioners claim that one party is entitled to a
the land is theirs. judgment as a matter of law.
YKR Corp. v. Yulo died. After his death, Whether or not the Yes. We cannot thus A prudent examination of
Philippine Agri- his late wife Teresa J Yulo Sandiganbayan properly ascertain the evidence on record
Business Center and their six children erred in law when whether the facts which yields to no other conclusion
Corp., became substitutes. His it granted the the latter could not make that there exists a genuine
wife then died. The respondent’s any truthful admission or issue of fact as raised in
denial are so plainly and both petitions. In ruling on
petitioners have three other motion for necessarily within their the issue of whether a
siblings who claim that they summary knowledge. genuine issue of fact exists,
"have gone their own judgment even if there was no mention of any
separate way[s] when Luis A. the same is not circumstance or situation
Yulo died. [p]etitioners have supported by upon which the court a quo
no knowledge or information deposition, derived its conclusion that
sufficient to form a belief if affidavit or the matters requested for
their siblings are aware of the admission "ought to be
proceedings in
admission on the
records. within the personal
(Sandiganbayan), including knowledge" of YKR
respondent's move for a Corporation and seven out
summary judgment and the of the ten Yulo Heirs.
assailed resolutions of the
Sandiganbayan."

Rule 36 — Judgments Final Orders and Entry


Sumbilla v. Matrix Petitioner issued Philippine Whether or not the Yes.. Unfortunately, in the Under the doctrine of finality
Finance Corp., Business Bank Checks as penalty imposed present case, the MeTC and immutability of
payment for her loan. in the MeTC Decision is already final judgments, a decision that
However, the checks were Decision, which is and executory after has acquired finality
dishonored having been already final and petitioner failed to timely becomes immutable and
file a Notice of Appeal. unalterable and may no
drawn against a closed executory, may longer be modified in any
account. still be modified. respect, even if the
modification is meant to
correct erroneous
conclusions of fact or law,
and whether it will be made
by the court that rendered it
or by the highest court of the
land. Upon finality of the
judgment, the Court loses its
jurisdiction to amend, modify
or alter the same.
Nonetheless, the Court has
the power and prerogative to
suspend its own rules and to
exempt a case from their
operation if and when justice
requires it. Substantial
justice dictates that the
penalty meted on the
petitioner be accordingly
corrected within the
maximum limits prescribed
under Section 1 of BP 22.
Delfino v. Delfino owned several Whether or not the Yes. The clarification The doctrine of finality and
Anasao, parcels of agricultural land Order of Secretary made by Secretary immutability of judgments is
in Laguna. He sold some Pangandaman Pangandaman falls under grounded on fundamental
coconut land, leaving him violated the rule on the fourth exception. considerations of public
with rice land. The tenanted immutability of final policy and sound practice to
judgments? the effect that, at the risk of
portion was tilled by
respondents, and placed occasional error, the
under Operation land judgments of the courts
transfer. must become final at some
definite date set by law.
There are exceptions to the
general rule, namely: (1) the
correction of clerical errors;
(2) the so-called nunc pro
tunc entries which cause no
prejudice to any party; (3)
void judgments; and (4)
whenever circumstances
transpire after the finality of
the decision rendering its
execution unjust and
inequitable. The clarification
made by Secretary
Pangandaman falls under
the fourth exception.
Club Filipino, Inc. The employees of Club Whether or not No. In the present case, As a general rule, the filing
v. Bautista, Filipino Inc were Club Filipino, this court granted leave to of second Motions for
represented by a union, Inc.’s filing of the petitioner Club Filipino, Reconsideration of a
CLUFEA. There were Supplemental Inc. to file the judgment or final resolution
Supplemental Motion for is prohibited as provided by
bargaining agreements Motion for
Reconsideration in the Rule 52, Section 2 of the
between the two, and Reconsideration Resolution. The Rules of Court. The
CLUFEA made several prevented the Supplemental Motion for petitioner’s Supplemental
demands for a new Resolution from Reconsideration, Motion for Reconsideration
agreement, which was becoming final therefore, is no longer is in the nature of a second
denied. CLUFEA filed before and executory. prohibited. Motion for Reconsideration.
the NCMB a request for For this court to entertain
preventive mediation. The second Motions for
meeting ended with the Reconsideration, the second
parties' respective panels Motions must present
declaring a deadlock in "extraordinarily persuasive
negotiation. CLUFEA filed reasons and only upon
with the NCMB a Notice of express leave first
Strike on the ground of obtained." Once leave to file
bargaining deadlock. is granted, the second
CLUFEA conducted a strike Motion for Reconsideration
vote under the Department of is no longer prohibited This
Labor and Employment's case falls under the
supervision with the majority exception
of CLUFEA's total union
membership voting to strike.
CLUFEA staged a strike on
the ground of bargaining
deadlock.
NTC v. Public biddings were Whether or not the Yes. The Court finds no Section 1, Rule 45 of the
Aplhaomega conducted by TRANSCO. CA erred in reason to disturb the Rules of Court provides that
Integrated Corp., AIC was licensed affirming the CIAC factual findings of the a petition for review on
transmission line Arbitral Tribunal's CIAC Arbitral Tribunal on certiorari under the said rule,
contractor, and was part of findings that AIC the matter of AIC's as in this case, "shall raise
was entitled to its entitlement to damages only questions of law which
the biddings. They were claims for damages which the CA affirmed as must be distinctly set forth.”
awarded 6 government as a result of project being well supported by It is well-settled that findings
construction projects. delays. evidence and properly of fact of quasi-judicial
However, they encountered referred to in the record. . bodies, which have acquired
difficulties and incurred expertise because their
losses because TRANSCO jurisdiction is confined to
breached contract, so they specific matters, are
gave up the projects to generally accorded not only
TRANSCO. AIC, a duly respect, but also finality,
licensed transmission line especially when affirmed by
contractor, participated in the the CA. The CIAC
public biddings conducted by possesses that required
TRANSCO and was awarded expertise in the field of
six (6) government construction arbitration and
construction projects and the factual findings of its
encountered difficulties and construction arbitrators are
incurred losses allegedly due final and conclusive, not
to TRANSCO's breach of their reviewable by this Court on
contracts, prompting it to appeal. Indeed, the rule is
surrender the projects to well entrenched: Specialis
TRANSCO under protest. derogat generali. When two
Charged against TRANSCO: rules apply to a particular
breached the contracts by its case, that which was
failure to: (a) furnish the specially designed for the
required Detailed said case must prevail over
Engineering; (b) arrange a the other.
well-established right-of-way
to the project areas; (c)
secure the necessary permits
and clearances from the
concerned local government
units (LGUs); (d) ensure a
continuous supply of
construction materials; and
(e) carry out AIC's requests
for power shut down.
DBP v. Guarina DBP gave a loan to GC to Whether or not the Under the circumstances, DBP's actuations were
Agricultural and finance the development of Decision of the CA DBP's foreclosure of the legally unfounded. It is true
Realty its resort complex in Iloilo, and its resolution mortgage and the sale of that loans are often secured
Development so GC executed a denying the mortgaged properties by a mortgage constituted
Corp promissory note and real petitioner’s Motion at its instance were on real or personal property
premature, and, therefore, to protect the creditor's
estate mortgage over for void and ineffectual. interest in case of the default
several real properties as Reconsideration of the debtor. In other words,
security, plus a chattel were issued in GC would not incur in delay
mortgage. The loan was accordance with before DBP fully performed
released in several law. its reciprocal obligation.
installments. GC used to Considering that it had yet to
proceeds to defray costs of release the entire proceeds
improvements, and of the loan, DBP could not
demanded balance on the yet make an effective
loan but DBP refused. DBP demand for payment upon
paid some supplies of GC GC to perform its obligation
under the loan. Hence, GC
without it’s approval. It was
found by DBP that GC had would not be in default
not completed the without the demand.
construction works, so DBP
demanded that GC
expedite the completion of
the project. Not happy with
the non-action, DBP
initiated extrajudicial
foreclosure proceedings. A
notice of foreclosure sale
was sent to GC, which was
published leading the
clients and patrons of GC
to think that the resort had
closed.
Rule 37 — New Trial or Reconsideration

Gonzales v. Solid Gonzales employment was Whether or not the Yes.The matter before As a rule, a second
Cement terminated by his employer, second motion for us — in the Rule 45 motion for reconsideration
Corporation, respondent. However, the reconsideration is petition questioning the is a prohibited pleading
LA held that it was an allowed? CA's Rule 65 under the Rules of Court,
illegal dismissal. So determination — is the and this reason alone is
Gonzales filed a motion for scope of the benefits sufficient basis for us to
execution of reinstatement. awarded by the LA, as dismiss the present
Solid reinstated Gonzales modified on appeal and second motion for
in the payroll. But the ultimately affirmed by reconsideration. The
NLRC modified the this Court, which ruling ruling in the original case,
decision be reduing the has become final and as affirmed by the Court,
amount of damages. which now must be has been expressly
implemented as a declared final. The
matter of law. Given immutability principle,
these considerations, to rather than being
reopen this case on absolute, is subject to
second motion for well-settled exceptions,
reconsideration would among which is its
not actually embroil the inapplicability when a
Court with changes in decision claimed to be
the decision on the final is not only erroneous,
merits of the case, but but null and void
would confine itself
solely to the issue of the
CA's actions in the
course of determining
lack or excess of
jurisdiction or the
presence of grave
abuse of discretion in
reviewing the NLRC's
ruling on the execution
aspect of the case.
Tadeja v. People Ruben Bernardo had died, The confession of Newly discovered
and the witness. Elena and Whether or not to Plaridel does not meet evidence refers to that
Jacinta, were watching a grant the this requisite. He which (a) is discovered
public dance during a petitioner’s motion participated in the trial after trial; (b) could not
fiesta. There they saw to reopen the case before the RTC and have been discovered
Ruben being hacked to for reception of even gave testimony as and produced at the trial
death by petitioners. further evidence in to his defense. It was even with the exercise of
Petitioners claim that the trial court. only after he and reasonable diligence; (c)
Ruben and his son went to petitioners had been is material, not merely
the barangay in a drunken convicted by the trial cumulative, corroborative
state and was waving a court that he or impeaching; and (d) is
knife and cursing at the absconded. of such weight that it
crowd. would probably change
the judgment if admitted.
The most important
requisite is that the
evidence could not have
been discovered and
produced at the trial even
with reasonable diligence;
hence, the term "newly
discovered."
SJS Officers v. A comprehensive plan and Whether or not The allegation of The denial of a motion for
Lim relocation plan was there is encroachment on the reconsideration signifies that
submitted to the RTC of interference or policy making power of the grounds relied upon
Manila by Chevron, encroachment the political departments have been found, upon due
Pilipinas Shell Petroleum, with the policy of the government is deliberation, to be without
bereft of merit. The merit, as not being of
and Petron.The presiding making powers of prayer that the submission sufficient weight to warrant a
judge would monitor the the political of an updated modification of the judgment
strict enforcement of the departments of comprehensive plan and or final order. It means not
decision. the government. relocation schedule, only that the grounds relied
including the period for upon are lacking in merit but
relocation, be deferred also that any other, not so
until after the Motion is raised, is deemed waived
resolved with finality is and may no longer be set up
denied. The compliance in a subsequent motion or
period prescribed in the application to overturn the
Decision shall remain. judgment; and this is true,
whatever may be the title
given to such motion or
application, whether it be
"second motion for
reconsideration" or "motion
for clarification" or "plea for
due process" or "prayer for a
second look," or "motion to
defer, or set aside, entry of
judgment," or . . ., etc . This
Resolution is final. Under
pain of contempt, no further
pleadings, motions or
papers in the guise of the
above-enumerated
submissions shall, thus, be
entertained in these cases.
Rule 38 — Relief from Judgments, Orders, or Other Proceedings

Afdal v. Carlos, A complaint for unlawful Whether or not the Yes. A petition for relief from
detainer against petitioners RTC erred in judgment in forcible entry
Gulbajar and Joh Does was dismissing their and unlawful detainer cases,
filed by Carlos. It was alleged petition for relief as in the present case, is a
that they were occupying a from judgment? prohibited pleading. The
parcel of land in respondents legal justification is that
name, and that respondents Section 13(4) of Rule 70 of
had allowed them to stay the Rules of Court provides
there by mere tolerance. It petitions for relief from
was claimed that petitioner judgment are prohibited. The
sold the property to same is also provided in the
respondents but allowed Rules of Summary
petitioners to stay in the Procedure. The reason for
property. So respondent this is to achieve an
demanded that petitioners expeditious and inexpensive
turn over the property to him, determination of the cases
as he needed such for his subject of summary
personal use. There were procedure. Moreover,
several attempts to serve Section 1, Rule 38 of the
summons and complaint on Rules of Court provides that
petitioners, but they still failed when a judgment or final
to answer. order is entered, or any
other proceeding is
thereafter taken against a
party in any court through
fraud, accident, mistake or
excusable negligence, he
may file a petition in such
court and in the same
case praying that the
judgment, order or
proceeding be set aside. A
petition for relief from
judgment, if allowed by the
Rules and not a prohibited
pleading, should be filed
with and resolved by the
court in the same case from
which the petition arose.
City of Dagupan The City of Dagpuan caused Whether or not the Yes. Considering the As a rule, a motion without
v. Maramba the demolition of a lack of notice of circumstances of the a notice of hearing is
commercial fish center, hearing in a motion present case, we believe considered pro forma and
without giveing notice to for reconsideration that the requirements of does not affect the
Maramba. There was also is excusable procedural due process reglementary period for the
included a threat of taking negligence that were substantially appeal or the filing of the
over the property. This made allows the filing of a complied with, and that requisite pleading. As an
Maramba file a complaint for petition for relief of the compliance justified a integral component of
injunction with damages and judgment? departure from a literal procedural due process, the
a restraining order. According application of the rule on three-day notice required by
to Maramba, the demolition notice of hearing. This the Rules is not intended for
was unlawful and that the court held that "when the the benefit of the movant.
"complete demolition and adverse party has actually Rather, the requirement is
destruction of the previously had the opportunity to be for the purpose of avoiding
existing commercial fish heard, and has indeed surprises that may be
center of plaintiff is valued at been heard through sprung upon the adverse
Five Million (₱10,000,000.00) pleadings filed in party, who must be given
pesos." The word, "ten," was opposition to the motion, time to study and meet the
handwritten on top of the the purpose behind the arguments in the motion
word, "five." Maramba asked rule is deemed duly before a resolution by the
for a judgment "ordering served.” court. The test is the
defendant corporation to pay presence of the opportunity
plaintiff the amount of Ten to be heard, as well as to
Thousand (₱10,000.00) have time to study the
pesos for the actual and motion and meaningfully
present value of the oppose or controvert the
commercial fish center grounds upon which it is
completely demolished by based.
public defendant." The word,
"million," was handwritten on
top of the word, "thousand,"
and an additional zero was
handwritten at the end of the
numerical figure. The
handwritten intercalation was
notexplained in any part of the
records and in the
proceedings.
Rule 39, Sections 1 to 14 — Execution, Satisfaction and Effect of Judgments

Quicho v. Reyes, Petitioner alleged that the Whether or not Yes. it is clear that the Section 9, Rule 39 of the
Jr procedure observed by Reyes Sheriff Reyes sheriff shall demand from Rules of Court provides for
in implementing the alias writ gravely abused his the judgment obligor the the procedure as to how
violated the 2002 Revised authority relative to immediate payment in execution of judgments for
Manual for Clerks of Court. the implementation cash, certified bank check money is enforced. The
He cited the manual: "if the of the writ of or any other mode of duties of a sheriff are: (1) to
judgment obligor cannot pay execution? payment acceptable to the first make a demand from
all or part of the obligation in judgment obligee. If the the obligor for the immediate
cash, certified bank check or judgment obligor cannot payment of the full amount
other mode of payment pay by these methods stated in the writ of
acceptable to the judgment immediately or at once, he execution and of all lawful
obligee, the officer shall levy can exercise his option to fees; (2) to receive payment
upon the properties of the choose which of his in the form of cash, certified
judgment obligor of every kind property can be levied bank check payable to the
and nature whatsoever which upon. obligee, or any other form of
may be disposed of for value payment acceptable to the
and not otherwise exempt latter; (3) to levy upon the
from execution giving the properties of the obligor, not
latter the option to exempt from execution, if
immediately choose which the latter cannot pay all or
property or part thereof may part of the obligation; (4)
be levied upon, sufficient to give the obligor the
satisfy the judgment." He opportunity to exercise the
states that as the holder of option to choose which
the assets and properties of property may be levied
Traders Royal Bank (TRB), upon; (5) in case the option
BOC was given the option to is not exercised, to first levy
choose which property to be on the personal properties of
surrendered to satisfy the the obligor, including the
judgment. It was only when garnishment of debts due
BOC was unable to exercise the obligor and other credits,
the option that Reyes could i.e., bank deposits, financial
levy on other properties. BOC interests, royalties,
was forced to surrender under commissions and other
protest a real estate property personal properties not
to satisfy the judgment and capable of manual delivery
preserve its other properties or in the possession or
from being wrongfully levied control of third parties; and
by Reyes. It was argued that (6) to levy on real properties
Reyes did not give BOC a if the personal properties are
chance to exercise that insufficient to answer for the
option. What happened was judgment.
that Reyes blow-torched the
locked grill door of BOC’s
cash vault and forcibly took
the money deposits of its
clients as well as its
computers. Reyes sowed
terror by bringing with him
agents of the NBI who were in
full-battle gear and carrying
high-powered firearms. Atty.
Quicho concluded that these
illegal acts of Reyes
warranted his relief as sheriff
of the RTC. Reyes denied the
charges against him.
According to him, he did not
violate any law when he
refused to accept BOC’s offer
of a property located in
Paranaque City to satisfy the
judgment debt. Atty. Quicho
refuted Reyes’ argument that
BOC was not entitled to
exercise the option to choose
the properties to be levied;
he explained that under
Section 9, Rule 39 of the
Rules of Court, it was clear
that if the judgment obligor
could not pay the judgment
debt in cash, certified bank
check or other mode of
payment acceptable to the
judgment oblige, he still had
the option to choose which of
his properties he could offer
to satisfy the obligation.
Rule 40 — Appeal from the Municipal Trial Courts to the Regional Trial Courts

Heirs of dela There was a previous case, Whether or not the No. The alleged failure of Section 9, Rule 41 of the
Rosa v. Calderon- wherein petitioners were said Court of Appeals the clerks of court to Revised Rules of Court,
Bargas to illegally enter a parcel of erred in denying the furnish petitioners copies states, in part, that a party's
land in Rizal, and they were petitioners' Petition of the letter of transmittal appeal by notice of appeal is
held to vacate the premises and and Notice of Appealed deemed perfected as to him
and pay damages. Case that resulted in their upon the filing of the notice
inability to file their of appeal in due time. The
memorandum on time can RTC acting as an appellate
no longer be reviewed by court acquired jurisdiction
this Court. over the case of the
petitioners upon their filing of
the notice of appeal on
October 5, 1998. The filing
of the notice of appeal in
due time and the payment of
the appropriate fees by the
petitioners perfected their
appeal in the RTC. As a
necessary consequence
thereof, the MTC was
divested of jurisdiction over
their case. From the filing of
the written notice of appeal,
petitioners' appeal was
perfected without need of
any further act, and,
consequently, the trial court
lost jurisdiction over the
case, both over the record
and over the subject of the
case.
Encarnacion v. Two lots in Isabela covered Was accion No. After a careful In this jurisdiction, the three
Amigo by TCTs were registered publiciana and not evaluation of the evidence kinds of actions for the
under the name of unlawful detainer on record of this case, we recovery of possession of
Encarnacion. The lots formed proper find that the Court of real property are a.) Accion
part of a single track of land Appeals committed no interdictal, or an ejectment
owned by Valiente. But reversible error in holding proceeding which may be
Valiente sold the same to that the proper action in either that for forcible entry
Mallapitan, who sold it to this case is accion or unlawful detainer, which
Magpantay. Magpantay died, publiciana; and in ordering is a summary action for
and his widow executed an the remand of the case to recovery of physical
affidavit of waiver. In this the RTC for further possession where the
waiver, she waived her right proceedings. The length dispossession has not lasted
over the property in favor of of time that the petitioner for more than one year, and
her son-in-law, Encarnacion. was dispossessed of his should be brought in the
Respondent allegedly entered property made his cause proper inferior court b.)
the premises and took of action beyond the ambit Accion publiciana or the
possession of a portion of an accion plenary action for the
without the permission of interdictal and effectively recovery of the real right of
Magpantay. Despite made it one for accion possession, which should be
demands, respondent did not publiciana. After the lapse brought in the proper
vacate. of the one-year period, the Regional Trial Court, and
suit must be commenced c.) Accion reinvindicatoria,
in the Regional Trial Court which is an action for the
via an accion recovery of ownership which
publiciana which is a suit must be brought in the
for recovery of the right to proper Regional Trial Court.
possess.

Canlas v. Tubil Tubil together with her Which court (MTC MTC The allegations in the
husband, claim that they are or RTC) has complaint are what
the owners of a lot. As proof, jurisdiction over the determines the nature of the
she says that before the lot matter? action as well as the court
was titled, it was declared for which has jurisdiction over
tax purposes in her name. the case. In the instant case,
Respondents erected a respondent’s allegations in
residential house on the land the complaint clearly make a
on which they are presently case for an unlawful
occupying. detainer, essential to confer
jurisdiction on the MTC over
the subject matter. She sent
on January 12, 2004 a letter
demanding that petitioners
vacate the property but they
failed and refused to do so.
The complaint for unlawful
detainer was filed on June 9,
2004, or within one year
from the time the last
demand to vacate was
made.

Rule 41 — Appeal from the Regional Trial Courts

Silverio, Jr. v. CA An intestate proceeding was Whether or not the The denial of the motion Sec. 1(a), Rule 41 of the
filed by Ricardo Silberio for Omnibus orders are for reconsideration of an Rules of Court provides that
the settlement of his interlocutory orders order of dismissal of a no appeal may be taken
deceased wife’s estate. While which are not complaint is not an from an order denying a
the case was pending, subject to appeal interlocutory order, motion for reconsideration.
petitioner filed a petition to under Sec. 1 of however, but a final order In all the above instances
remove the adminstrator, Rule 41 as it puts an end to the where the judgment or final
Ricardo C Silvero sr. Ricardo particular matter resolved, order is not appealable, the
Silberio Sr. filed an intestate or settles definitely the aggrieved party may file an
proceeding for the settlement matter therein disposed appropriate special civil
of his deceased wife’s estate. of, and nothing is left for action under Rule 65. The
During the pendancy of the the trial court to do other rationale behind the rule
case, petitioner Ricardo than to execute the order. proscribing the remedy of
Silberio, Jr. filed a petition to appeal from an interlocutory
remove Ricardo C. Silverio, order is to prevent undue
Sr. as the administrator of the delay, useless appeals and
subject estate. undue inconvenience to the
appealing party by having to
assail orders as they are
promulgated by the court,
when they can be contested
in a single appeal. The
appropriate remedy is thus
for the party to wait for the
final judgment or order and
assign such interlocutory
order as an error of the court
on appeal.
Lebin v. Mirasol The administrator of the Whether or not the No. The elimination of the record
Estate of L.J. Hodges was RTC erred in on appeal in most cases,
relayed an offer by petitioners dismissing the retaining the record on
to purchase an asset of the petitioners’ appeal appeal only for appeals in
estate. A deposit of 20% of for their failure to special proceedings and in
the offer was made. The timely file a record other cases in which the
administrator stated that on appeal Rules of Court allows
Erlinda Lebin was the actual multiple appeals was among
occupant of Lot. 18. Atty. the innovations introduced
Tabares was commissioned by Batas Pambansa Blg. 12.
by the RTC to do an ocular No record on appeal shall be
inspection of Lot 18 to see if it required to take an appeal.
was true. In the report, it was In lieu thereof, the entire
confirmed, and the RTC record shall be transmitted
granted the administrator’s with all the pages
motion for the approval of the prominently numbered
offer. Mirasol offered to consecutively, together with
purchase the lot, but then an index of the contents
found out that her house was thereof. The petition for
actually standing on Lot 18. review must be filed with the
Mirasol then filed a petition for Court of Appeals within 15
relief from the order. While days from notice of the
the resolution was pending, judgment, and as already
the petitioners made the last stated, shall point out the
payment for installment, but error of fact or law that will
the motion for execution of warrant a reversal or
the deed of sale was not modification of the decision
acted upon by the RTC. or judgment sought to be
reviewed. An ordinary
appeal is taken by merely
filing a notice of appeal
within 15 days from notice of
the judgment, except in
special proceedings or
cases where multiple
appeals are allowed in which
event the period of appeal is
30 days and a record on
appeal is necessary. There
is therefore no longer any
common method of appeal
in civil cases to the Supreme
Court and the Court of
Appeals.
Briones v. Ruby J Henson filed a Whether or not the No. From an estate proceeding
Henson-Cruz petitioner to allow the will of CA erred in not perspective, the Special
her late mother with the RTC dismissing the Administrator's commission
of Manila. One of the respondent’s is no less a claim against the
deceased’s daughters, Lilia petition for estate than a claim that third
Henson-Cruz, opposed the mandamus parties may make. Section
petition of Ruby. According to 8, Rule 86 of the Rules
her, Ruby had understated recognizes this when it
the value of their late mother’s provides for "Claim of
estate and acted with Executor or Administrator
unconscionable bad faith. Lilia Against an Estate." Hence,
asked that her mother’s by the express terms of
holographic will be disallowed the Rules, the ruling on
and that she be appointed as the extent of the Special
the intestate administratrix. Administrator's
Lilia subsequently moved for commission - effectively, a
the appointment of an interim claim by the special
special administrator of the administrator against the
estate of her late mother. The estate - is the lower
trial court then made court's last word on the
petitioner Briones the special matter and one that is
administrator of the estate. appealable.
Atty Briones submitted the
special administrator’s final
report, and prayed that he be
paid a commission of around
10m. The respondents
opposed the approval of the
final report. Respondents filed
an audit request with the trial
court, which the court granted

Family Savings Pryce Gases is a debtor of Whether or not the No. In this case, BFB did A petition for corporate
Bank, Inc. v. the IFC and the FMO. The Court of Appeals not perfect the appeal rehabilitation is considered a
Pryce Gases, Inc., IFC and the FMO filed a committed a when it failed to file the special proceeding. Thus,
petitioner for rehabilitation reversible error in record on appeal. The the period of appeal
with the RTC of Makati as sustaining the RTC filing of the notice of provided in paragraph 19(b)
Pryce was unable to service decision in appeal was not sufficient of the Interim Rules Relative
it’s debts. dismissing BFB’s because at the time of its to the Implementation
appeal? filing, the Rules required of Batas Pambansa Blg. 129
the filing of the record on for special proceedings shall
appeal and not merely a apply, that is, the period of
notice of appeal. Appeal is appeal shall be 30 days
not a matter of right but a since a record of appeal is
mere statutory required. In this case, BFB
privilege. The party who filed a notice of appeal on 3
seeks to exercise the right November 2003, before the
to appeal must comply effectivity of the AM. Hence,
with the requirements of at the time of filing of BFB’s
the rules, failing in which appeal, the applicable mode
the right to appeal is lost. of appeal is Section 2, Rule
41 of the 1997 Rules of Civil
Procedure which provides
that a party’s appeal by
record on appeal is deemed
perfected as to him with
respect to the subject matter
thereof upon approval of the
record on appeal filed in due
time.
Rule 42 — Petition for Review from the Regional Trial Courts to the Courts of Appeals

Brgy. Sangalang A case was lodged before the Whether or not the No. In the case at bar, it On this score, Section 2,
v. Brgy. Maguihan Sangguinang Bayan. In this Court should is clear that when the Rule 41 of the Rules of
case, petitioner claims that entertain petitioner’s case was appealed to the Court provides that the
the lots are within their appeal? RTC, the latter took appeal to the Court of
territorial jurisdiction, wheras cognizance of the case in Appeals in cases decided by
respondent claims that it is the exercise of its the Regional Trial Court in
within their territorial appellate jurisdiction, not the exercise of its appellate
boundary. The case was then its original jurisdiction. jurisdiction shall be
referred to a hearing Hence, any further appeal by petition for review in
committee. The committee from the RTC Decision accordance with Rule 42.
said that the properties must conform to the Based on the foregoing, it is
belonged to petitioner. provisions of the Rules of apparent that petitioner has
Court dealing with said availed itself of the wrong
matter. remedy. Since the RTC tried
the case in the exercise of
its appellate jurisdiction,
petitioner should have filed a
petition for review under
Rule 42 of the Rules of
Court, instead of an ordinary
appeal under Rule 41.

Fernandez v. CA Private respondent Whether or not


No. The Court of Appeals . The Rule requires that in
Concepcion Olivares filed a mere filing of a
has not yet acquired an appeal by way of Petition
complaint against petitioner motion for extension
jurisdiction over the case For Review, the appeal is
for unlawful entry. of time to file the
because Fernandez deemed perfected as to the
petition for review
merely filed a motion for petitioner upon the timely
automatically
extension of time to file filing of the petition and the
divested the RTC of
petition but not the petition payment of docket and other
its jurisdiction over
itself. In general, in order lawful fees. In the discussion
the case? for a Court to have of the Committee on the
authority to dispose of the revision of the Rules of
case on the merits, it must Court, it was emphasized
acquire jurisdiction over that to perfect the appeal,
the subject matter and the party has to file the
over the parties. petition for review and to pay
Jurisdiction over the the docket fees within the
person is acquired by prescribed period.
service of summons or by
voluntary appearance.
Rule 43 — Appeals from the Court of Tax Appeals and Quasi-Judicial Agencies to the Court of Appeals

St. Martin Funeral Alleged is that private Whether or not Consequently, all such The Court is of the
Homes v. NLRC respondent worked as an appeals from the petitions should hence considered opinion that ever
operations manager of NLRC can be filed forth be initially filed in the since appeals from the
petitioner. However, there directly with the Court of Appeals in strict NLRC to the Supreme Court
was no contract of SC? observance of the were eliminated, the
employment executed doctrine on the hierarchy legislative intendment was
between him and petitioner. of courts as the that the special civil action
His name was also not appropriate forum for the of certiorari was and still is
included in the monthly relief desired. the proper vehicle for judicial
payroll. He was then review of decisions of the
dismissed from his NLRC. Therefore, all
employment for references in the amended
misappropriating 38k. Section 9 of B.P. No. 129 to
Petitioner claims that supposed appeals from the
respondent was not his NLRC to the Supreme Court
employee. are interpreted and hereby
declared to mean and refer
to petitions
for certiorari under Rule 65.
Phillips Seafood Philips Seafood Philippines Whether or not the No. E.O. No. 226 contains no
(Philippines) Corporation was granted a tax review by the Office provision specifically
Corporation v. income holiday for six years of the President of governing the remedy of a
Board of for locating in a less the decisions of party whose application for
Investments, developed area. Petitioner respondent BOI is an ITH has been denied by
used to supply semi- allowed? the BOI. The right to appeal
processed raw materials to is not a constitutional,
Philips Seafood, Inc, an natural or inherent right – it
affiliate corporation, before is a statutory privilege and of
the latter closed due to statutory origin and,
financial difficulties. As they therefore, available only if
were unable to recover from granted or provided by
it’s financial reverses. PSPI statute. The BOI was
stopped operations. Petitioner created by virtue of E.O. No.
acquired title to the plant and 226 at the time when then
informed respondent of President Corazon Aquino
transfer. Respondent was exercising legislative
infromed petitioner that it’s powers under the Freedom
ITH would extend. Constitution. E.O. No. 226
apparently allows two
avenues of appeal from an
action or decision of the
BOI, depending on the
nature of the controversy.
One mode is to elevate an
appeal to the Office of the
President when the action or
decision pertains to either of
these two instances: first, in
the decisions of the BOI
over controversies
concerning the
implementation of the
relevant provisions of E.O
No. 226 that may arise
between registered
enterprises or investors and
government agencies under
Article 7; and second, in an
action of the BOI over
applications for registration
under the investment
priorities plan under Article
36.
Rule 44 — Ordinary Appealed Cases

Mercado v. Sps Alleged in the complaint of Whether or not the No. Under Section 8, Rule 10 of
Espina petitioners is that they are the Court of Appeals the Rules of Court, an
heirs of the late spouses erred in giving due amended complaint
Santiago and Sofronia course to supersedes an original one.
Mercado. Both were owners respondents’ As a consequence, the
of the subject parcel of land. second motion to original complaint is deemed
After the death of Santiago dismiss? withdrawn and no longer
and Sofronia, petitioners considered part of the
inherited the lot. Respondents record. In the present case,
then claimed that the lot was the Amended Complaint is,
theirs, as gthey had bought it thus, treated as an entirely
from Josefa, who in turn new complaint. As such,
bought it from Genivera, who respondents had every right
bought it from Mercado. to move for the dismissal of
Petitioners say that Josefa, the said Amended
through fraudulent Complaint.
machinations, obtained title
over the subject property in
her name. A declaration of
nullity of the deed of sale was
prayed for.
Maricalum Mining Marinduque Mining and Whether the Court's Yes. The general rule is that one
Corp. v. Industrial Corporation was Decisions in DBP v. party's appeal from a
Remington sued by Remington Industrial CA and PNB v. judgment will not inure to the
Industrial Sales Corporation for CA inured to the benefit of a co-party who
payment of around 900 benefit of petitioner failed to appeal; and as
thousand pesos worth of which was not a against the latter, the
construction materials and party to either case judgment will continue to run
other merchandise. The its course until it becomes
complaint was amended 4 final and executory. To this
times, and impleaded other general rule, however, one
parties because they were exception stands out: where
assigness/transferees of the both parties have a
real and personal properties, commonality of interests, the
chattels, machineries, appeal of one is deemed to
equipment, and other assets be the vicarious appeal of
of Marinduque Mining. the other. The adjudication
rendered in DBP v.
CA and PNB v. CA is plain:
private respondent has no
cause of action against
DBP, PNB and their
transferees, including
petitioner, for they are
corporate entities separate
and distinct from
Marinduque Mining, and
cannot be held liable for the
latter's obligations to private
respondent.

Corazon Cruz v. Petitioner Corazon Cruz Was the dismissal In the case at bar, the As held in the case of
MIAA entered into a contract with of the case proper? Court finds that the CA Medida v. CA:An appellee
MIAA for lease of a 1400 committed a reversible who has not himself
square meter property which error in sustaining the appealed cannot obtain from
petitioner would use as a dismissal of the Pasig the appellate court any
commercial arcade for lease case on the ground of affirmative relief other than
to other businesses. improper venue because the ones granted in the
However, according to her, the same was not an error decision of the court below.
respondent did not inform her raised by Cruz who was He cannot impugn the
that part of the leased the appellant before it. correctness of a judgment
premises was subject to Jurisprudence dictates not appealed from by him.
public use (easement), and as that the appellee’s role in He cannot assign such
a result she suffered losses the appeal process is errors as are designed to
such as being unable to confined only to the task have the judgment modified.
obtain a building permit and of refuting the assigned All that said appellee can do
certificate of electrical errors interposed by the is to make a counter-
inspection from the Manila appellant. assignment of errors or to
Electric Company, tenants not argue on issues raised at
paying rent, etc. Since her the trial only for the purpose
tenants could not pay, she of sustaining the judgment in
was unable to pay her own his favor, even on grounds
rent Dec 2004 onwards. And not included in the decision
as some of Cruz’s stalls were of the court a quo nor raised
located in the easement area, in the appellant's
MMDA demolished them, assignment of errors or
causing her damages. She arguments.
tool this into consideration
when computing her rent,
however MIAA terminated the
lease contract.
Rule 45 — Appeal by Certiorari to the Supreme Court

DAR v. Berenguer Respondents were registered Whether or not the No. The CA did not err in there are exceptions to this
owners of several residential CA erred in not treating the petition for rule, to wit: (a) when public
and industrial lands in dismissing certiorari as a petition for welfare and the
Barangay Bibincahan, respondent’s review. There are advancement of public
Sorsogon. The respondents petition for certiorari precedents in that regard. policy dictates; (b) when the
received from the Department broader interest of justice so
of Agrarian Reform that their requires; (c) when the writs
land fell under the CARP. issued are null and void;
This was protested, and or (d) when the questioned
respondents filed in the office order amounts to an
of the DAR an application for oppressive exercise of
exclusion of landholdings judicial authority. As will be
from the CARP coverage. shown forthwith, exception
Without acting on the (c) applies to the present
application, their titles and case.Furthermore, while a
certificates were canceled. motion for reconsideration is
a condition precedent to the
filing of a petition
for certiorari, immediate
recourse to the extraordinary
remedy of certiorari is
warranted where the order is
a patent nullity, as where the
court a quo has no
jurisdiction; where petitioner
was deprived of due process
and there is extreme
urgency for relief; where the
proceedings in the lower
court are a nullity for lack of
due process; where the
proceeding was ex parte or
one in which the petitioner
had no opportunity to object.

Panganiban v. Petitioner was hired by Tara Whether or not the No. Preliminarily, considering the
Tara Trading Ship Trading Shipmanagement on petitioner can file grounds raised by petitioner,
Management Inc., behalf of respondent Shiling both a petition for it appears that he
SDB BHD to work as an oiler. review under Rule denominated this petition as
Aroung 2006, petitioner 45 and a petition for one under Rule 45, but he
started exhibiting signs of certiorari under filed it as both a petition for
mental instability and was Rule 65 review under Rule 45 and a
diagnosed with brief psychotic petition for certiorari under
disorder. DEspite demands Rule 65 of the Rules of
for compensation, Court. The applicable rule is
respondents failed and refued Rule 45, which clearly
to comply with their provides that decisions, final
contractual obligations. orders or resolutions of the
CA in any case, regardless
of the nature of the action or
proceeding involved, may be
appealed to this Court
through a petition for review.
This remedy is a
continuation of the appellate
process over the original
case. Recourse under Rule
65 cannot be allowed either
as an add-on or as a
substitute for appeal.The
procedural infirmity
notwithstanding, the Court
shall treat this petition as
one filed under Rule
45 only and shall consider
the alleged grave abuse of
discretion on the part of the
CA as an allegation of
reversible error

Rule 47 — Annulment of Judgments or Final Orders and Resolutions

Llamas v. CA On November 1978, Whether or not Rule No. The remedy cannot Section 1, Rule 47 of the
petitioners conspired and, 47 may be applied be resorted to when the Rules of Court, limits the
despite knowing that their to criminal cases RTC judgment being scope of the remedy of
parcel of land in Paranaque questioned was rendered annulment of judgment to
was mortgaged to Rural Bank in a criminal case. The the following:
of Imus, seel such to Conrado 2000 Revised Rules of
P Avilla. Petitioners were Criminal Procedure itself Section 1. Coverage. — This
doung duilty and sentenced to does not permit such Rule shall govern the
two months imprisonment. recourse, for it excluded annulment by the Court of
Rule 47 from the Appeals of judgments or
enumeration of the final orders and resolutions
provisions of the 1997 in civil actions of Regional
Revised Rules of Civil Trial Courts for which the
Procedure which have ordinary remedies of new
suppletory application to trial, appeal, petition for
criminal cases. relief or other appropriate
remedies are no longer
available through no fault of
the petitioner.
Sps. Arcenas v. On January 23, 2002, the Whether or not the Yes. Section 1, Rule 47 Petitioner's claim that she
Queen City spouses Dolores and Oscar CA is correct in provides that it does not was present when
Development Arcenas filed with the dismissing the allow a direct recourse to respondent bank's counsel
Bank Regional Trial Court (RTC) of petition for a petition for annulment of moved for the issuance of
Roxas City, an Action for annulment of judgment if other the order of non-suit against
Declaratory Relief against judgement? appropriate remedies are her was not proven by any
respondent Queen City available, such as a evidence.There was indeed
Development Bank, docketed petition for new trial, a failure to show, to our
as Civil Case No. V-006-01- appeal or a petition for satisfaction, that petitioner
2002, and was raffled off to relief.17 If petitioner fails to could not have availed of the
Branch 15. The Spouses avail of these remedies ordinary and appropriate
Arcenas prayed for the without sufficient remedies under the Rules.
declaration of their rights as justification, she cannot Thus, she cannot resort to
lessors under the contract of resort to the action for the remedy under Rule 47 of
lease. annulment of judgment the Rules; otherwise, she
under Rule 47, for would benefit from her
otherwise, she would inaction or negligence.
benefit from her inaction
or negligence Petitioner
tries to justify her failure to
avail of the appropriate
remedies on a promise of
settlement. However,
such promise was not an
excuse for petitioner's
counsel not to lift the
order of non-suit and to
file a petition for relief.
Genato A lot situated in Caloocan Whether or not We disagree with the Well entrenched in our
Investments Inc. city was registered under the CA was reasoning of the CA and jurisdiction is the doctrine
v. Barrientos the name of the petitioner. correct in respondents that that a court has no power to
Due to an alleged dismissing on petitioner in this particular do so, as that action may
deficiency in real property the grounds that case should have filed lead to confusion and
taxes, the office of the city the petition for either an action for seriously hinder the
treasurer of Caloocan city annulment of reconveyance or administration of justice
sold the land at public judgement is not annulment of the auction
auction, and respondent the proper sale, because to do so
was the highest bidder and remedy would have required the
the office of the city court hearing the action to
treasurer issued a modify or interfere with
certificate of sale of the judgment or order of
delinquent property. another co-equal court,
Petitioner was not aware of especially in this case
any of the proceedings where the said judgment
before the office of the city ororder had attained
treasurer. REspondent finality
prayed for the
consolidation of ownership
of the proper, despite the
fact that the delinquency
sale only involved one lot
of the property.

Rule 51 — Judgment

Natalia Realty v. Petitioner alleged that Whether or not Yes. Judge Querubin The general rule is when a
CA respondents were the writ of
could have ended the court's judgment or order
occupying and illegally execution to
delays had he readily becomes final and
squatting on certain should have
complied with the executory, it is the
portions of the subject been allowed February 6, 1995 ministerial duty of the trial
property. Private Resolution of the Court of court to issue a writ of
respondents claim that Appeals ordering him to execution to enforce the
they are the wners of their execute the final orders judgment or order.29 In this
respoective houses and and decision in Civil Case case, because of the
lots that have been in their No. 359-A. Instead, Judge innumerable delays, the
possession before the Querubin wrote a letter to enforcement of the final
outbreak of World War II the Court of Appeals orders and decision has
asking it to specify the been long overdue. The
final orders and decision delays are partly attributable
that should be enforced. to the fact that the case went
In response to the query, through a number of
the Court of Appeals presiding judges.
issued another resolution
identifying these final
orders and decision.While
we agree with petitioner
that Judge Querubin’s
query is not provided for
in the Rules of Court, it is
nevertheless not
prohibited.
Philippine Hawk A vehicular accident Whether or not The procedure in the Rule 51 SEC. 8. Questions
Corp. v. Lee occurred in Quezon which the court of Supreme Court being that may be decided. -- No
killed respondent’s appeals erred in generally the same as that error which does not affect
husband and caused awarding other in the Court of Appeals, the jurisdiction over the
respondent physical kinds of unless otherwise indicated subject matter or the validity
injuries. The accident damages in favor (see Secs. 2 and 4, Rule of the judgment appealed
involved a motorcycle, a of respondent, 56), it has been held that from or the proceedings
passenger jeek, and a buy. when this was the latter is clothed with therein will be considered
The buy was owned by not appealed ample authority to review unless stated in the
petitioner and was bieng from the trial matters, even if they are assignment of errors, or
driven by Margarito Avila courts decision not assigned as errors on closely related to or
appeal, if it finds that their dependent on an assigned
consideration is error and properly argued in
necessary in arriving at a the brief, save as the court
just decision of the case. pass upon plain errors and
clerical errors.1. Sec. 8,
which is an amendment of
the former Sec. 7 of this
Rule, now includes some
substantial changes in the
rules on assignment of
errors. The basic procedural
rule is that only errors
claimed and assigned by a
party will be considered by
the court, except errors
affecting its jurisdiction over
the subject matter. To this
exception has now been
added errors affecting the
validity of the judgment
appealed from or the
proceedings therein.Also,
even if the error complained
of by a party is not expressly
stated in his assignment of
errors but the same is
closely related to or
dependent on an assigned
error and properly argued in
his brief, such error may
now be considered by the
court. These changes are of
jurisprudential origin
Rules 52 and 53 — Motions for Reconsideration and New Trial

Gonzales III v. Christian Kalaw filed separate Whether on not the Yes. Gonzales cannot be Even if we consider this
Office of the charges with the PNP IAS Office of the guilty of gross neglect of provision to be mandatory,
President against Manila Police District president’s ruling duty and/or inefficiency the period it requires cannot
Senior Inspector Mendoza that Gonzales had since he acted on the apply to Gonzales since he
and four others for robbery, been grossly case forwarded to him is a Deputy Ombudsman
grave threat, robbery negligent for taking within nine days. In finding whose obligation is to review
extortion, and physical injury. nine days instead of Gonzales guilty, the the case; he is not simply a
Police Senior Superintendent five days to review OP72 relied on Section 8, Hearing Officer tasked with
Atty. Clarence Guinto filed an the motion for Rule III of Administrative the initial resolution of the
administrative charge for reconsideration was Order No. 7 (or the Rules motion. In Section 6 of
grave isconduct with the baseless of Procedure of the Office Administrative Order No. 7
National Police Commission of the Ombudsman, series on the resolution of the case
against Mendoza based on of 1990, as amended) in and submission of the
the allegations of Kalaw.After ruling that Gonzales proposed decision, the
preparing a draft decision on should have acted on period for resolving the case
MEdoza’s case, the deputy Mendoza’s Motion for does not cover the period
obudsman, who found Reconsideration within within which it should be
Mendoza guilty, forwarded the five days. reviewed
records to the office of the
ombudsman. This was
approved. Pending action by
ombudsman, Mendoza
hijacked a toursit bus and
held 21 foreign tourists and
four Filipino tour assistants
hostages. Mendoza died.
Ombudsman was found guilty
of gross misconduct in
handling the case against
Mendoza. Gonzales, the
deputy pmbudsman, was
charged for gross neglect of
duty.
Rule 59 — Receivership

Vivares v. Reyes Severino Reyes was the Whether or not No. We sustain the CA Sec. 3, Rule 59 of the 1997
father of respondent Jose receivership was ruling that the trial court Rules of Civil
Reyes and Torcuato Reyes. justified acted arbitrarily in Procedure.Anchored on this
Upon his death, the two came granting the petition for rule, the trial court should
upon their inheriance and had appointment of a receiver have dispensed with the
an oral partition of properties. as "there was no sufficient services of the receiver,
When Torcuato dies, his will cause or reason to justify more so considering that the
was up for probate. Petitioner, placing the disputed alleged fraud put forward to
the executor of the will when properties under justify the receivership was
Ignaling (also petitioner ) was receivership." Petitioners not at all
declared a lawful heir to cannot now impugn the established.Petitioners
Torcuato. Believing that oral partition entered into advance the issue that the
Torcuato did not receive his by Torcuato and receivership should not be
full share in the estate of respondent and hence recalled simply because the
Severino, petitioners instituted cannot also assail the adverse party offers to post
an action for partition and transfers made by a counterbond. At the
recovery of real estate. respondent of the lots outset, we find that this
which were subject of said issue is barred by
agreement, considering estoppel.10 While the CA
that Torcuato also sold made a statement that the
properties based on said trial court should have
verbal arrangement. discharged the appointed
receiver on the basis of the
proposed counterbond, such
opinion does not jibe with
the import of Sec. 3, Rule
59. The rule states that the
"application may be denied
or the receiver discharged."
In statutory construction, the
word "may" has always been
construed as permissive. If
the intent is to make it
mandatory or ministerial for
the trial court to order the
recall of the receiver upon
the offer to post a
counterbond, then the court
should have used the word
"shall." Thus, the trial court
has to consider the posting
of the counterbond in
addition to other reasons
presented by the offeror why
the receivership has to be
set aside.
Rule 60 — Replevin

Smart Astorga was employed by Whether or not Yes. Contrary to the CA’s
Communications Smart Communications as the replevin was ratiocination, the RTC
v. Astorga a Distric Sales manager of proper rightfully assumed
the Corporate SAles jurisdiction over the suit
Marketing Group. SMART and acted well within its
launghed an organizational discretion in denying
realignment to achieve Astorga’s motion to
more efficent operations. dismiss. SMART’s
Part of the reorganization demand for payment of
was the outsourcing of the the market value of the
marketing and sales force. car or, in the alternative,
To soften the blow, SNMI the surrender of the car,
agreed to absorb the is not a labor, but a civil,
personnel who would be dispute. It involves the
recommended by SMART, relationship of debtor and
but Astorga landed last in creditor rather than
the performance employee-employer
evaluation. SMART offered relations.33 As such, the
her a supervisory position dispute falls within the
in the customer care jurisdiction of the regular
department but she courts.
refused due to a lower
salary rank and rate.
DEspite the abolition of her
office, she continued
reporting for work, so
SMART terminated her.
Astorga then filed a
complaint for illegal
dismissal. In the
meantime, SMART sent a
letter to Astorga
demanding that she pay
the current market value of
the Honda which was
given to her under the
company’s car plan
program. She did not do
this, and SMART filed a
replevin.
Rule 63 — Declaratory Relief

Monetary Board Respondent established a Whether or not the No. the decision of the Declaratory relief is defined
v. Philippine pension loan product for bona petition for BSP Monetary Board as an action by any person
Veterans Bank fide veterans or their surviving declaratory relief is cannot be a proper interested in a deed, will,
spouses, as well as salary proper subject matter for a contract or other written
loan product for teachers and petition for declaratory instrument, executive order
low salaried employees to relief since it was issued or resolution, to determine
provide financial assitance to by the BSP Monetary any question of construction
veterans and teachers. Board inthe exercise of its or validity arising from the
Respondent devised a quasi-judicial powers or instrument, executive order
program by charging a functions. or regulation, or statute; and
premium in the form of a for a declaration of his rights
higher fee known as credit and duties thereunder. The
Redemption Fund from said only issue that may be
borrowers. An exampination raised in such a petition is
was conducted by the the question of construction
Supervision and Examination or validity of provisions in an
Department of the Banko instrument or statute.9 Ergo,
Sentral ng Pilipinas, which the Court, in CJH
found, among other things, Development Corporation v.
that respondent’s collection of Bureau of Internal
premiums from the proceeds Revenue,10 held that in the
of the various salary and same manner that court
pension loans violated decisions cannot be the
Section 54 of RA 8791, proper subjects of a petition
whichs tates that banks shall for declaratory relief,
not directly engage in the decisions of quasijudicial
insurance business as agencies cannot be subjects
insurer. Respondent wrote a of a petition for declaratory
letter justifying the existence relief for the simple reason
of the CRF. Petitioners issued that if a party is not
a resolution directing agreeable to a decision
respondent’s trust and either on questions of law or
investment department to of fact, it may avail of the
return to the borrowers all the various remedies provided
balances of the CRF. by the Rules of Court.
Rule 65 — Certiorari, Prohibition and Mandamus

Dacudao v. Sec. Petitioners were among the Did petitioners No. .Firstly, petitioners In Bañez, Jr. v.
of Justice investors whom Angeles and property bring their have unduly disregarded Concepcion, we explained
his associates in the Legacy petition for the hierarchy of courts by why, to wit:The Court must
Group allegedly defrauded certiorari, coming directly to the enjoin the observance of the
through the Legacy Group’s prohibition, and Court with their petition for policy on the hierarchy of
buy back agreement that mandamus directly certiorari, prohibition and courts, and now affirms that
earned them check payments to the court? mandamus without the policy is not to be
that were dishonored. After tendering therein any ignored without serious
their written demands went special, important or consequences. The
unheeded, they initiated a compelling reason to strictness of the policy is
number of charges for justify the direct filing of designed to shield the Court
syndicated estafa against the petition.We from having to deal with
Angeles. emphasize that the causes that are also well
concurrence of jurisdiction within the competence of the
among the Supreme lower courts, and thus leave
Court, Court of Appeals time to the Court to deal with
and the Regional Trial the more fundamental and
Courts to issue the writs more essential tasks that the
of certiorari, prohibition, Constitution has assigned to
mandamus, quo warranto, it. The Court may act on
habeas corpus and petitions for the
injunction did not give extraordinary writs of
petitioners the certiorari, prohibition and
unrestricted freedom of mandamus only when
choice of court forum. An absolutely necessary or
undue disregard of this when serious and important
policy against direct resort reasons exist to justify an
to the Court will cause the exception to the policy.
dismissal of the recourse.

Bank of Traders Royal Bank (TRB) Whether or not the Yes. . Bancommerce Section 1, Rule 65 of the
Commerce v. proposed to sell to petitioner direct filing of the invoked certain Rules of Court provides that
Radio Philippines Bank of Commerce petition for ceritorari recognized exceptions to a petition for certiorari may
Network, (Bancommerce) for P10.4 by Bancommerce the rule.12 It had to forego only be filed when there is
billion its banking business was valid? the filing of the required no plain, speedy, and
consisting of specified assets motion for reconsideration adequate remedy in the
and liabilities. Bancommerce of the assailed RTC Order course of law. Since a
agreed subject to prior because a) there was an motion for reconsideration is
Bangko Sentral ng Pilipinas' urgent necessity for the generally regarded as a
(BSP's) approval of their CA to resolve the plain, speedy, and adequate
Purchase and Assumption (P questions it raised and remedy, the failure to first
& A) Agreement. any further delay would take recourse to is usually
prejudice its interests; b) regarded as fatal omission
under the circumstances,
a motion for
reconsideration would
have been useless; c)
Bancommerce had been
deprived of its right to due
process when the RTC
issued the challenged
order ex parte, depriving it
of an opportunity to
object; and d) the issues
raised were purely of
law.In this case, the
records amply show that
Bancommerce’s action fell
within the recognized
exceptions to the need to
file a motion for
reconsideration before
filing a petition for
certiorari.
Yusay v. CA Petitioners owned a parcel of Does prohibition lie No. Here, however, the The rule and relevant
land in Mandaluyong. Half against remedy of prohibition was jurisprudence indicate that
was used as residence, the expropriation not called for, considering prohibition was not available
rest rented out to nine other that only a resolution to the petitioners as a
families.The land was their expressing the desire of remedy against the adoption
only source of income. The the Sangguniang of Resolution No. 552, for
mayor tried to expropriate the Panglungsod to the Sangguniang
land to develop it for cost expropriate the Panglungsod, by such
housing for priveleged but petitioners’ property was adoption, was not exercising
deserving city inhabitants. issued. As of then, it was judicial, quasi-judicial or
premature for the ministerial functions, but
petitioners to mount any only expressing its collective
judicial challenge, for sentiment or opinion. Verily,
thepower of eminent there can be no prohibition
domain could be against a procedure
exercised by the City only whereby the immediate
through the filing of a possession of the land under
verified complaint in the expropriation proceedings
proper court. Before the may be taken, provided
City as the expropriating always that due provision is
authority filed such made to secure the prompt
verified complaint, no adjudication and payment of
expropriation proceeding just compensation to the
could be said to exist. owner. This bar against
Until then, the petitioners prohibition comes from the
as the owners could not nature of the power of
also be deprived of their eminent domain as
property under the power necessitating the taking of
of eminent domain. private land intended for
public use, and the interest
of the affected landowner is
thus made subordinate to
the power of the State. Once
the State decides to
exercise its power of
eminent domain, the power
of judicial review becomes
limited in scope, and the
courts will be left to
determine the appropriate
amount of just compensation
to be paid to the affected
landowners.
Star Special Petitioners were the owners Whether or not the No. Considering that the It has been held, however,
Watchman and of two parcels of land in remedy of COA still retained its that a resort to the remedy
Detective Agency Puerto Princesa. Before mandamus is primary jurisdiction to of mandamus is improper if
v. Puerto Puerto Princesa became a proper to compel adjudicate money claim, the standard modes of
Princesa City city, the national governmnet respondents to petitioners should have procedure and forms of
established a military camp, comply with the filed a petition for remedy are still available
known as Western Command. decision certiorari with this Court and capable of affording
Petitioners property was used pursuant to Section 50 of relief.
as a road right-of-way leading P.D. No. 1445. Hence, the
to the camp, called Wescom COA's refusal to act did
road. Petitioners filed an not leave the petitioners
action for payment of just without any remedy at all.
compensation against Puerto
Princessa. The RTC ruled in
favor of petitioners. The write
of execution was issued
directing respondents to
satisfy the money judgement.
1. Samson v. Gabor
G.R. No. 182970

Facts of the Case:

Spouses Gabor own a parcel of land in Rizal. They sold the land to petitioner, who
executed a deed of sale to Ramos. Respondent filed an action for legal redemption
with the RTC. Petitioner and Ramos then executed an agreement revoking the
transfer. Respondent Spouses Gabor are registered owners of a parcel of land in
Rizal Province.

Procedural history
The RTC dismissed the suit but the CA reversed and upheld the Respondents’ right
of legal redemption.

Statement of Issue(s):

Whether or not the complaint states a cause of action.

Holding:

No. Section 2, Rule 2 of the Rules of Court provides that a cause of action is the
act or omission by which a party violates the right of another. In this case, aside
from the fact the respondent spouses had mortgaged the property to respondent
bank, there is no other allegation of act or omission on part of respondent bank in
violation of the right of petitioner.
2. Uniwide Holdings Inc. v. Alexander Cruz
G.R. No. 171456

Facts of the Case:


Uniwide entered into a franchise agreeemnt with Cruz. It was agreed that it would
be five years and it would be that way to establish and operate a Uniwide Family
Store. AArticle 10.2 of the Agreement called for the franchisee to pay a monthly
serice fee. Inc ase of delay, it was provided that there would be charged an interest.
Article 27.5 provided that the franchisee consented to the exclusive jurisdiction of the
courts of Quezon.

Procedural History:

When Cruz’s accounts remained unsettled, UHI filed a complaint for collection of
sum of money before the Paranaque City RTC. Cruz filed a motion to dismiss on
ground of improper venue, which was granted by the RTC. Hence, this petition for
review on certiorari.

Statement of Issue(s):

Whether or not a case based on several causes of action is dismissible on ground of


improper venue?

Holding:

When construing with Section 5, Rule 2 (which states the general rule where
personal actions may be commenced and tried where the plaintiff or defendant
resides, at the election of the plaintiff, the exception being that parties may, before
filing of action, validly agree in writing on an exclusive venue) where there is a
joinder of causes of action between the same parties one of which does not arise out
of the contract where the exclusive venue was stipulated upon, the complaint, as in
the one at bar, may be brought before other venues provided that such other causes
of action falls within the jurisdiction of the court and the venue lies therein. In this
case, the deeds, unlike the Agreement, bear no exclusive venue stipulation with
respect to the causes of action thereunder. Hence, the general rule on venue applies
- that the complaint may be filed in the place where the plaintiff or defendant resides.
The restriction should be strictly construed as relating solely to the agreement for
which the exclusive venue stipulation is embodied. Expanding the scope of such
limitation on a contracting party will create unwarranted restrictions which the parties
might find unintended or worse, arbitrary and oppressive.
3. Lourdes Suits v. Binaro
G.R. No. 204729

Facts of the Case:

Two executive contracts were executed with respondent Binaro for room
accomodations for two groups of students, as petitioner owns a hotel. Although
respondent paid the total contract price, it is claimed that there was an unpaid
balance for damages to furniture.

Procedural History:

Petitioner filed a Statement of Claim for collection of sum of money with damages
before the Metropolitan Trial Court of Makati. The MeTC dismissed the complaint
with prejudice for lack of cause of action. Petitioner filed a petition for certiorari
before the RTC, arguing that a dismissal based on the ground that the complaint
states no cause of action cannot be deemed a dismissal with prejudice under the
Rules, and that lack of cause of action is not a valid ground for dismissal. RTC ruled
against petitioner. Petitioner filed an MR which was denied. Hence, this petition.

Statement of Issue(s):

Whether or not the MeTC committed grave abuse of discretion in dismissing with
prejudice based on lack of cause of action.

Holding:
The courts are not precluded from dismissing a case for lack of cause of action. In
the case of Macaslang v. Zamora, the Court provided that failure to state cause of
action and lack of cause of action are really different from each other. On the one
hand, failure to state cause of action refers to insufficiency of the pleading, and is a
ground for dismissal under Rule 16 of the Rules of Court. On the other hand, lack of
cause of action refers to a situation where the evidence does not prove the cause of
action alleged in the pleading.
4. Iron and Steel Authority v. Court of Appeals
G.R. No. 102976

Facts of the Case:


Iron and Steel Authority was created with the power to initiate expropriation of land
required for basic iron and steel facilities. The National Steel Corporation had an
expansion program, and thus the president issued a proclamation withdrawing from
sale or settlement a large track of public land in Iligan. Since certain parts of such
public land were occupied by non-operational fertilizer plants owned by respondent,
an LOI was issued directing the National Steel Corporation to negotiate with
respondent for compensation of occupancy rights.

Procedural History:

ISA commenced eminent domain proceedings against MCFC in the RTC of Iligan
City. While the trial was ongoing, the statutory existence of petitioner ISA expired.
MCFC then filed a Motion to Dismiss on the ground that no valid judgment could be
rendered against ISA which had ceased to be a juridical person, which the trial court
granted. Petitioner moved for reconsideration, arguing that despite expiration of term,
its juridical existence continued until winding up of affairs could be completed, and
alternatively, that the Republic of the Philippines, being the real party-in-interest,
should be allowed to be substituted for petitioner. The trial court denied the MR. The
CA denied on appeal. Hence, this petition for review.

Statement of Issue(s):

Whether or not the Republic of the Philippines is entitled to be substituted for ISA in
view of the expiration of ISA’s term.

Holding:

In this case, petitioner is a non-incorporated agency or instrumentality of the


Republic. Thus, its powers, duties, functions, assets, and liabilities are properly
regarded as folded back into the Government of the Republic. It is elementary that
when the statutory term of a non-incorporated agency expires, the powers, duties
and functions, as well as the assets and liabilities of that agency revert back to and
are re-assumed by the Republic, in the absence of special provisions of law. When
the expiring agency is an incorporated one, the consequences of such expiry must
be looked for in the charter and supplementarily in the provisions of the Corporation
Code. Rule 3, Section 3 of the Rules of Court expressly recognize the role of
representative parties, that a party authorized by statute may sue or be sued without
joining the party for whose benefit the action is presented or defended. In this case,
ISA instituted the expropriation proceedings in its capacity as an agent or
representative of the Republic pursuant to its authority under PD 272. From these
premises, the expiration of ISA’s statutory term did not justify dismissal of the
eminent domain proceedings.
5. Spouses Oco v. Limbaring
G.R. No. 161298

Facts of the Case:

Sabas Limbaring subdivided his lot and then executed deeds of sale in favor of
Jenniferand Sarah Jane Limbaring. The daughter of Sabas, Percita Oco, filed a
perjry case against her uncle who was the father of Jenifer and Sarah. In the pretrial,
it was agreed that the parties would reconvey two parcels of land to petitioner, and
that respondent would pay expenses incurred to transfer the title. But Percita left
without paying.

Procedural History:

Respondent filed against petitioner. Petitioner filed a Motion to Dismiss on the


ground that respondent was not the real party in interest. In his opposition to the
Motion to Dismiss, respondent contended that he was a trustor. RTC denied the
Motion to Dismiss. After filing their Comments to Respondent’s Formal Offer of
Exhibits, petitioner filed a Demurrer to Evidence. The RTC granted the demurrer and
dismissed the Complaint on the ground that respondent was not the real party in
interest. On appeal, the CA reversed

Statement of Issue(s):

Whether or not Respondent was a real party in interest in the suit to rescind the
Deeds of Reconvenyance.

Holding:

Under Article 1448 of the Civil Code on implied trusts, if the person to whom the title
is conveyed is a child, legitimate or illegitimate, of the one paying the price of the
sale, no trust is implied by law, it being disputably presumed that there is a gift in
favor of the child. In this case, respondent’s alleged acts - paying the price of subject
properties, naming his children as owners - raise the presumption that a gift was
effected in their favor. Respondent failed to rebut this presumption. Absent any clear
proof that a trust was created, he cannot be deemed a real party in interest.
6. Golangco v. Fung
G.R. No. 157952

Facts of the Case:

Respondent issued an office memorandum maliciously imputing against petitioner


that he had committed bribery. This was sent to petitioner’s superiors in POEA and
this casued prejudice to petitioner.
Procedural History:

A case was then filed where petitioner alleged that the imputations against him were
wrong and libelous. The prosecution therein requested that a subpoena ad
testificandum be issued and served on Atty Ramos, Resident Ombudsman of POEA,
to compel him to testify in the criminal case. Nevertheless, the prosecution failed to
present Ramos as a witness because no subpoena had been issued. Consequently,
the RTC issued an order terminating the prosecution’s presentation of evidence.
Petitioner filed a special civil action for certiorari, claiming that the RTC judge
committed grave abuse of discretion for not issuing the subpoena to require Ramos
to appear and testify. The Court of Appeals dismissed the petition.

Statement of Issue(s):

Whether or not the Court of Appeals correctly ruled on the petition for certiorari of the
petitioner.

Holding:

The OSG is to represent “the Government in the Supreme Court and the Court of
Appeals in all criminal proceedings; represent the Government and its officers in the
Supreme Court, the Court of Appeals, and all other courts or tribunals in all civil
actions and special proceedings in which the Government or any officer thereof in
his official capacity is a party, according to the Administrative code. In this case, the
People of the Philippines were indispensable parties, and petitioner not joining the
People as a party in his action for certiorari in the Court of Appeals was fatal and
enough cause for summary rejection of his petition. Although the petition for
certiorari bore the conformity of the public prosecutor, such conformity alone did not
suffice. The authority of the City Prosecutor or his assistant to appear for and
represent the People was confined only to the proceedings in the trial court
7. Equitable PCI Bank v. Heirs of Tiu
G.R. No. 178529

Facts of the Case:

Atonio Tiu wanted to secure loans obtained by Ching from Petitioner Equitable Bank,
so a real estate mortgage in favor of petitioner was issued. Tui also executed an an
Amendement to the Real Estate Mortgage which increased the amount secured by
the mortgage. This was signed by his wife which said, “with my marital consent”. Tiu
died, and the loan remained unsettled.
Procedural History:

Petitioner filed a petitioner for sale for the foreclosure. Respondents Heirs of Tiu filed
a Complaint before the RTC of Tacloban against petitioner and the Clerk of Court for
annulment of the AREM, alleging that the AREM is without force and effect, having
been executed without the valid consent of the wife of mortgage who at the time of
execution was already suffering from advanced Alzheimer’s Disease and hence
incapable of giving consent. Petitioner filed a Motion to Dismiss on the ground the
respondents were not the real parties-in-interest. RTC denied the Motion. Petitioner
filed a Petition for Certiorari, Prohibition, and Mandamus before the Court of Appeals
which affirmed the decision of the RTC. Hence, the present petition.

Statement of Issue(s):

Whether or not the complaint filed by Respondents, without impleading Matilde who
must also be Antonio’s heir and who was principally obliged under the AREM, is
dismissible for lack of cause of action.

Holding:
Rule 3, Section 2 of the Rules of Court provides that as a general rule, every action
must be prosecuted or defended in the name of the real party in interest. Section 3 of
the same Rule provides that the beneficiary shall be included in the title of the case
and shall be deemed to be the real party in interest. In this case, since the
mortgaged property is presumed conjugal, Matilde is obliged principally under the
AREM. Therefore, she is the real party in interest and hence, the action must be
prosecuted in her name as she stands to be benefited or injured in the action.

8. Department of Health v. Phil Pharmawealth Inc


Facts of the Case:
An administrative order was issued by the DOH. This gave out guidelines and
procedures for the accreditation of government suppliers for pharmaceutical
products for sale or distribution to the public. This was then amended by other
administrative orders. These orders provided that the accreditation may be recalled,
suspended, or revoked after due deliberation, hearing, and notice by the DOH
Accreditation Committee. The DOH then issued a Memorandum hearing and notice
by the DOH accreditation Committee. Next, a list and category of sanctions to be
imposed on accredited government suppliers of pharmaceuticals. The DOH then
issues a memorandum which invited representatives of 24 accredited drug
companies to a meeting where they were directed to submit within 10 days their
respective explanations on the adverse findings covering their products. Instead of
submitting a written explanation, PPI sent a late letter stating that it referred the
matter to their lawyers. Usec Galon then informed PPI that it’s accreditation had
been suspended.

Procedural History:

A complaint seeking to declare null and void the relevant DOH administrative
issuance's was filed before the RTC of Pasig City against DOH, former Sec
Romualdez and Usec Galon, alleging undue delegation violation of due process.
Petitioners moved for dismissal, claiming that the case was one against the State.
The trial court dismissed the case on the ground of principle of state immunity from
suit. PPI appealed to the CA. The CA reversed the trial court decision.

Statement of Issue(s):

Whether or not the case should be dismissed for being a suit against the State.

Holding:
As a general rule, a state may not be sued. However, if it consents, either expressly
or impliedly, then it may be the subject of a suit. An unincorporated government
agency without any separate juridical personality of its own enjoys immunity from suit
because it is invested with an inherent power of sovereignty. In this case, the DOH,
being an unincorporated agency of the government, can validly invoke the defense
of immunity from suit because it has not consented to be sued. The doctrine of state
immunity extends also to complaints filed against state officials for acts done in the
discharge and performance of their duties.

9. Air Transportation Office v. Spouses Ramos


G.R. No. 159402
Facts of the Case:

It was discovered by Spouses Ramos that a portion of their land was being used as
part of the runway of Loakan airport, operated by petitioner Air Transportation Office
(ATO). The respondents, Spouses Ramos, then agreed to sell the land to ATO, but
ATO did not pay.

Procedural History:

The respondents filed an action for collection against ATO. In their answer, ATO and
co-defendants invoked the issuance of Proclamation No. 1358, where President
Marcos reserved parcels of land including respondents’ for use of the Loakan Airport.
They asserted that the RTC had no jurisdiction to entertain the action without State’s
consent considering that the deed of sale had been entered into in the performance
of governmental functions. The RTC rendered its decision. On appeal, the CA
affirmed the RTC decision.

Statement of Issue(s):

Whether or not the ATO could be sued without the State’s consent.

Holding:

There is a need to distinguish between an unincorporated government agency


performing governmental functions and one performing proprietary functions. Section
3, Article XVI of the 1987 Constitution expressly provides the immunity of the State
from suit. In exercising the right of eminent domain, the State exercises its jus imperii,
as distinguished from its proprietary rights, or jus gestionis; yet, even in that area,
where private property had been taken in expropriation without just compensation
being paid, the defense of immunity from suit could not be set up by the State
against an action for payment by the owners. Immunity has not been upheld in favor
of the proprietary functions whose function was not in pursuit of a necessary function
of government, but was essentially a business. In previous jurisprudence, the Court
declared that the CAA, predecessor of ATO, is an agency not immune from suit, it
being engaged in functions pertaining to a private entity. Therefore, the CA thereby
correctly appreciated the juridical character of the ATO as an agency of the
Government not performing purely governmental or sovereign function, but was
instead involved in the management of Loakan Airport, an activity that was not
exclusive prerogative of the State in its sovereign capacity. Furthermore, the doctrine
of sovereign immunity cannot be successfully invoked to defeat a valid claim for
compensation arising from the taking without just compensation and without proper
expropriation proceedings being first resorted to of the plaintiffs’ property.

10. Lotte Phil Co Inc v. Dela Cruz


G.R. No. 166302
Facts of the Case:

Petitioner hired respondents to work in a confectionery facility. Petitioner then


entered into a contract with Maintenance and Janitorial Services to provide
manpower for needed maintenance, utility, janitorial, and other services. Respondent
then was fired.

Procedural History:

Respondent lodged a labor complaint against both petitioner Lotte and 7J for illegal
dismissal. The Labor Arbiter rendered judgment declaring 7J as employer of
respondents and also found 7J guilty of illegal dismissal. Respondents appealed to
the NLRC praying that Lotte be declared as their direct employer because 7J is
merely a labor-only contractor. NLRC affirmed the LA decision. Respondents filed
with the Court of Appeal, which reversed the decision and declared Lotte as the real
employer of respondents and that 7J was merely a labor-only contractor and agent
of Lotte. Hence, this petition for review on certiorari, alleging that the Court of
Appeals should have dismissed the petition for failure to implead 7J who is an
indispensable party, pursuant to Section 3, Rule 46 of the Rules of Court.

Statement of Issue(s):

Whether or not 7J is an indispensable party and should have been impleaded in


respondents’ petition in the Court of Appeals.

Holding:

What is an indespensible party? It is important to answer this first before we delve


into the discussion. An indispensable party is a party in interest without whom no
final determination can be had of an action. The joinder of indispensable parties is
mandatory. The absence of an indispensable party renders all subsequent actions of
the court null and void. In this case, 7J is an indispensable party. It is a party in
interest because it will be affected by the outcome of the case. No final ruling on this
matter can be had without impleading 7J, whose inclusion is necessary for the
effective and complete resolution of the case and in order to accord all parties with
due process.

11. Chua v. Torres


G.R. No. 151900
Facts of the Case:

An RBC check in favor of Caltex Service Center was issued by Chua in payment of
purchase of oil. But the check was dishonored by drawee bank. Beltran did not
bother to check who had issued the check and instead instituted an action against
petitioner Christine Chua for BP 22. The policce officers went to her residence, in the
auto repair shop of her brother, and even her uni.

Procedural History:

Christine filed a complaint for damages before the RTC of Caloocan city, impleading
also her brother Jonathan as a necessary co-plaintiff. Named as defendants therein
were respondents Torres and Beltran. Upon motion of respondents, the RTC
ordered the dismissal of complainant on the ground that Chua had not executed a
certification against forum shopping. After the RTC denied the motion for
reconsideration, the matter was elevated directly to the Court by way of petition for
review under Rule 45.

Statement of Issue(s):

Whether or not a co-plaintiff impleaded only as necessary party, who has no claim
for relief, should also make a certification against forum shopping.

Holding:

Section 5, Rule 7 of the Rules of Court makes no distinctions that would expressly
exempt a necessary party from executing the certification against forum shopping.
Nevertheless, a misjoined party plaintiff has no business participating in the case as
a plaintiff in the first place, and it would make little sense to require the misjoined
party in complying with all the requirement expected of plaintiff. At the same time,
Section 11, Rule 3 of the Rules of Court provides that neither misjoinder nor non-
joinder of parties is ground for dismissal of action. Clearly, misjoinder of parties is not
fatal to the complaint. The rule prohibits dismissal of a suit on the ground of non-
joinder or misjoinder of parties.

12. Office of the City Mayor of Paranaque City v. Ebio


G.R. No. 178411
Facts of the Case:

Respondents claim to be the absolute owners of land in Paranaque. The land was
due to an accretion of Cut-cut creek. It was asserted by respondents that the original
occupant was their great grandfather Jose Vitalez. Jose gave the land to his son,
and his son continuously and exclusively occupied and possessed such lot.
Respondent married the son’s daughter, and the couple made their house on the lot.
Then the son executed a transfer of rights in favor of Ebio. So respondents were
surprised when several public officers proceeded to cut 8 coconut trees planted on
such lot.

Procedural History:

Respondents went to the RTC of Paranaque City and applied for a writ of preliminary
injunction against petitioners. The RTC issued an Order denying the petition for lack
of merit. Respondents then elevated the matter to the CA. The CA reversed the RTC
Decision.

Statement of Issue(s):

Whether or not the State is an indispensable party to respondents’ action for


prohibitory injunction.

Holding:

The State is neither a necessary nor indispensable party to an action where no


positive act shall be required from it or where no obligation shall be imposed. The
law that governs over the accreted portion is Article 84 of the Spanish Law of Waters
of 1866 and Article 457 of the New Civil Code which provide that alluvial deposits
along banks of a creek do not form part of the public domain as the alluvial property
automatically belongs to the owner of the estate to which it may have been added.
Moreover, the action for prohibition seeks to enjoin the city government of
Paranaque from proceeding with its implementation of the road construction project.

13. Bulawan v. Aquende


G.R. No. 182819
Facts of the Case:

Bulawan alleges that she is the owner of a certain lot, and that she had bought the
property from the Yaptengco brothers, who in turn claim that they had inherited the
property from Yap Chin Chun. Yap claimed ownership also over the same property.
Yap says that in another civil case, the trial court had already declared the
subdivision survey of the Yaptengco brothers simulated. This means that Yap Chin
Chun is still the rightful owner. The trial court ruled for Bulawan, This was then
affirmed by the Court of Appeals. When the decision became final and executory, the
trial court issued a writ of execution. This writ was then questioned, as Aquende
claims that he was unaware of any litigation on his property

Procedural History:

Aquende filed a Notice of Appearance, praying for partial annulment of the decision,
which the trial court denied. Thereafter, Aquende filed a petition for annulment of
judgment before the Court of Appeals on the grounds of extrinsic fraud and lack of
jurisdiction, adding that he is an indispensable party and the trial court did not
acquire jurisdiction over his person because he was not impleaded as a party in the
case. The Court of Appeals ruled in favor of Aquende. Hence, this petition for review
on certiorari.

Statement of Issue(s):

Whether or not respondent is an indispensable party.

Holding:

Section 7, Rule 3 of the Rules of Court states that an indispensable party is a party
in interest without whom no final determination can be had of an action. The absence
of an indispensable party renders all subsequent actions of the court null and void for
want of authority. In this case, during the proceedings before the trial court, the
answers of Yap and Register of Deeds should have prompted the court to inquire
further whether there were other indispensable parties. It would be the height of
inequity to allow respondent’s title to be nullified without being given the opportunity
to present any evidence in support of his ostensible ownership of property.

14. National Power Corporation v. Provincial Government of Bataan


G.R. No. 180654
Facts of the Case:

National Power Corporation received a notice of franchise tax delinquency from the
provincial government of Bataan. Respondents then sent again notices of tax due.
The NPC then replied that it had ceased to be liable for payment of such tax ever
since the enactment of EPIRA. Ignoring NPC’s view, a warrant of levy was issued
on 14 real properties causing their sale at public auction.

Procedural History:

Petitioner filed with the Regional Trial Court of Mariveles, alleging that the
foreclosure had no legal basis. The RTC dismissed the petition. Petitioner appealed
to the CA but respondent moved to dismiss on ground of lack of jurisdiction over
subject matter, arguing that appeal should be lodged with the Court of Tax Appeals.
CA granted respondent’s motion and dismissed the petition.

Statement of Issue(s):

Whether or not NPC still owned or operated the business subject to local franchise
tax.

Holding:

It was held that saying that NPC was liable for local franchise tax is a nullity. EPIRA
created the National Transmission Corporation (TRANSCO) and transferred to it
NPC’s electrical transmission function. The NPC therefore ceased to operate that
business in Bataan by operation of law. Since local franchise tax is imposed on the
privilege of operating a franchise, it is clear that such tax is not a liability of the NPC.
Further, the EPIRA created the Power Sector Assets and Liabilities Management
Corporation (PSALM Corp) and transferred to it all of NPC’s “generation assets”. It
also transferred all existing liabilities of the NPC to PSALM Corp, presumably
including the unpaid liability for local franchise tax. Since the subject properties
belong to PSALM Corp and TRANSCO, they are certainly indispensable parties to
the case that must be necessarily included before it may properly go forward.

15. Chiang Kai Shek School v. Court of Appeals


G.R. No. 58028
Facts of the Case:

Private respondent Oh was dismissed from his teaching position in Chian Kai Shek
school, after having taught for a period of 33 years, for no reason.

Procedural History:

Respondent then demanded separation pay, benefits, and moral and exemplary
damages against petitioner. The CFI dismissed the compaint. The Court of Appeals
set aside the decsion. Petitioner then filed a review on certiorari.

Statement of Issue(s):

Whether or not a school that has not been incorporated may be sued.

Holding:

Under Rule 3, Section 1 of the Rules of Court, it states quite plainly that only natural
or juridical persons may be parties in a civil action. Act No. 2706, as amended,
imposes the obligation upon schools recognized by the government to incorporate
under the Corporation Law within 90 days from such recognition. In this case,
although the school has not been incorporated, petitioner cannot now invoke its own
non-compliance with the law to immunize it from private respondent’s complaint.

16. Gochan v. Young


G.R. No. 131889
Facts of the Case:
The daughter of Gochan, Alice, and the mother of respondents, had inherited 50
shares of stock in the corporation Gochan Realty from her father. When Alice died,
she left 50 shares to her husband, John Young Sr. The RTC of Cebu adjudicated
6/14 of these shares to her children. John Sr requested Gochan to partition the
shares of his late wife. This would be done by canceling the stock certificates in his
name. Then, new certificates would be issued in the name of respondents. Gochan
Realty refused, citing the right of first refusal given to the remaining stockholders.

Procedural History:

Respondents filed a complaint with the SEC for issuance of shares of stock to the
rightful owners. Petitioners moved to dismiss the complaint, alleging lack of
jurisdiction over subject matter, that respondents were not the real parties in interest,
and that the cause of action was barred by the Statute of Limitations. SEC granted
the motion to dismiss. Respondent then filed a Petition for Review with the Court of
Appeals. The CA ruled that Respondents Spouses Uy had the capacity to sue and
that the Intestate Estate of John Young Sr. was an indispensable party.

Statement of Issue(s):

1. Whether or not Spouses Uy have the personality to file suit before the SEC.
2. Whether or not Spouses Uy could bring a derivative suit in the name of
Gochan Realty.
3. Whether or not the intestate estate of Young Sr. is an indispensable party in
the case.

Holding:

1. Uy’s averment in the complaint - that the purchase of her stocks by the
corporation was null and void - is deemed admitted. Cecilia then remains a
stockholder of the corporation in view of the nullity of the Contract of Sale.
Although she was no longer registered as a stockholder in the records as of
filing of case, the admitted allegations in the complaint made her still a bona
fide stockholder of Gochan Realty as between the parties.
2. In previous jurisprudence, the Court has recognized the right of a single
stockholder to file derivative suits to bring about a redress of the wrong done
directly to the corporation and indirectly to the stockholders. In this case, the
complaint alleges all the components of a derivative suit. The personal injury
suffered cannot disqualify them from filing a derivative suit.
3. Section 3, Rule 3 of the Rules of Court only permit an executor or
administrator to represent or bring suits on behalf of the deceased, but this
does not prohibit the heirs from representing the deceased. Since the shares
are still in Young Sr.’s name, no final determination can be had without his
estate being impleaded in the suit. His estate is thus an indispensable party
with respect to the cause of action dealing with the registration of shares in
the names of the heirs of Alice. Regardless, the case can no longer be
remanded to SEC. RA 8799 transferred SEC’s jurisdiction over cases
involving intra-corporate dispute to courts of general jurisdiction or the
regional trial courts.
17. Judge Sumaljag v. Spouses Literato
G.R. No. 149787

Facts of the Case:


A complaint for the nullity of deed of sale of real property was executed between
Josefa and spouses Diosdidit and Literato. It was alleged that this deed of sale was
spurious. Josefa was the sister of Menendez, and they were two of six heirs who
inherited the property in equal parts. Respondents filed an amended answer with
counterclaim, impleading Petitioner Sumaljag with Josefa on the allegation that
petitioner, at the instance of Josefa, occupied the lots without respondents’ authority.

Procedural History:

Menendez then filed with the RTC. After Josefa’s death, Atty. Puray - petitioner’s and
Josefa’s common counsel - filed a motion for substitution, praying that the deceased
be substituted by petitioner. The RTC denied and instead ordered the appearance of
her full-blood sister Michaeles. Petitioner went to the CA on a petition for certiorari to
question the interlocutory order, which was dismissed for lack of merit.

Statement of Issue(s):

Whether or not the deceased may be substituted by petitioner.

Holding:

In this case, although counsel for Josefa did in fact notify the court of the fact of her
death, he was not able to properly give the court the name and address of the legal
representatives. The person, petitioner herein, that counsel gave as substitute was
not one of those mentioned under Section 16 of Rule 3. What does section 16, Rule
3, state? It states that it is the duty of counsel to inform the court within 30 days after
the death of his client of the fact of death, and to give the name and address of the
deceased’s legal representative or representatives. The “legal representatives” that
the provision speaks of refer to those authorized by law - administrator, executor or
guardian - who, under the rule on settlement of estate, is constituted to take over the
estate of the deceased. Section 16 of the same Rule also provides that the heirs of
the deceased may be allowed to be substituted, without requiring appointment of an
executor or administrator. Thus, counsel could have validly manifested to the court
the transfer of Josefa’s interests in the subject matter of litigation pursuant to Section
19, Rule 3. But this can happen only while the client-transferor was alive and while
the manifesting counsel was still the effective and authorized counsel for the client-
transferor, not after the death of the client when the lawyer-client relationship has
terminated. Therefore, in applying the Rule, her heirs are her surviving sisters and
the children of her deceased sister, who should be her legal representatives.

18. O. Ventanilla Enterprises vs. Tan,


GR No. 180325, 20 February 2013

Facts of the case:


Alfredo S Tan and private responded Adelina Tan were leased to properties by petitioner.
The properties in question were in Cabanatuan City. The Tans, however, did not follow the
terms of the lease. Petitioner then filed a complaint against the Tans for termination of
contract. A case was then filed in the RTC

Procedural history:

Petitioner filed a complaint against the Tans with the RTC. Private respondent appealed.
Petitioner then filed a motion for execution pending appeal which was granted by the trial
court. The trial court issued Orders lifting and cancelling the Notice of Levy on private
respondent’s properties and also several bank accounts. Petition for Review on Certiorari
under Rule 45 of the Rules of Court, was the filed to the Court of Appeals which ruled that
there was no extraordinary judgement in the case that would merit a recall of the entry of
judgement to reopen the case.

Issue:
Whether or not the the Court of Appeals was correct.

Holding:

The CA was correct in ruling that there is no extraordinary circumstance in this case that
would merit a recall of the entry of judgment to reopen the case. The reason given by
petitioner, that its former counsel had died before the CA Decision was promulgated, hence,
it was not properly notified of the judgment, is too tenuous to be given serious consideration.
In Mojar, et al. v. Agro Commercial Security Service Agency, Inc.,[17] the Court explained
that it is the party's duty to inform the court of its counsel's demise, and failure to apprise the
court of such fact shall be considered negligence on the part of said party.

19. Sarsaba vs. Fe vda. De Te,


GR No. 175910. 30 July 2009

Facts of the case: Respondent represented by her attorney-in-fact, Faustino Castañeda,


filed with the RTC, a Complaint for recovery of motor vehicle, damages with prayerfor the
delivery of the truck pendente lite against petitioner, Sereno,Lavarez and the NLRC of
Davao City. Respondent alleged, among others, that: (1) she is the wife of the late Pedro Te,
the registered owner of the truck, as evidenced by the Official Receipt and Certificate of
Registration. Petitioner alleges that that there was no showing that the heirs have filed an
intestate estate proceedings of the estate of Pedro Te, or that respondent was duly
authorized by her co-heirs to file the case; and that the truck was already sold to Gasing on
March 11, 1986 by one Jesus Matias, who bought the same from the Spouses Te. Corollarily,
Gasing was already the lawful owner of the truck when it was levied on execution and, later
on, sold at public auction. On October 17, 2005, petitioner filed an Omnibus Motion to
Dismiss the Case on the following grounds: (1) lack of jurisdiction over one of the principal
defendants; and (2) to discharge respondent's attorney-in-fact for lack of legal personality to
sue. It appeared that the respondent, Fe Vda. de Te, died on April 12, 2005. Respondent,
through her lawyer, argues that respondent's death did not render functus officio her right to
sue since her attorney-in-fact, Faustino Castañeda, hadlong testified on the complaint on
March 13, 1998 for and on her behalf and,accordingly, submitted documentary exhibits in
support of the complaint.

Issue: What is the legal effect of death of the plaintiff during the pendency of the case

Procedural History: Petition for review on certiorari with prayer for preliminary injunction
assailing the Order dated March 22, 2006 of the Regional Trial Court, Branch 19, Digos City,
Davao Del Sur, in Civil Case No. 3488

Holding: When a party to a pending action dies and the claim is not extinguished, the Rules
of Court require a substitution of the deceased. In such cases, a counsel is obliged to inform
the court of the death of his client and give the name and address of the latter's legal
representative. The rule on substitution of parties is governed by Section 16, 46 Rule 3 of
the 1997Rules of Civil Procedure, as amended. The rule on substitution by heirs is not a
matter of jurisdiction, but a requirement of due process. The rule on substitution was crafted
to protect every party's right to due process. It was designed to ensure that the deceased
party would continue to be properly represented in the suit through his heirs or the duly
appointed legal representative of his estate. It is only when there is a denial of due process,
as when the deceased is not represented by any legal representative or heir, that the court
nullifies the trial proceedings and the resulting judgment therein. In the case before Us, it
appears that respondent's counsel did not make any manifestation before the RTC as to her
death. In fact, he had actively participated in the proceedings. Neither had he shown any
proof that he had been retained by respondent's legal representative or any one who
succeeded her.

20. Dagdag vs Tongnawa


GR. No. 161166-67, 3 February 2005.

Facts of the case:


Respondents were sent a memorandum by Dagdag, the mayor of the municipality of
Tanudan, asking them why they should not be administratively santioned for acts
unbecoming of public servants and failure to perform their duties. They were given 72 hours.
Michael Tongnawa and Antonio Gammod, respondents, are the municipal engineer and
municipal planning and development coordinator.The Municipal Grievance Committee found
respondents liable for insubordination, non-performance of duties and absences without
official leaves. Respondents were then suspended for two months.

Procedural History:Respondents appealed to CSC. CSC affirmed the order of the petitioner.
Respondents filed an MR but was denied. Respondents filed a Petition for Review in the CA
which was granted, reversing the CSC ruling. Petitioner filed an MR but was denied.

Issue:
Who may appeal from the Decision of the Court of Appeals?

Holding:
The CSC and the mayor of Tanudan are real parties in interest in this case and, therefore,
can contest the decision of the CA.
The established rule is that a real party in interest is one who would be benefited or injured
by the judgment, or one entitled to the avails of the suit. The word "interest," as
contemplated by the Rules, means material interest or an interest in issue and to be affected
by the judgment, as distinguished from mere interest in the question involved or a mere
incidental interest. Stated differently, the rule refers to a real or present substantial interest
as distinguished from a mere expectancy, or a future, contingent, subordinate, or
consequential interest. As a general rule, one who has no right or interest to protect cannot
invoke the jurisdiction of the court as party-plaintiff in an action.

21. Torres vs. Rodellas,


GR No. 177836, 4 September 2009

Facts of the case:


Respondent left her family and went to Saudi Arabia. Edwino moved to Respondent’s house
in Occidental Mondoro. Edwino occupied the portion vacated by respondent’s sister. He
claims that respondent sold him the house, and he has an affidavit of relinquishment/sale of
right. Edwino also filed an MSA with the DENR in his own name for the property.
Respondent’s MSA application was turned down for Edwino. Respondent’s son returned. He
learned of Edwino’s claim and filed with the community environment and natural resources
office (CENRO) against Edwino’s MSA on the ground that the affidavit of relinquishment was
forged as respondent was abroad

Procedural History: The CA issued a resolution dismissing the petition assailing the
decision of the Office of the President. A petition for review on Certiorari under Rule 45 was
then filed.

Issue:
Whether or not the counsel of Edwino has the personality to file Motion for Reconsideration

Holding:
Yes. Because pf A.O No. 18, s.1987, Rules of Court apply in a suppletory character
whenever practicable. Sec. 16, Rule 3, Rules of Court, is applicable here. Substitution of a
deceased party by heirs is allowed in actions that survive the death of a party thereto,
particularly, causes of action that affect primarily and princippally property and properety
rights, injuries to the person being merely incidental. In this case, the action outlives the
death of Edwino as this case involves the parties’s respective rights to acquire property
which is the MSA grant. While it is not mentioned when Edwino’s exact date of death is in
Motion for Reconsideration filed before the Office of the President, considering counsel told
Office of the President of Edwino’s death in a delayed manner, sec. 16, Rule 3 merely
provides for disciplinary action against counsel in case of such failure to comply with its duty.
In said section, one can find no mention that counsel would be without legal personality to
appear for the benefit of the client or its heirs.

22. Re: Query of Mr. Roger C. Prioreschi Re Exemption from legal and filing fees of
the Good Shepherd Foundation, Inc., AM No. 09-6-9-SC 19 August 2009

Facts of the case:


Good Shepherd Foundation is an organization which works for and with indigent persons, to
quote “the poorest among the poor, to the newly born and abandoned babies, to children
who never saw the smile of their mother, to old people who cannot afford a few pesos to pay
for common prescriptions, to broken families who returned to a normal life. The administrator
of this asked the Supreme Court if he could be exempt from the payment of legal fees.

Procedural History:
The administrator of Good Shepherd foundation wrote a letter to the Supreme Court

Issue: Can the Courts grant to Good Shepherd who works for indigent and underprivileged
people, from payment of legal fees?

Holding:
Article III, Sec 11 of the 1987 Constitution is clear in it’s intent , where the bases or the
exemption from legal and filing fees is the free access clause. The clear intent of the
language of the provisions of the Rules of Court show that only a natural party litigant is
indigent. The Good Shepherd Foundation, not being a natural litigant, cannot be exempt.

23. Marcos-Araneta v. Court of Appeals,


GR No. 154096, 22 August 2008

Facts of the case:


The Benedicto group refused to reconvey 65% stockholdings of FEMII and UEC as part of
their oblidation to hold those shares and their fruits in trust and for the benefit of petitioner.
Thus, petitioner instituted two complaints which were later consolidated. Respondent and his
daughter tried to have the cases dismissed, which the RTC did. Petitioner then filed an
amended complaint. Julita Benedicto took over after her husband died.

Procedural history
The RTC entertained the Amended Complaint. Julita and Francisca moved to dismiss the
amended complaint, but RTC denied. Benedicto’s wife Julita and Francisca filed their
Answer to the amended complaint then they went to the CA via a petition for certiorari. The
verification portion of the joint petition and the certification on non-forum shopping bore only
Francisca’s signature. An Affidavit was executed by Julita in favor of Francisca. The CA
rendered a Decision, setting aside the assailed RTC orders and dismissing the amended
complaints. Irene filed an MR, but the same was denied. Hence, this Petition for Review.

Issue: Is an affidavit enough for forum shopping?

Holding: Regarding the certificate of non-forum shopping, the general rule is that all the
petitioners or plaintiffs in a case should sign it. However, time and again it is stressed that
the rules on forum shopping, which were designed to promote the orderly administration of
justice, do not interdict substantial compliance with its provisions under justifiable
circumstances. As has been ruled by the Court, the signature of any of the principal
petitioners or principal parties, as Francisca is in this case, would constitute a substantial
compliance with the rule on verification and certification of non-forum shopping. It cannot be
overemphasized that Francisca herself was a principal party.

24. Sps. Ang vs. Sps. Ang,

GR No. 186993, 22 August 2012

Facts of the case:

A $300,000 loan was obtained by respondents from petitioners, as well as a promissory note
executed for the latter, where respondents promised to pay petitioners with interest of 10%
per year upon demand. But respondents failed to pay. The loan already amounted to
720,000 plus interest, petitioners, residents of LA, executed an SPA to Atty Aceron to file an
action in court against respondents.

Procedural history:

Case was filed in the RTC QC. Dismissal was moved by the respondent on the grounds of
improper venue since the complaint can only be filed in the court of the place where either
they or the petitioners reside, and respondent resides in Bacolod while petitioner resides in
LA.

Issue: Was the dismissas of the complaint on the ground that venue was proper?

Holding:

The rules on the venue of personal actions are fixed for the convenience of the plaintiffs and
their witnesses. If one were to invoke Section 3, Rule 3 of the Rules of Court, one would
insist that Atty. Aceron, being their attorney-in-fact, is deemed a real party in interest in the
case below and can prosecute the same before the RTC. Such being the case, the
petitioners assert, the said complaint for collection of sum of money may be filed in the court
of the place where Atty. Aceron resides, which is the RTC of Quezon City.

25. Universal Robina Corp. vs Lim, GR No. 154338, 5 October 2007

Facts of the case:


There was a contract of sale between Universal Robina Corporation and Albert Lim.
Petitioner sold to Lim grocery products amounting to 808k. After tendering partial payments,
Lim did not settle his obligation despite demands.

Procedural history:
Petitioner filed with the RTC of Quezon City a complaint.RTC dismissed the complaint motu
proprio on the grounds of lack of jurisdiction and improper venue because Universal
Robina’s principal office is in Pasig City and Albert Lim is from Laoag City. Petitioner then
filed a motion for reconsideration. RTC granted the motion and admitted petitioner’s
amended complaint. On December 6, 1999, summons was served upon respondent. For his
failure to file an answer seasonably and upon motion of petitioner, the trial court issued an
Order declaring him in default and allowing petitioner to present its evidence ex parte.

Issue: WON the trial court may dismiss motu proprio petitioner’s complaint on the ground of
improper venue?

Holding:
A court may not dismiss an action motu proprio on the ground of improper venue as it is not
one of the grounds wherein the court may dismiss an action motu pro prio on the basis of the
pleadings. Sec. 2. Rule 4 of the Rules of Court provide that in venue of personal actions. All
other actions may be commenced and tried where the plaintiff or any of the principal
plaintiffs resides, or where the defendant or any of the principal defendants resides, or in the
case of a non resident defendant where he may be found, at the election of the plaintiff. Sec.
4 Rule 4 of the same Rules provide that this Rule shall not apply. Where the parties have
validly agreed in writing before the filing of the action on the exclusive venue thereof. In
personal actions, the plaintiff may commence an action either in the place of his or her
residence or the place where the defendant resides. The parties may agree to a specific
venue which could be in a place where neither of them resides. In addition, it is stated under
Sec. 1, Rule 9 of the Rules of court that improper venue not impleaded in the motion to
dismiss or in the answer is deemed waived.The court may only dismiss an action motu
proprio in case of lack of jurisdiction over the subject matter, litis pendentia, res judicata and
prescription.

26. ANICETO G. SALUDO JR. V. AMERICAN EXPRESS INTL INC, GR. No. 159507, 19
April 2006

Facts of the case:

AMEX is a corporation doing business in The Philippines which engages in providing credit
and other credit facilities. It is situated in Makati. Plaintiff is a member of the House of
Representatives who lived in Leyte. Petitioners credit card from that company was
dishonored as well as his daughter. He was then charged with late payment, and his cards
were cancelled. Petitioner claimed moral damages as a result of AMEX’s act which was
committed in gross and evident bad faith. In their answer, respondents specifically denied
the allegations in the complaint. Further, they raised the affirmative defenses of lack of
cause of action and improper venue. On the latter, respondents averred that the complaint
should be dismissed on the ground that venue was improperly laid because none of the
parties was a resident of Leyte

Procedural history

CA held that not one of the parties was a resident of Southern Leyte, hence venue was
improperly laid.

Issue: WON venue was improperly laid

Holding: The rule on venue, like other procedural rules, is designed to insure a just and
orderly administration of justice, or the impartial and evenhanded determination of every
action and proceeding., so while petitioner’s complaint for damages against respondents
before the court is a personal action governed by Section 2, Rule 4 of the Rules of Courts,
the choice of venue for personal actions cognizable by the RTC is given to plaintiff but not to
plaintiff's whim because the matter is regulated by the Rules of Court.

27. SAN LUIS VS. SAN LUIS, GR. No. 133743, 6 February 2007

Facts of the case:

San Luis contracted three marriages. The first marriage, there were six children, the second,
one son, the third he had no son. Respondent wanted a dissolution of their conjugal
partnership assets and filed a petition for letters of administration before the RTC. The heirs
from the first wife filed a motion to dismiss, but the trial court denied. Stating at the time of
San Luis’ death, since he was the governor of Laguna, it should have been filed there.

Procedural History:

Consolidated petitions for review assailing the February 4, 1998 Decision of the CA which
reversed and set aside the September 12, 1995 and January 31, 1996 Resolutions of the
Regional Trial Court of Makati City, Branch 134 in SP. Proc. No. M-3708, and its May 15,
1998 Resolution denying petitioners motion for reconsideration.

Issue:

Was venue was properly laid?

Holding: The petition for letters of administration of the estate of Felicisimo should be filed
in the Regional Trial Court of the province "in which he resides at the time of his death.
There is a distinction between "residence" for purposes of election laws and "residence" for
purposes of fixing the venue of actions. This is in accord with Section 1, Rule 73 of the
Rules of Court. In election cases, "residence" and "domicile" are treated as synonymous
terms, that is, the fixed permanent residence to which when absent, one has the intention of
returning. However, for purposes of fixing venue under the Rules of Court, the "residence" of
a person is his personal, actual or physical habitation, or actual residence or place of abode,
which may not necessarily be his legal residence or domicile provided he resides therein
with continuity and consistency.

28. United overseas bank vs Rosemoor Mining, GR No. 159669, 12 March 2007

Facts of the case:


A mortgage agreement was entered between petitioner and respondent, wherein respondent
mortgaged 6 parcels of land in Nueva Ecija. Extrajudicial foreclosure of the properties was
made, and petitioner was the highest bidder.

Procedural history:

Rosemoor’s filed separate complaints against the Bank, one before the Regional Trial Court
of Manila and the other before the Regional Trial Court of Malolos, Bulacan. Manila case -
Rosemoor and Dr. Pascual filed a Complaint, originally captioned as one for Damages,
Accounting and Release of Balance of Loan and Machinery and for Injunction" before the
Manila RTC. The Bank moved for the dismissal of the original and amended complaints on
the ground that the venue had been improperly laid. The motion was denied. The Bank filed
another motion to dismiss the Second Amended Complaint on the ground of forum-shopping.
They say that as between the action before the Manila RTC and the petition before the
Malolos RTC, there is identity of parties, rights asserted, and reliefs prayed for, the relief
being founded on the same set of facts, but to no avail. The Malolos RTC denied the motion
to dismiss. . Despite receipt of the Order, the Bank opted not to file its answer. It filed
instead a motion for reconsideration. Meanwhile, respondents moved to declare the Bank in
default. Thereafter, Malolos RTC issued an order denying the Bank’s motion for
reconsideration for lack of merit and at the same time declaring the Bank in default for failure
to file its answer. The decision in the Malolos case was also appealed to the Court of
Appeals.

Issue: Whether or not Rosemoor committed forum-shopping in filing the two cases against
the bank

Holding: Section 1, Rule 4 of the 1997 Rules of Civil Procedure, provides that actions
affecting title to or possession of real property, or interest therein, shall be commenced and
tried in the proper court which has jurisdiction over the area wherein the real property
involved, or a portion thereof, is situated.The venue of the action for the nullification of the
foreclosure sale is properly laid with the Malolos RTC. In the present case, there is only one
proceeding sought to be nullified and that is the extra-judicial mortgage foreclosure sale. And
there is only one initial transaction which served as the basis of the foreclosure sale and that
is the mortgage contract.

29. LUCAS VS. FABROS, AM No. MTJ-99-1266, 31 January 2000

Facts of the case:


Elenita did not appear the Preliminary Conference, and so Judge Fabros dismissed the case.
However, Elenita filed a motion for reconsideration and such was granted. Lucas, who was
the respondent in that case, filed another case against the judge for gross ignorance, stating
Section 19 of the Rules of Summary Procedure, as a motion for reconsideration is prohibited.
Respondent explained, however, that it was granted in the interest of justice.

Procedural history:
The complaint was referred to the Office of the Court Administrator for evaluation, report and
recommendation. A

Issue: WON the motion for reconsideration filed by Elenita is a prohibited pleading as
contemplated under the Revised Rules on Summary Proceeding?

Holding: The order of dismissal issued by respondent judge due to failure of a party to
appear during the preliminary conference is obviously not a judgment on the merits after trial
of the case. As a rule, a motion for reconsideration is a prohibited pleading under the Rule
on Summary Procedure. This rule, however, applies where the judgment sought to be
reconsidered is one rendered on the merits. As held by the Court in an earlier case involving
Sec. 19 (c) of the Revised Rules on Summary Procedure: "The motion prohibited by this
Section is that which seeks reconsideration of the judgment rendered by the court after trial
on the merits of the case."

30. BONGATO VS MALVAR, GR No. 141614, 14 August 2002

Facts of the case:


Petitioner was accused of unlawfully entering a parcel of land belonging to the spouses.
Petitioner then filed a motion for extension of time, which was denied by the MTCC, as it was
proscribed under the Rule on Summary Procedure. Petitioner filed an answer (with a new
counsel) which the MTCC disregarded, as it was filed beyond the ten-day reglementary
period. Petitioner filed a motion to dismiss with another counsel which the MTCC denied.
Thereafter, the MTCC rendered a decision ordering petitioner to vacate the land in question,
and to pay rentals, attorneys fees, and the costs of the suit.

Procedural history
RTC judge affirmed MMTC. Petitioner filed a motion for reconsideration. CA held that the lot
referred to in the present controversy was different from that involved in the anti-squatting
case. It further ruled that the MTCC had jurisdiction, and that it did not err in rejecting
petitioners Motion to Dismiss.

Issue:
WON the Court of Appeals gravely abused its discretion in ruling that the Motion to Dismiss
was a prohibited pleading

Holding: Pursuant to Section 36 of BP 129, the Court on June 16, 1983, promulgated the
Rule on Summary Procedure in Special Cases, a motion to dismiss or quash is a prohibited
pleading. Under the 1991 Revised Rule on Summary Procedure, however, a motion to
dismiss on the ground of lack of jurisdiction over the subject matter is an exception to the
rule on prohibited pleadings.

31. PASCUAL VS JOVELLANOS, AM No. MTJ-02-1429, 4 October 2002

Facts of the case:


Lorenzo Manaois was accused of forcible entry, but the complaint was dismissed as it was
without prejudice for being insufficient in some material allegations. Petitioner then corrected
this, but instead of answer. Manaois filed a motion to strike out as he claims the complaint is
false. No answer was submitted, and the motion to strike out was granted. Complainant
accused judge of neglect of duty.

Procedural history:

In his Comment, respondent denied the allegations in the Complaint. He accused Atty.
Peregrino, complainant’s counsel in the forcible entry case, of having a penchant for filing
administrative cases against him instead of appealing decisions before the proper court.
After investigation of this case, the OCA found that respondent failed to apply the Rule on
Summary Procedure, which he ought to have been very conversant with, because it was a
common procedure in municipal courts.

Issue: WON Respondent failed to apply the Rules on Summary Procedure?

Holding: Verily, judgment should have been rendered based on the allegations of the
Complaint and the evidence presented therein, inasmuch as the defendant failed to file his
answer after the lapse of ten (10) days from the service of the summons. Section 6 of the
Rule allows the trial court to render judgment, even motu proprio, upon failure of the
defendant to file an answer within the reglementary period. Moreover, under Section 10 of
the Rule, respondent was duty-bound to render his decision within thirty (30) days from
receipt of the last affidavits and position papers, or the expiration of the period for filing them.
This notwithstanding, he has not yet ruled on the Motion for Summary Judgment dated
December 15, 1999, filed in accordance with Section 6 of the Rule on Summary Procedure.

32. Victoria vs Brewmaster international, GR. No 182779, 23 August 2010

Facts of the case:


Filed for a complaint for sum of money against petitioner, Respondent is a marketing
company who sells beer. Accordingly, petitioner got a credit of 182. 502 pesos. As proof,
there are sales invoices. It says that despite demands, they did not pay. Petitioner responds,
denying any knowledge of the obligation.

Procedural history
Petitioner did not appear in the scheduled prelim conf. MeTC dismissed the complaint,
ratiocinating that respondent, as plaintiff, failed to meet the burden of proof required to
establish its claim by preponderance of evidence. Respondent elevated the case to the RTC
through a notice of appeal. RTC affirmed the decision of the lower court. Respondent then
went to the CA through a petition for review. There, it succeeded in obtaining a judgment in
its favor.

Issue: WON the CA erred in its decision

Holding: The basic requirement under the rules of procedure is that a complaint must make
a plain, concise, and direct statement of the ultimate facts on which the plaintiff relies for his
claim. Ultimate facts: substantial facts which either directly form the basis of the plaintiff's
primary right and duty or directly make up the wrongful acts or omissions of the defendant.
They refer to the principal, determinative, constitutive facts upon the existence of which the
cause of action rests. Thus, the CA erred.

33. MONGAO VS PRYCE, GR No. 156474, 16 August 2005

Facts of the case:


Respondent agreed to sell to the latter for the price of 5 million a parcel of land in Cotabato.
Respondent paid the 550,000 earnest money, but after considerable delay, offered to pay
the balance, but such was rejected. Respondent did not pay the check to petitioner Mongao
despite written and oral demands.

Procedural history:

Respondent refuted petitioners' allegations in the answer, said earnest money was allegedly
part of the amount directly paid by respondent corporation to the DBP in order to redeem
certain properties of the Animas family. Petitioners moved for judgment on the pleadings on
the ground that the answer admitted the material allegations of the complaint and, therefore,
failed to tender an issue.. RTC granted the motion while CA reversed and remanded the
case for trial on the merits.

Issue: WON judgment on the pleadings was proper?

Holding:
Sec. 1, Rule 19 of the ROC provides that where an answer fails to tender an issue, or
otherwise admits the material allegations of the adverse party's pleading, the court may, on
motion of that party, direct judgment on such pleading. Respondent’s answer admitted that
there was a perfected contract of sale between it and petitioner and that respondent refused
to tender payment of the purchase price solely to petitioner. Now, if an answer does in fact
specifically deny the material averments of the complaint in the manner indicated by said
Section 10 of Rule 8, and/or asserts affirmative defenses in accordance with Sections 4 and
5 of Rule 6, a judgment on the pleadings would naturally not be proper.

34. BUNGCAYAO v. FORT ILOCANDIA PROPERTY HOLDINGS, GR. No. 170483, 19


April 2010

Facts of the case:


Petitioner is one of two entrepeneurs who added improvements on the foreshore area of a
beach where a hotel was constructed. The other entrepeneurs began setting up their own
stalls in the area, and formed a Beach Resport Owner’s Association. Six parcels of land in
the beach were transferred to the Philippine Tourism Authority. Petitioner and other D’Sietro
members then applied for a foreshore lease with the CENRO and was granted a provisional
permit. DENR then denied the lease applications.

Procedural history:
The D'Sierto members appealed the denial of their applications. The then DENR Secretary
denied the appeal. Petitioner then filed an action for declaration of nullity of contract before
the RTC of Laoag, City against respondent. Petitioner alleged that his son had no authority
to represent him and that the deed was void and not binding upon him. The trial court
confirmed the agreement of the parties to cancel the Deed of Assignment, Release, Waiver
and Quitclaim and the return of P400,000 to respondent. Petitioner's counsel, however,
manifested that petitioner was still maintaining its claim for damages against respondent.
Petitioner and respondent agreed to consider the case submitted for resolution on summary
judgment. Thus, the trial court considered the case submitted for resolution. Petitioner filed a
motion for reconsideration, alleging that he manifested in open court that he was
withdrawing his earlier manifestation submitting the case for resolution. Respondent filed a
Motion for Summary Judgment

ISSUE: WON respondent’s counterclaim was compulsory

Holding: The only claim that remained was the claim for damages against respondent. A
compulsory counterclaim is any claim for money or any relief, which a defending party may
have against an opposing party, which at the time of suit arises out of, or is necessarily
connected with, the same transaction or occurrence that is the subject matter of the plaintiff's
complaint. It is compulsory in the sense that it is within the jurisdiction of the court, does not
require for its adjudication the presence of third parties over whom the court cannot acquire
jurisdiction, and will be barred in the future if not set up in the answer to the complaint in the
same case. Any other counterclaim is permissive.

35. Philtranco Service Enterprises v. CA, GR No. 161909, 25 April 2012.

Facts of the Case:


On Felix Paras way home to Manila from Bicol, he boarded a vehicle owned and operated by
Inland Trailways inc. The bus was bumped by another vehicle owned and opearated by
Philtranco. The said accident brought damage to the vehicles, and caused physiacal injuries to
the passengers. Paras was taken to the hospital, where he was diagnosed with several injuries
such as hematoma. PAras went to two operations affecting the fractured portions of his body.

Procedural History:
Inland denied responsibility, by alleging that its driver Coner had observed an utmost and
extraordinary care and diligence to ensure the safety of its passengers. Inland invoked the
Police Investigation Report that the Philtranco bus driver of Apolinar Miralles was which
violently bumped the rear portion of the Inland bus, and therefore, the direct and proximate
cause of Paras' injuries. Upon leave of court, Inland filed a third-party complaint against
Philtranco and Apolinar Miralles. RTC rendered decision that the third-party defendant
Philtranco and Apolinar Miralles are hereby ordered to pay plaintiff jointly and severally. All the
parties appealed to the CA on different grounds.

Statement of the Issue:


Whether or not Paras can recover damages moral damages in this suit based on quasi-delict
Ruling:
Under the Section 12 Rule 6 of Rules of Court— A third-party complaint is a claim that a
defending party may, with leave of court, file against a person not a party to the action, called
the third-party defendant, for contribution, indemnity, subrogation or any other relief, in respect
of his opponent's claim. In Balbastro v. Court of Appeals, Section 12 of Rule 6 of the Revised
Rules of Court authorizes a defendant to bring into a lawsuit any person "not a party to the
action . . . for contribution, indemnity, subrogation or any other relief in respect of his
opponent's claim." The impleader of new parties under this rule is proper only when a right to
relief exists under the applicable substantive law. The requisites for a third-party action
are, firstly, that the party to be impleaded must not yet be a party to the action; secondly, that
the claim against the third-party defendant must belong to the original defendant; thirdly, the
claim of the original defendant against the third-party defendant must be based upon the
plaintiff's claim against the original defendant; and, fourthly, the defendant is attempting to
transfer to the third-party defendant the liability asserted against him by the original plaintiff. It
is worth adding that allowing the recovery of damages by Paras based on quasi-delict, despite
his complaint being upon contractual breach, served the judicial policy of avoiding multiplicity
of suits and circuity of actions by disposing of the entire subject matter in a single litigation.

36. Sameer Overseas Placement Agency Inc, GR No. 152579, 7 August 2009

Facts of the case:

Respondent was hired under a one year employment contract with Sameer Overseas
Placement Agency Inc. She went to Taiwan to work in Wacol as quality control. There, she
was asked to work as a cutter. A certain Mr Huwang then informed her without prior notice that
she was terminated and that “she should immediately report to their office to get her salary and
passport.” She was asked to “prepare for immediate repatriation.” She earned only a total of 9k
NT because Wacoal deducted to cover her plane ticket expense back to Manila.
Procedural history :

Joy filed a complaint for illegal dismissal with the NLRC against petitioner and Wacoal. LA
dismissed the complaint. NLRC reversed LA’s decision. CA affirmed the ruling of the National
Labor Relations Commission

ISSUE: Whether or not Cabiles was entitled to the unexpired portion of her salary due to illegal
dismissal.

HELD:

The award of the three-month equivalent of respondent’s salary should be increased to the
amount equivalent to the unexpired term of the employment contract. In Serrano
v. Gallant Maritime Services, Inc. and Marlow Navigation Co., Inc., this court ruled that the
clause “or for three (3) months for every year of the unexpired term, whichever is less” is
unconstitutional for violating the equal protection clause and substantive due process. Sameer
contends that both the June 6, 2001 Petition and the July 5, 2001 Motion for Reconsideration
filed by ASBT before the Court of Appeals were signed by Mildred Santos, as corporate
president, who is not a member of the Bar. As such, Sameer argues that both the Petition and
the Motion for Reconsideration should be considered unsigned pleadings which produce no
legal effect, pursuant to the last paragraph of Section 3, Rule 7 of rules of court. Obviously, the
rule allows the pleadings to be signed by either the party to the case or the counsel
representing that party

37. Anderson v. Ho, GR No. 172590, 7 January 2013.

Facts:

An ejectment case was filed with the MeTC. This case was then dismissed. Later, it was
appealed to the RTC which also dismissed the same. Thus, petitioner filed a motion for
reconsideration. This was then denied by the RTC. Wanting to file with the CA a petitioner for
review under Rule 42 of the Rules of Court, Atty Oliva, petitioner’s counsel. Filed a motion for
extension of time of 15 days. This was granted by the CA. Later, said counsel sought another
extension of 15 days as the petition had been finalized and sent to Anderson in hawaii for her
to read as well as sign.

Procedural History:

When it was the last day of the extended period, the petition was not yet been sent back. An
additional extension was sought. CA granted the said motion for extension. Atty. Oliva was
finally able to file the Petition for Review but the certification against forum shopping attached
thereto was signed by him on Anderson’s behalf without any accompanying authority to do so.
Hence, the CA dismissed the case as the certification of forum shopping was signed not by the
petitioner herself but by her counsel without authority to do so. A motion for reconsideration
was filed which was also denied by the CA. Thus, petitioner now invokes before the SC the
liberal interpretation of the rules of procedure

Issue:
Whether or not the rules on certification against forum shopping may be relaxed in this case.

Ruling:

The need to abide by the Rules of Court and the procedural requirements it imposes has been
constantly underscored by this Court. One of these procedural requirements is the certificate of
non-forum shopping which, time and again, has been declared as basic necessary and
mandatory for procedural orderliness. In Vda. De Formoso v. Philippine National Bank the
Court reiterated the guidelines respecting non-compliance with or submission of a defective
certificate of non-forum shopping, the relevant portions of which are as follows: As to
certification against forum shopping, non-compliance therewith or a defect therein, is generally
not curable by its subsequent submission or correction thereof, unless there is a need to relax
the Rule on the ground of ‘substantial compliance or presence of ‘special circumstances or
compelling reasons’.

38. Fuentebella v. Castro, GR No. 150865, 30 June 2006.

Facts:
Respondent engaged the funeral services of Rolling Hills Memorial Park for the interment of
the remains of her dead husband, Freddie Castro. As the burial was taking place and the
casket of her deceased husband was to be lowered, it was discovered that the dimensions of
the vault did not fit with the measurements of the casket. Because of this, the casket was lifted
and placed under the heat of the sun for one hour in front of all the mourners. To her
humiliation, the employees of petitioner measured the casket by using a spade.

Procedural History:

Darlica filed a complaint for damages before the MTCC. Darlica withdrew the complaint
because amount of damages claimed exceeds PhP 200,000. Darlica filed a similar complaint
with the RTC of Negros Occidental. Petitioners filed with the Court of Appeals a petition for
certiorari. The petition, however, was dismissed by the CA. A perusal of the records discloses
that the verification and the certification against forum shopping was signed by a certain
Lourdes Pomperada without any showing or indication that she is duly authorized by the
petitioners to sign for and in their behalf

ISSUE:

Was the certification false because there was already a prior case before the MTCC

Ruling:

An omission in the certificate of non-forum shopping about any event that would not constitute
res judicata and litis pendentia, as in the present case, is not fatal as to merit the dismissal and
nullification of the entire proceedings considering that the evils sought to be prevented by the
said certificate are not present. Hence, in any event, the trial court correctly held that the
submission of a false certification shall constitute indirect contempt of court, without prejudice
to the corresponding administrative and criminal sanctions.

39. Cagayan Valley v. Commissioner of Internal Revenue, GR No. 151413, 13 February


2008.
Facts:
Petitioner says that it granted 20% sales discounts to qualified senior citizens on purchases of
medicine. This was in accordance with RA 7432. This was treated as deductions from the
gross sales in order to arrive at the net sales instead of treating them as tax credit as provided.
They then filed a claim with the BIR for tax refund/ tax credit. BIR did not act on the tax credit
so Cagayan filed a petition for review before the CTA.

Procedural History:

The CTA denied the tax credit to Cagayan notwithstanding Cagayan’s entitlement to a tax
credit. Aggrieved, Cagayan elevated the matter before the CA One of the issues raised before
the CA is the irregularity of the verification and certification of non-forum shopping as it was
signed by the president without any proof of authorization by the Board of Directors of
Cagayan.

ISSUES:

Whether Cagayan’s president can sign the subject verification and certification sans the
approval of its Board of Directors.

RULING:

The president can sign the verification and certification. The Court has recognized the authority
of some corporate officers to sign the verification and certification against forum shopping. the
following officials or employees of the company can sign the verification and certification
without need of a board resolution: (1) the Chairperson of the Board of Directors, (2) the
President of a corporation, (3) the General Manager or Acting General Manager, (4) Personnel
Officer, and (5) an Employment Specialist in a labor case. The rationale applied in the
foregoing cases is to justify the authority of corporate officers or representatives of the
corporation to sign the verification or certificate against forum shopping, being “in a position to
verify the truthfulness and correctness of the allegations in the petition
40. Chua v. Metropolitan Bank and Trust Company, GR No. 182311, 19 August 2009

Facts:
Chua was president of the company Chua and Filiden. The company got a 4 million peso loan
from metrobank, secured by a real estate mortgage. However, the value of the collateral was
more than the loan, so Chua was given an open credit line for future loans. They were unalbe
to pay their obligations, so metrobank entered into a debt settlement agreement with them,
whereby the loan was restructured. Finally, it was demanded that the company pay their
liabilities. When petitioners were unable to pay, Metrobank tried to extra-juducually forcelose
the REM constituted on subject properties.

Procedural History:

RTC issued an Order directing that the said sale be reset. The auction sale, however,
proceeded, and a Certificate of Sale was accordingly issued to respondent Metrobank as the
highest bidder of the foreclosed properties.-Chua amended the Injunction Complaint alleged
that the Certificate of Sale was a falsified document since there was no actual sale that took
place on 8 November 2001. Chua also prayed in their Amended Complaint for the award of
damages-RTC-Branch 257 denied Chua’s application for injunction on the ground that the sale
of the foreclosed properties rendered the same moot and academic.

Issue:
Whether or not successively filing Civil Case No. CV-01-0207 and Civil Case No. CV-05-0402
amounts to forum shopping.

Ruling:
Petitioners were unable to state in the Certificate of Non-Forum Shopping the existence of Civil
Case No. CV-01-0207 pending before RTC-Branch 258. If the forum shopping is not
considered willful and deliberate, the subsequent case shall be dismissed without prejudice, on
the ground of either litis pendentia or res judicata. However, if the forum shopping is willful and
deliberate, both (or all, if... there are more than two) actions shall be dismissed with
prejudice..[43] In this case, petitioners did not deliberately file Civil Case No. CV-05-0402 for
the purpose of seeking a favorable decision in another forum. Otherwise, they would not have...
moved for the consolidation of both cases. Thus, only Civil Case No. CV-05-0402 is dismissed
and the hearing of Civil Case No. CV-01-0207 before RTC-Branch 258 will be continued.

41. Metrobank v. Santos, GR No. 157867, 15 December 2009.


Facts:
A loan was obtained by respondent from Metrobank of the amoung of two million pesos. To
secure payment, a real estate mortgage over a condominium unit in Makati and all it’s
improvements were issued. When the loan was not paid despite demand, Metrobank instituted
an extrajudicial foreclosure proceedings against the real estate morgage, where Metrobank
was the highest bidder. So Metrobank registered the certificate of sale. The redemption period
lapsed without De Koning redeeming the property. Metrobank demanded that possession over
the condominium unit be turned in.
Procedural History:
The lower court issued an order setting the ex parte hearing of Metrobank's petition.
Respondent’s counsel appeared and manifested that he filed a motion to dismiss on the
ground that Metrobank's petition violated Section 5, Rule 7 of the Rules of Court which
requires the attachment of a certification against forum shopping to a complaint or other
initiatory pleading. When Metrobank failed to disclose the existence of two pending cases in
the certification attached to its petition, it failed to comply with the mandatory requirements of
the Rules so that its petition should be dismissed. The RTC agreed with respondent and
dismissed Metrobank's petition for having a false certification of non-forum shopping. The
lower court denied Metrobank's motion for reconsideration. Metrobank thus elevated the matter
to the CA on a petition for certiorari on January 5, 2001.The CA affirmed the dismissal of
Metrobank's petition. It explained that Section 5, Rule 7 of the Rules is not limited to actions,
but covers any "initiatory pleading" that asserts a claim for relief. The CA denied Metrobank's
subsequent motion for reconsideration.

Issue:
Whether or not THE COURT OF APPEALS COMMITTED A MISAPPREHENSION OF FACTS

Ruling:
Metrobank claims that an ex parte petition for the issuance of a writ of possession is not an
initiatory pleading asserting a claim. Rather, it is a mere incident in the transfer of title over the
real property which was acquired by Metrobank through an extrajudicial foreclosure sale, in
accordance with Section 7 of Act No. 3135, as amended. Thus, the petition is not covered by
Section 5, Rule 7 of the Rules and a certification against forum shopping is not required. A writ
of possession may issue either (1) within the one year redemption period, upon the filing of a
bond, or (2) after the lapse of the redemption period, without need of a bond. In order to obtain
a writ of possession, the purchaser in a foreclosure sale must file a petition, in the form of
an ex parte motion, in the registration or cadastral proceedings of the registered property. The
reason why this pleading, although denominated as a petition, is actually considered a motion
is best explained in Sps. Arquiza v. CA, where it is stated that the certification against forum
shopping is required only in a complaint or other initiatory pleading. The ex parte petition for
the issuance of a writ of possession filed by the respondent is not an initiatory pleading.

42. Vda. de Daffon v. CA, GR No. 129017, 20 August 2002.


FACTS:
Petitioner and her husband had one child, Joselito, who married respondent and they had six
children. Joselito dies after his father. Respondent, with her six children, instituted an action for
partition against petitioner over the properties left by petitioner’s husband which formed part of
his conjugal partnership.
Procedural History:
Petitioner filed a motion to dismiss, but the trial court denied the same. Petitioner's motion for
reconsideration was also denied by the trial court. Subsequently, petitioner filed a petition
for certiorari with the Court of Appeals, but the latter dismissed the same.
Issue:
Whether or not the Court of Appeals erred
Ruling:
In this case, the Court of Appeals was correct. The Supreme Court ruled that in the
determination of whether a complaint fails to state a cause of action, only the statements in the
complaint may be properly considered. The complaint needs only to allege the ultimate facts
on which the plaintiffs rely for their claim. The allegations in the complaint in the case at bar
are sufficient to establish respondent's right to the estate of Amado. By stating their
relationship to the deceased, they established their line of succession as the basis for their
claim. Their rights to succeed as heirs were transmitted from the moment of death of the
decedent. The Court likewise ruled that the trial court did not commit grave abuse of discretion
in denying petitioner's motion to dismiss, which was the subject of the petition for certiorari.

43. Titan Construction v. David, GR No. 169548, 15 March 2010.

Facts:

Spouses David acquired a lot in White Plains, Quezon City, registered under the name of
“MARTHA S. DAVID, of legal age, Filipino, married to Manuel A. David". Spouses then
separated and no longer communicated. Manuel then discovered that Martha had previously
sold the property to Titan Construction Corporation for 1m through a deed of sale in the name
of Titan. Manuel filed a Complaint for Annulment of Contract and Reconveyance against Titan
before the RTC.Manuel alleges that the sale executed by Martha in favor of Titan was without
his knowledge and consent.
Procedural History:
The RTC issued a Decision which (i) invalidated both the Deed of Sale and TCT No. 130129;
(ii) ordered Titan to reconvey the property to Martha and Manuel; (iii) directed the Register of
Deeds of Quezon City to issue a new title in the names of Manuel and Martha; and (iv) ordered
Titan to pay P200,000.00 plus P1,000.00 per appearance as attorney's fees, and P50,000.00
as costs of suit. the CA affirmed the Decision of the trial court but deleted the award of
attorney's fees and the amount of P50,000.00 as costs
Issue:
Whether or not there is a Failure to Deny the Genuineness and Due Execution of the SPA
Ruling:
While Section 8, Rule 8 is mandatory, it is a discovery procedure and must be reasonably
construed to attain its purpose, and in a way as not to effect a denial of substantial justice. The
interpretation should be one which assists the parties in obtaining a speedy, inexpensive, and
most important, a just determination of the disputed issues. It is true that the reply filed by
Manuel alleging that the special power of attorney is a forgery was not made under oath.
However, the complaint, which was verified by Manuel under oath, alleged that the sale of the
subject property executed by his wife, Martha, in favor of Titan was without his knowledge,
consent, and approval, express or implied; and that there is nothing on the face of the deed of
sale that would show that he gave his consent thereto. In Toribio v. Bidin, it was held that
where the verified complaint alleged that the plaintiff never sold, transferred or disposed their
share in the inheritance left by their mother to others, the defendants were placed on adequate
notice that they would be called upon during trial to prove the genuineness or due execution of
the disputed deed of sale.

44. Consolidated Bank v. Del Monte Motor Works, GR No. 143338, 29 July 2005.

Facts:
A complaint for recovery of sum of money against respondents was filed by petitioner, who
impleades the spouse of Morales in order to bind their conjugal partnership of gains. Petitioner
says that it extended a loan to respondents in the amount of 1 million pesos. Under the
promissory note, respondents bound themselves to pay petitioner the full amount of the loan
through twenty five monthly installments of 40,000 a month. Respondents defaulted on their
monthly installments despite demand.

Procedural History:
Petitioner filed an Ex-Parte Motion to Declare the Defendants in Default which was opposed by
the defendants upon the ground that they were never served with copies of the summons and
of petitioner's complaint. Petitioner filed a motion praying that the presiding judge, Judge
Ricardo D. Diaz, of the court a quo inhibit himself from this case. This motion to inhibit was
denied by the trial court.
Issue:
Did the Court of Appeals make a mistake
Ruling:
Rule 8 Section 8 of the Rules of Court on the matter provides:
SEC. 8. How to contest such documents. — When an action or defense is founded upon a
written instrument, copied in or attached to the corresponding pleading as provided in the
preceding section, the genuineness and due execution of the instrument shall be deemed
admitted unless the adverse party, under oath, specifically denies them and sets forth what he
claims to be the facts; but the requirement of an oath does not apply when the adverse party
does not appear to be a party to the instrument or when compliance with an order for an
inspection of the original instrument is refused. In the case of Permanent Savings and Loan
Bank v. Mariano Velarde, this Court held that Respondent also denied any liability on the
promissory note as he allegedly did not receive the amount stated therein, and the loan
documents do not express the true intention of the parties. Respondent reiterated these
allegations in his "denial under oath," stating that the "promissory note sued upon, assuming
that it exists and bears the genuine signature of herein defendant, the same does not bind him
and that it did not truly express the real intention of the parties as stated in the defenses.
There can be no dispute to the fact that the allegations in the ANSWER of both defendants,
they denied generally and specifically under oath the genuineness and due execution of the
promissory note and by way of special and affirmative defenses herein states that he
(MORALES) never signed the promissory note attached to the complain in his personal and/or
individual capacity. Moreover, what appears in the record was an admission of paragraphs 1 &
2 but they deny generally and specifically the rest of the allegations. It would be considered
that there is a sufficient compliance of the requirement of the law for specific denial.

45. Luistro v. CA, GR No. 158819, 16 April 2009.

FACTS:

First Gas Power Corporation entered into a Substation interconnection agreement with
Meralco and NPC, where First Gas would finance, construct, and commission a 230 kilovolt
electric power transmission line. Under the SIA, First Gas had to acquire easement of right of
way over certain lands. Frist Gas entered into a Contract of Easement of Right of Way with
Luistro, an owner of a parcel of land in Batangas where First Gas was granted perpetual
easement. However, Luisitro’s counsel wrote a letter to Frist Gas asking for a temporary
stoppage of all kinds of work within the vicinity of petitioner’s residential hoouse pending
settlement of petitioner’s grievance that the house and other improvements lay underneath the
transmission wire/line being constructed and would endanger the life and health of the persons
in the vicinity.

Procedural History:

RTC: Luistro filed a complaint for Rescission/Amendment And Or Modification of Contract Of


Easement According to Luistro: he entered into the Contract under misrepresentation. First
Gas filed a MOTION TO DISMISS for failure to state cause of action. RTC denied MTC. RTC
also denied MR.CA: First Gas filed petition for certiorari before the CA. CA ordered dismissal
for failure to state cause of action. CA denied the MR filed by Luistro.

Statement of the Issue:

Whether the complaint alleges fraud with particularity as required under Section 5, Rule 8 of
the 1997 Rules of Civil Procedure

Ruling:
Section 5, Rule 8 of the 1997 Rules of Civil Procedure states that “In all averments of fraud or
mistake, the circumstances constituting fraud or mistake must be stated with particularity.
Malice, intent, knowledge or other condition of the mind of a person may be averred generally.”
Again, the complaint falls short of the requirement that fraud must be stated with particularity.
The complaint merely states “That sometime in the year of 1997, the consolidator-facilitator of
the Defendants FGPC and Balfour by means of fraud and machinations of words were able to
convince[] the plaintiff to enter into ‘CONTRACT OF EASEMENT OF RIGHT OF WAY’ wherein
the latter granted in favor of the defendant FGPC the right to erect [its] Tower No. 98 on the
land of the plaintiff situated at Barangay Maigsing Dahilig, Lemery 4209 Batangas including the
right to Install Transmission Lines over a portion of the same property for a consideration
therein stated, a xerox copy of said contract is hereto attached as [] ANNEXES “A” up to “A-4”
of the complaint” AND; “That the said contract, (Annexes “A” up to “A-4”) was entered into by
the plaintiff under the “MISREPRESENTATION, PROMISES, FALSE AND FRAUDULENT
ASSURANCES AND TRICKS” of the defendants”

46.Indiana Aerospace v. Commission on Higher Education, GR No. 139371, 4 April 2001.

Facts of the case


Petitioner, a school, posted an advertisement in the Manila Bulletin. Engineering (TPRAME)
inquired whether petitioner had acquired university status. Dr Vera referred the letter to CHED,
and CHED to it’s regional director in Cebu, requesting the office to conduct an investigation. As
a consequence of said Report, respondent's Legal Affairs Service was requested to take legal
action against petitioner. Respondent directed petitioner to desist from using the term
University, including the use of the same in any of its alleged branches.
Procedural History:
Respondent filed a Special Appearance with Motion to Dismiss. The CA ruled that petitioner
had no cause of action against respondent. Petitioner failed to show any evidence that it had
been granted university status by respondent as required under existing law and CHED rules
and regulations. A certificate of incorporation under an unauthorized name does not confer
upon petitioner the right to use the word "university" in its name.
ISSUE:
Whether or not the order of default is valid
RULING:
In the case of Lina v. Court of Appeals discussed the remedies available to a defendant
declared in default, as follows: (1) a motion to set aside the order of default under Section 3(b),
Rule 9 of the Rules of Court, if the default was discovered before judgment could be rendered;
(2) a motion for new trial under Section 1(a) of Rule 37, if the default was discovered after
judgment but while appeal is still available; (3) a petition for relief under Rule 38, if judgment
has become final and executory; and (4) an appeal from the judgment under Section 1, Rule
41, even if no petition to set aside the order of default has been resorted to.

47. Monzon v. Spouses Relova, GR No. 171827, 17 April 2008.

Facts of the Case:


Spouses Relova and Perez allege that Monzon issued a promisory note in their favor worth
P600k and secured by a lot in Tagaytay city. A deed of absolute sale over the parcel of land
was later executed in favor of the Perez spouses. The same thing happened with spouses
Relova where a promisory note in the amount of 200,000 was issued and secured by a lot. A
deed of conditional sale over the parcel of land was later issued in favor of spouses Relova.
Monzon was indevted to the Coastal Lending Corporation, who extrajudically foreclosed the
proeprty of Monzon. The winning bidder in this extrajudicial forecloseure was Addio properties.
Of the amount paid by Addio, there was a residue of roughlyP1.6M (indebtedness of Monzon
was only around P3.4M while Addio paid P5M for the property that‘s why there‘s an excess).
This residue is in the custody of Atty. Luna as Branch Clerk of Court. Monzon argues that she
has already fulfilled her obligation to the spouses via dacion en pago evidenced by the Deed of
Conditional Sale and the Deed of Absolute Sale.
Procedural History: RTC: Due to Monzon and counsel’s absence on said hearing date despite
due notice, granted an oral Motion by the respondents by issuing an Order allowing the ex
parte presentation of evidence by respondents. Atty. Luna should deliver the residue to
spouses Relova and Perez. At this point in time, Addio properties intervened. The CA affirmed
RTC, denied Monzon’s appeal, the violation of due process since she was not allowed to
present in court again after not appearing in first hearing.

Issue :
Whether or not Monzon is declared in default

Ruling:
Section 3, Rule 9
“(a) Effect of order of default. — A party in default shall be entitled to notice of subsequent
proceedings but not to take part in the trial.”
Failure to file a responsive pleading within the reglementary period, and not failure to appear
at the hearing, is the sole ground for an order of default, except the failure to appear at a pre-
trial conference wherein the effects of a default on the part of the defendant are followed, that
is, the plaintiff shall be allowed to present evidence ex parte and a judgment based thereon
may be rendered against the defendant.

The RTC applied the effects of a default order upon petitioner under Section 3, Rule 9 of the
Rules of Court which states the “If the defending party fails to answer within the time allowed
therefor, the court shall, upon motion of the claiming party with notice to the defending party,
and proof of such failure, declare the defending party in default. Thereupon, the court shall
proceed to render judgment granting the claimant such relief as his pleading may warrant,
unless the court in its discretion requires the claimant to submit evidence. Such reception of
evidence may be delegated to the clerk of court.”

48. Sps. Lambino v. Presiding Judge, GR No. 169551, 24 January 2007.

Facts of the case:


Petitioner, a lawyer, and his wife secured a housing loan from private respondent BPI. The interest rate
was 19% pe annum payable in 180 monthly installments. A mortgage loan agreement was executed by
petitioners over their property as security. However, petitioners failed to pay the monthly amortizations
from January 15, 1995 to May 15, 1995. Private respondent filed a petition for the extrajudicial
foreclosure of the MLA with the Ex-Officio Sheriff of the RTC of Valenzuela City and sought to have the
property sold to satisfy the balance of petitioners' loan account. The public auction was set at 10:00 a.m.
on July 11, 1995. Petitioners filed a complaint for annulment of the MLA and the extrajudicial foreclosure
sale with a prayer for a Temporary Restraining Order (TRO) before the RTC of Valenzuela City.
Procedural History:
The court suspended pretrial to enable the parties to settle the matter amicably. the trial court issued an
Order denying the motion of petitioners in its finding that the alleged escalating and arbitrary rate of
interest and other charges imposed by private respondent had accrued long before the complaint was
filed.
Issue:
Whether the court of appeals made a mistake
Ruling:
Section 6 of Rule 10 which states that “Upon motion of a party, the court may, upon reasonable notice
and upon such terms as are just, permit him to serve a supplemental pleading setting forth transactions,
occurrences or events which have happened since the date of the pleading sought to be supplemented.
If the court deems it advisable that the adverse party should plead thereto, it shall so order, specifying
the time therefor.” The rule is a useful device which enables the court to award complete relief in one
action and to avoid the cost of delay and waste of separate action. Thus, a supplemental pleading is
meant to supply deficiencies in aid of the original pleading and not to dispense with or substitute the
latter. A supplemental complaint must be consistent with, and in aid of, the cause of action set forth in
the original complaint. A new and independent cause of action cannot be set up by such complaint. The
supplemental complaint must be based on matters arising subsequent to the original complaint related
to the claim or defense presented therein, and founded on the same cause of action. However, although
the facts occur before the commencement of the suit if a party does not learn of their existence until
after he has filed his pleading, he may file a supplemental pleading. As a general rule, leave will be
granted to file a supplemental complaint which alleges any material fact which happened or came within
plaintiff's knowledge since the original complaint was filed, such being the office of a supplemental
complaint. The purpose of the rule is that the entire controversy might be settled in one action; to avoid
unnecessary litigation; prevent delay, unnecessary repetition of effort; unwarranted expense of litigants;
to broaden the scope of the issues in an action owing to the light thrown on it by facts, events and
occurrences which have accrued after the filing of the original pleading; to bring into record the facts
enlarging or charging the kind of relief to which plaintiff is entitled.

49. Philippine Ports Authority v. Gothong, GR No. 158401, 28 January 2008.

Facts of the Case:


After the expiration of the lease contract of Veterans Shipping Corporation over the Marine Slip
Way in the North Harbor then petitioner WG&A requested respondent PPA for it to be allowed
to lease and operate the said facility. Thereafter, then President Estrada issued a
memorandum addressed to the Secretary of the Department of Transportation and
Communication (DOTC) and the General Manager of PPA.
Procedural history:
Petitioner WG&A commenced an Injunction suit before the Regional Trial Court of Manila.
Petitioner claims that the PPA unjustly, illegally and prematurely terminated the lease contract.
It likewise prayed for the issuance of a temporary restraining order to arrest the evacuation.
Petitioner filed a motion for reconsideration of the aforesaid order but the same was again
denied in an order. WG&A then filed a petition for certiorari with the CA seeking the nullification
of the aforementioned RTC orders. CA granted respondent's petition, thereby setting aside the
RTC orders and directing the RTC to admit respondent's second amended complaint pursuant
to Section 3, Rule 10 of the 1997 Rules of Civil Procedure.
Issue:
whether the CA erred in ruling that the RTC committed grave abuse of discretion when it
denied the admission of the second amended complaint.
Ruling:
The Court has emphasized the import of Section 3, Rule 10 of the 1997 Rules of Civil
Procedure in Valenzuela v. Court of Appeals, which states that the Section 3, Rule 10 of the
1997 Rules of Civil Procedure amended the former rule in such manner that the phrase "or that
the cause of action or defense is substantially altered" was stricken-off and not retained in the
new rules. The clear import of such amendment in Section 3, Rule 10 is that under the
new rules, "the amendment may (now) substantially alter the cause of action or
defense." This should only be true, however, when despite a substantial change or alteration
in the cause of action or defense, the amendments sought to be made shall serve the higher
interests of substantial justice, and prevent delay and equally promote the laudable objective of
the rules which is to secure a "just, speedy and inexpensive disposition of every action and
proceeding."

50. Wallem Philippines Shipping, Inc. v. S.R Farms, Inc., GR No. 161849, 9 July 2010

Facts of the Case:


A shipment was made of Indian Soya Bean meal to Manila. The vessel was owned by Consti-
Feed with respondent as it’s ship agent. The shipment arrived in Manila, and was discharged.
All throughout the entire period of unloading, good and dair weather condition prevailed.
However, there was a shortage of the shipment.
Procedural History:
Petitioner then filed a Complaint for damages. The complaint was dismissed at the instance of
respondent on the ground that it has no cause of action against the former. Upon motion of
RCS, the case against it was likewise dismissed for lack of cause of action. RTC rendered its
Decision dismissing respondent's complaint, as well as the opposing parties' counterclaims
and crossclaims. Respondent filed an appeal with the CA. CA reversed.
ISSUE:
Whether the filing of amended complaint against petitioner should retroact to the date of fling
of original complaint
Ruling:
The settled rule is that the filing of an amended pleading does not retroact to the date of the
filing of the original; hence, the statute of limitation runs until the submission of the
amendment. It is true that, as an exception, this Court has held that an amendment which
merely supplements and amplifies facts originally alleged in the complaint relates back to the
date of the commencement of the action and is not barred by the statute of limitations which
expired after the service of the original complaint. The exception, however, would not apply to
the party impleaded for the first time in the amended complaint. The rule on the non-
applicability of the curative and retroactive effect of an amended complaint, insofar as newly
impleaded defendants are concerned, has been established as early as in the case of Aetna
Insurance Co. v. Luzon Stevedoring Corporation. In the said case, the defendant Barber Lines
Far East Service was impleaded for the first time in the amended complaint which was filed
after the one-year period of prescription. The order of the lower court dismissing the amended
complaint against the said defendant on ground of prescription was affirmed by this Court.

51. Spouses Dionisio v. Linsangan, GR No. 178159, 2 March 2011.


Facts of the case:
Gorgonio Cruz owned agricultural lands in San Rafael, Bulacan. The tenant, Romualdo,
cultivated the lands. When Romualdo died, his widow was given permission by Cruz to
stay on the property provided that she would vacate it upon demand. Petitioners bought
property from Cruz. They trued to get Romualdo’s widow out.
Procedural history:
The MTC rendered judgment, ordering Wilfredo to vacate the land and remove his house
from it. RTC affirmed the MTC decision, holding that the case was one for forcible entry.
CA rendered judgment, reversing the decisions of the courts below, and ordering the
dismissal of the Dionisios' action. The CA held that, by amending their complaint, the
Dionisios effectively changed their cause of action from unlawful detainer to recovery of
possession which fell outside the jurisdiction of the MTC.
Issue:
Whether or not the Dionisios' amendment of their complaint effectively changed their cause
of action from one of ejectment to one of recovery of possession

Ruling:

An amended complaint that changes the plaintiff's cause of action is technically a new
complaint. Consequently, the action is deemed filed on the date of the filing of such
amended pleading, not on the date of the filing of its original version. Thus, the statute of
limitation resumes its run until it is arrested by the filing of the amended pleading. The Court
acknowledges, however, that an amendment which does not alter the cause of action but
merely supplements or amplifies the facts previously alleged, does not affect the reckoning
date of filing based on the original complaint. The cause of action, unchanged, is not barred
by the statute of limitations that expired after the filing of the original complaint. To determine
if an amendment introduces a different cause of action, the test is whether such amendment
now requires the defendant to answer for a liability or obligation which is completely different
from that stated in the original complaint. Here, both the original and the amended complaint
required Wilfredo to defend his possession based on the allegation that he had stayed on
the land after Emiliana left out of the owner's mere tolerance and that the latter had
demanded that he leave.

52. Sps. Barraza v. Campos, Jr.


G.R. No. L-50437; 28 February 1983.

Facts of the Case:

Respondent owned a lechon business, however petitioner used the trade name in
the CFO.

Procedural History:

A complaint for damages was filed. Petitioners in the CFI filed an “urgent ex parte
motion” for extension of time of 15 days within which to file an answer, which the CFI
granted on Nov. 2.

Statement of Issues:
Was the Order of respondent judge declaring petitioners in default for failure to file
their answer within the reglementary period provided by law correct?

Holding:

Respondent judge granted petitioners a 15-day extension to file their answer, or up


to Nov. 18. Instead of filing their answer, petitioners filed a motion to dismiss
complaint on Nov. 17, or one (1) day before the expiration of the period as extended
by the court. This is clearly allowed under Sec. 1, Rule 16.
53. David v. Gutierrez-Fruelda
G.R. No. 170427; 30 January 2009.

Facts of the Case:

A complaint for accounting, reconveyance, and damages with prayer for preliminary
attachment against petitioner. It was claimed that petitioner fraudulently exceeded
his special power of attorney to put land in his name and did not remit and account
any money from the transaction involving their lands.

Procedural History:

Service of summons was not served as petitioner was abroad. Petitioner also failed
to answer the service by publication, so private respondents moved that petitioner be
declared in default. On July 14, 2005, petitioner filed a motion for extension of 15
days to file answer, but the RTC declared petitioner in default in an Order dated July
15. The RTC noted that the period to file petitioner's Answer lapsed on May 19, 2005,
60 days after the last publication on March 19, 2005, and that petitioner failed to
answer despite the "many opportunities" given to him. The RTC also denied
petitioner's motion for extension to file Answer.

Statement of Issues:

Did the RTC commit grave abuse of discretion in denying petitioner's motion to lift
order of default?

Holding:

Petitioners voluntary appearance was equivalent to service of summons. It has cured


any alleged defect in the service of summons. Petitioner's motions were not motions
to dismiss the complaint on the ground of lack of jurisdiction over his person. On the
contrary, the motions invoked the RTC's jurisdiction while seeking the following
affirmative reliefs: to grant extension, deny the motion to declare petitioner in default
and lift the order of default. Thus, petitioner waived any defect in the service of
summons by publication or even want of process because for the RTC to validly act
on his motions, it necessarily acquired jurisdiction over his person. Unfortunately, his
claim that he has meritorious defenses is unsubstantiated. He did not even state
what evidence he intends to present if his motion is granted.
54. Republic v. Caguioa
G.R. No. 174385; 20 February 2013.

Facts of the Case:

A petitioner for TRO and mandatory injunction was filed against the Finance
Secretary, seeking to nullify section 6 of RA 9334.

Procedural History:

Respondent judge issued the preliminary injunction. The Republic filed a petition for
certiorari and prohibition with the SC. Private respondents filed before respondent
judge motions for leave to intervene and to admit complaints-in-intervention. Without
acting on the Republic's motion to suspend the proceedings, the respondent judge
granted on August 11, 2005 the private respondents' motions and complaints-in-
intervention. The respondent judge found the private respondents to be similarly
situated as the lower court petitioners; they stood, too, to be adversely affected by
the implementation of RA 9334. The Republic moved to reconsider the respondent
judge's August 11, 2005 order, arguing that it had been denied due process because
it never received copies of the private respondents' motions and complaints-in-
intervention.

Statement of Issues:

Whether the present petition was procedurally defective?

Holding:

The present petition was filed within the reglementary period. Contrary to the private
respondents' position, the 60-day period within which to file the petition
for certiorari is counted from the Republic's receipt of the July 5, 2006 order denying
the latter's motion for reconsideration. Section 4, Rule 65 of the Rules of Court is
clear on this point — "In case a motion for reconsideration or new trial is timely filed,
whether such motion is required or not, the sixty (60) day period shall be counted
from notice of the denial of said motion."
55. University of the Philippines v. Dizon
G.R. No. 171182; 23 August 2012.

Facts of the Case:

A construction agreement was entered into with petitioner and Stern Builders Corp
for a building in UPLB. UP did not pay all three buildings and the last one was
disallowed by COA. Stern Builders sued UP to collect the unpaid billing and to
recover damages.

Procedural History:

The RTC ruled in favor of Stern Builders Corp. UP filed a notice of appeal on June 3,
2002. Stern Builders opposed the notice of appeal on the ground of its filing being
belated, and moved for the execution of the decision. UP countered that the notice of
appeal was filed within the reglementary period because UP's Office of Legal Affairs
(OLS) in Diliman, Quezon City received the order of denial only on May 31, 2002. On
February 24, 2004, the CA dismissed the petition for certiorari upon finding that UP's
notice of appeal had been filed late.

Statement of Issues:

Whether the SC should give due course to UP’s petition for review in the name of
equity in order to reverse or modify the adverse judgment against it despite its finality?

Holding:

The service of the denial of the motion for reconsideration upon Atty. Nolasco of the
UPLB Legal Office was invalid and ineffectual because he was admittedly not the
counsel of record of UP. The rule is that it is on the counsel and not the client that
the service should be made. That counsel was the OLS in Diliman, Quezon City,
which was served with the denial only on May 31, 2002. As such, the running of the
remaining period of six days resumed only on June 1, 2002, rendering the filing of
UP's notice of appeal on June 3, 2002 timely and well within the remaining days of
UP's period to appeal. Verily, the service of the denial of the motion for
reconsideration could only be validly made upon the OLS in Diliman, and no other.
56. Cathay Metal Corp. v. Laguna West Multi-Purpose Cooperative, Inc.
G.R. No. 172204; 10 July 2014.

Facts of the Case:

A joint venture was entered into with respondent and farmer beneficiaries in CAvite.
While respondent was negotiating, petitioner entered into an irrevocable exclusive
right to buy contracts with the same farmers, where the farmer beneficiaries
committed themselves to sell to petitioner their agricultural properties upon
conversion. Respondent caused the annotation of its adverse claim on the
beneficiaries certificates of title. Contracts of sale of the properties were executed.
TCTs were also issued. A consolidated petition for cancellation of the adverse claims
were filed. The postman issued a certification which said that the reason for the
return was that the cooperative was inexistent. Therefore, petition was tried to be
filed personally by petitioner. This failed for the same reason. The RTC declared that
the service, by registered mail, was effected. Petitioner claimed that summons was
served upon respondent at its official registered address. Petitioner argued that
summons could only be validly served to respondent's official address as indicated in
its registration with the Cooperative Development Authority. This is because
respondent as a registered cooperative is governed by the Cooperative Code, a
substantive law that requires summons to be served to respondent's official address.
Substantive law takes precedence over procedural rules.

Procedural History:

Since no one received the summons, petitioner insisted that the RTC issue an order
to effect substituted service. Respondent still did not file its answer.

Statement of Issues:

Whether respondent was validly served with summons or notice of the hearing?

Holding:

The Rules of Court provides that notices should be sent to the enumerated officers.
The Cooperative Code provisions on notices cannot replace the rules on summons
under the Rules of Court. Rule 14, Section 11 of the Rules of Court provides an
exclusive enumeration of the persons authorized to receive summons for juridical
entities. These persons are the juridical entity's president, managing partner, general
manager, corporate secretary, treasurer, or in-house counsel. The enumeration in
Section 11 of Rule 14 is exclusive. Service of summons upon persons other than
those officers enumerated in Section 11 is invalid. Even substantial compliance is
not sufficient service of summons. This provision of the rule does not limit service to
the officers' places of residence or offices. If summons may not be served upon
these persons personally at their residences or offices, summons may be served
upon any of the officers wherever they may be found. Hence, petitioner cannot use
respondent's failure to amend its Articles of Incorporation to reflect its new address
as an excuse from sending or attempting to send to respondent copies of the petition
and the summons.
58.Rapid Realty and Development Corp. v. Villa
G.R. No. 184197; 11 February 2010.

Facts of the Case:

A declaration of nullity of subdivision plans was filed by petitioner. Summons were


attempted, but the first one, which was a personal service, failed. Zapanta, the court
process server, then resorted to substituted service, serving to the househelp who
did know acknowledge receipt.

Procedural History:

Respondents filed a Motion to Lift Order of Default, denying the existence of two
women helpers who allegedly refused to sign and acknowledge receipt of the
summons. The trial court set aside the Order of Default and gave respondents five
days to file their Answer. They failed to do so, thus petitioner filed a Motion to
declare them in default, which the trial court again granted. Respondents filed an
Omnibus Motion for reconsideration of the second order declaring them in default
and to vacate proceedings, which the court denied. The CA annulled the trial court's
Orders declaring respondents in default for the second time.

Statement of Issues:

Whether respondents, in filing the first Motion to Lift the Order of Default, voluntarily
submitted themselves to the jurisdiction of the court?

Holding:

It is settled that if there is no valid service of summons, the court can still acquire
jurisdiction over the person of the defendant by virtue of the latter's voluntary
appearance (Sec. 20, Rule 14 of the Rules of Court). The filing of motions to admit
answer, for additional time to file answer, for reconsideration of a default judgment,
and to lift order of default with motion for reconsideration, is considered voluntary
submission to the court's jurisdiction. This, however, is tempered by the concept of
conditional appearance, such that a party who makes a special appearance to
challenge, among others, the court's jurisdiction over his person cannot be
considered to have submitted to its authority. Prescinding from the foregoing, it is
thus clear that: (1) Special appearance operates as an exception to the general rule
on voluntary appearance; (2) Accordingly, objections to the jurisdiction of the
court over the person of the defendant must be explicitly made; and (3) Failure to do
so constitutes voluntary submission to the jurisdiction of the court, especially in
instances where a pleading or motion seeking affirmative relief is filed and submitted
to the court for resolution.
59. Green Star Express, Inc. v. Nissin-Universal Robina Corp.
G.R. No. 181517; 6 July 2015.

Facts of the Case:

Universal Robina Corp owned a van, which got into a vehicular accident which
resulted to the death of the van’s driver. A case was filed for reckless imprudence
resulting in homicide, and petitioner sent a demand letter to respondent. The criminal
case was dismissed.

Procedural History:

Tinio, one of NURC's employees, was the one who received the summons. NURC
filed a Motion to Dismiss claiming lack of jurisdiction due to improper service. The
RTC issued a Resolution denying NURC's motion to dismiss.The CA reversed the
RTC.

Statement of Issues:

Whether or not the summons was properly served on NURC, vesting the trial court
with jurisdiction.

Holding:

It is a well-established rule that the rules on service of summons upon a domestic


private juridical entity must be strictly complied with. Otherwise, the court cannot be
said to have acquired jurisdiction over the person of the defendant. NURC maintains
that the RTC did not acquire jurisdiction over it as the summons was received by its
cost accountant, Tinio. It argues that under Section 11, Rule 14 of the 1997 Rules of
Court, which provides the rule on service of summons upon a juridical entity, in
cases where the defendant is a domestic corporation like NURC, summons may be
served only through its officers. Here, Tinio, a member of NURC's accounting staff,
received the summons on January 22, 2004.
60. Delos Reyes v. Ramnani
G.R. No. 169135; 18 June 2010.

Facts of the Case:

A favorable decision was rendered for respondent in a civil case. A write of execution
was then issued by the RTC. The branch sheriff then conducted a public bidding.
Respondent was the highest bidder and a write of possession was issued. The
respondent filed an order to the sheriff to execute the final certificate of sale in her
favor. This was opposed by petitioner.

Procedural History:

The RTC granted the motion. This was affirmed by the CA. Petitioner contends that
the motion dated February 16, 2004 filed by respondent to compel the sheriff to
execute the final certificate of sale is fatally defective because it does not contain a
notice of hearing.

Statement of Issues:

Is a hearing needed for the motion?

Holding:

As a general rule, all written motions should be set for hearing under Section 4, Rule
15 of the Rules of Court excepted from this rule are non-litigious motions or motions
which may be acted upon by the court without prejudicing the rights of the adverse
party. As already discussed, respondent is entitled to the issuance of the final
certificate of sale as a matter of right and petitioner is powerless to oppose the
same. Hence, the subject motion falls under the class of non-litigious motions. At any
rate, the trial court gave petitioner an opportunity to oppose the subject motion as in
fact he filed a Comment/Opposition on March 1, 2004 before the trial court.
Petitioner cannot, therefore, validly claim that he was denied his day in court.
61. Preysler, Jr. v. Manila Southcoast Development Corp.
G.R. No. 171872; 28 June 2010.

Facts of the Case:

A complaint for forcible entry was filed by petitioner against the respondent.

Procedural History:

Such was filed in the MTC. The MTC ruled in favor of petitioner and ordered
respondent to vacate the disputed land in the name of petitioner and to return the
possession of the land to petitioner. RTC reversed the MTC decision and dismissed
petitioner's complaint. Petitioner received the RTC Decision and thereafter filed a
Motion for Reconsideration, which was set for hearing. It was only 6 days after the
scheduled hearing on Feb. 26, 2004, that respondent's counsel received a copy of
petitioner's Motion for Reconsideration. The rescheduled hearing on April 2, 2004
was again reset on May 7, 2004, and was further reset to Aug. 6, 2004. After the
hearing, respondent filed its Motion to Dismiss dated Aug. 9, 2004, claiming that
non-compliance with the three-day notice rule did not toll the running of the period of
appeal, which rendered the decision final.

Statement of Issues:

Whether the RTC was correct in dismissing petitioner's Omnibus Motion for allegedly
failing to comply with the three-day notice requirement?

Holding:

“The ordinary motion day is Friday. Hence, the notice should be served by Tuesday
at the latest, in order that the requirement of the three days may be complied with. If
notice be given by ordinary mail, it should be actually received by Tuesday, or if not
claimed from the post office, the date of the first notice of the postmaster should be
at least five (5) days before Tuesday.” Section 4 of Rule 15 provides that "[e]very
written motion required to be heard and the notice of the hearing thereof shall be
served in such a manner as to ensure its receipt by the other party at least three (3)
days before the date of the hearing, unless the court for good cause sets the hearing
on shorter notice." Thus, the date of the hearing should be at least three days after
receipt of the notice of hearing by the other parties. In this case, the petitioner's
Omnibus Motion was set for hearing on 12 November 2004. Thus, to comply with the
notice requirement, respondent should have received the notice of the hearing at
least three days before 12 November 2004, which is 9 November 2004.
62. PNB v. Deang Marketing Corp.
G.R. No. 177931; 8 December 2008.

Facts of the Case:

A dacion en pago arrangement in an agreement forged by petitioners transformed


respondent’ outstanding loan obligations into a seven year term loan. Respondents
filed before the RTC a Complaint. Summons was served on petitioner. Respondents
filed a Motion to Declare [Petitioner] in Default, which they set for hearing. On even
date, the trial court received petitioner's Motion for Extension of Time [30 days up to
June 11, 2006] to File Answer. Eight days prior to the slated hearing of respondents'
Motion to Declare [Petitioner] in Default, the trial court issued an Order denying said
motion and granting petitioner's Motion for Extension of Time to File Answer. To the
trial court's Order respondents filed a Motion for Reconsideration. In the meantime,
petitioner filed its Answer to the Complaint on May 25, 2006.

Procedural History:

The RTC denied respondents' Motion for Reconsideration of its May 16, 2006 Order
denying their Motion to Declare petitioner in default and granting the latter's Motion
for Extension. The CA annulled the RTC’s orders.

Statement of Issues:

Whether the CA was correct in declaring petitioner in default?

Holding:

The intention to delay was rather obvious. Petitioner's Motion for Extension of Time
to File Answer was laden with glaring lapses. It filed the Motion for Extension,
however, via a private courier (without any explanation for availing for such mode) on
May 14, 2006, which was received by the RTC on May 15, 2006 or ten days late. A
motion for extension of time to file a pleading must be filed before the expiration of
the period sought to be extended. The court's discretion to grant a motion for
extension is conditioned upon such motion's timeliness, the passing of which renders
the court powerless to entertain or grant it. Since the motion for extension was filed
after the lapse of the prescribed period, there was no more period to
extend. Petitioner was not candid enough to aver in the Motion for Extension that the
period had lapsed, as it still toyed with the idea that it could get away with it. The
allegations therein were crafted as if the said motion was timely filed. In requesting
for a 30-day extension or until June 11, 2006 to file answer, petitioner apparently
reckoned the date from which the extension would start on May 12, 2006, which
was not the last day of the 15-day period sought to be extended, it being May 5,
2006. By computation, petitioner actually sought more than 30 days, contrary to the
period of extension it purportedly requested. The counting of the period was
erroneous, even if one uses the material dates alleged by petitioner. No satisfactory
reason was adduced to justify the tardiness of the Answer and no compelling reason
was given to justify its admission.
63. Sps. Anunciacion v. Bocanegra
G.R. No. 152496; 30 July 2009.

Facts of the Case:

According to petitioners, respondents were served with summons and legal


processes through Atty Piazarri, whose office address is at 2830 Juan Luna St,
Tondo Manila. Atty Piazarro wrote a demand letter on behalf of respondents to
vacate the land, stating that respondents owned it.

Proceduralhistory

Petitioners filed before the RTC, Manila, a complaint for Quieting of Title.
Respondents, through their counsel, Atty. Caparas, filed a Motion to Dismiss on the
ground that the complaint stated no cause of action. Later, respondents filed a
Second Supplemental Motion to Dismiss and Manifestation, citing as grounds:
1) That the court has no jurisdiction over the person of the defending party; 2) That
the court has no jurisdiction over the subject matter of the claim; and 3) That the
pleading asserting the claim states no cause of action. The RTC sustained the
respondents, and the CA dismissed the petitioners’ appeal.

Statement of Issues:

Whether the CA erred acted with grave abuse of discretion?

Holding:

The dismissal of the complaint on the ground of lack of jurisdiction over the person of
the respondents after they had voluntarily appeared before the trial court clearly
constitutes grave abuse of discretion amounting to lack of jurisdiction or in excess of
jurisdiction on the part of the RTC. Quite apart from their voluntary appearance,
respondents' Supplemental Motion to Dismiss and Second Supplemental Motion to
Dismiss were clearly in violation of Rule 15, Section 8 in relation to Rule 9, Section 1
of the Rules. Under the Rules, the only grounds the court could take cognizance
of, even if not pleaded in the motion to dismiss or answer, are: (a) lack of
jurisdiction over the subject matter; (b) existence of another action pending between
the same parties for the same cause; and (c) bar by prior judgment or by statute of
limitations.
64. Panganiban v. Pilipinas Shell Petroleum Corp.
G.R. No. 131471; 22 January 2003.

Facts of the Case:

A sublease and dealer agreement was entered between Panganiban and Shell,
where Shell leased to Panganiban a gasoline station for a period which would
terminate at the same time as Shell’s lease. As Shell was not the owner of the lot,
and was leasing it under a lease agreement, it notified Panganiban that the
agreement was expiring and that Panganiban should wind up her business affairs,
but as she had believed it would expire later, she still occupied the lots and paid
rentals, although Shell did not accept.

Procedural History:

Panganiban filed a petition for declaratory relief with the RTC. Shell filed an unlawful
detainer case against Panganiban eight months after it submitted its Answer. Shell
filed a Manifestation. Shell claimed that the issue of the renewal of the lease should
be raised in the unlawful detainer case pending before the MTC. The MTC ruled in
favor of Shell. The RTC dismissed the petition for declaratory relief. The CA upheld
the order of the RTC dismissing the petition for declaratory relief on the ground
of litis pendentia.

Statement of Issues:

Whether the CA erred in affirming RTC-Makati's dismissal of Civil Case No. 95-1010
on motion of shell on the ground of litis pendentia which was filed long after Shell
had filed its answer?

Holding:

The requirement that a motion to dismiss should be filed within the time for filing the
answer is not absolute. Even after an answer has been filed, a defendant can still file
a motion to dismiss on the following grounds: (1) lack of jurisdiction, (2) litis
pendentia (3) lack of cause of action, and (4) discovery during trial of evidence that
would constitute a ground for dismissal. Litis pendentia is also one of the grounds
that authorize a court to dismiss a case motu proprio. In this case, the bona
fide existence of litis pendentia is beyond dispute.
65. Universal Robina Corp. v. Lim
G.R. No. 154338; 5 October 2007

Facts of the Case:

URC and Lim entered into a contract of sale where URC sold to Lim grocery
products totaling P808,059.88. After tendering partial payments, respondent refused
to settle his obligation despite petitioner's repeated demands.

Procedural History:

URC filed with the Quezon City RTC a complaint against Lim for a sum of money.
The RTC issued an Order dismissing the complaint motu proprio on grounds of lack
of jurisdiction and improper venue. URC filed a motion for reconsideration together
with an amended complaint alleging that the parties agreed that the proper venue for
any dispute relative to the transaction is Quezon City, which was granted by the RTC.
Summons was served upon Lim, but he failed to file an answer seasonably, and he
was declared in default upon motion of URC. However, the RTC again dismissed the
complaint on the ground of improper venue, stating: “There is not even a remote
connection by the parties to Quezon City, where this RTC sits, the plaintiff
corporation has principal office at Pasig City and the defendant is, as provided in the
complaint, from Laoag City.” The CA dismissed the appeal due to URC’s failure to
attach thereto an explanation why copies of the petition were not served by personal
service but by registered mail, in violation of Section 11, Rule 14 of the 1997 Rules
of Civil Procedure.

Statement of Issues:

Whether the trial court may dismiss motu proprio petitioner's complaint on the ground
of improper venue?

Holding:

No. Implicit from Sec. 1, Rule 9 of the Rules of Court is that improper venue not
impleaded in the motion to dismiss or in the answer is deemed waived. Thus, a court
may not dismiss an action motu proprio on the ground of improper venue as it is not
one of the grounds wherein the court may dismiss an action motu proprio on the
basis of the pleadings. In Dacoycoy v. IAC, this Court held that a trial court may
not motu proprio dismiss a complaint on the ground of improper venue. The court
may only dismiss an action motu proprio in case of lack of jurisdiction over the
subject matter, litis pendentia, res judicata and prescription. In the instant case,
respondent, despite proper service of summons, failed to file an answer and was
thus declared in default by the trial court. Verily, having been declared in default, he
lost his standing in court and his right to adduce evidence and present his defense,
including his right to question the propriety of the venue of the action.
66. Galindo v. Heirs of Roxas
G.R. No. 147969; 17 January 2005.

Facts of the Case:

Galindo possessed a lot of land and paid taxes thereon. When he died, his heirs
sold their rights and interests over the land to Roxas. Urbano, one of the heirs of
Galindo, was a minor then, so he did not sign the document (Documento de
Compromiso) but he executed an affidavit signifying his conformity to the sale
executed by his siblings; (4) Urbano later ratified the sale by executing an affidavit
signifying he was agreeable thereto. The CFI ordered the Galindo heirs to execute a
deed of absolute sale in favor of the heirs of Roxas. When Urbano died intestate, his
heirs executed an extrajudicial settlement of estate where they adjudicated unto
themselves as owners the undivided rights and interests of Urbano over the lot.

Procedural History:

The heirs of Roxas, through Reginald, filed a complaint against one of the heirs of
Urbano in the RTC. The RTC denied the motion, ruling that based on the plaintiffs'
allegation in the complaint that they were the legitimate children of the deceased
Roxas, such plaintiffs are the real parties-in-interest. The CA dismissed the petition
for certiorari.

Statement of Issues:

Whether the RTC committed a grave abuse of its discretion?

Holding:

An unauthorized complaint does not produce any legal effect. Section 1, Rule 3 of
the Rules of Court provides that only persons or juridical persons or entities
authorized by law may be parties in a civil action. Section 4, Rule 8 of the said
Rules further provides that facts showing the capacity of a party to sue or be sued or
the authority of a party to sue or be sued in a representative capacity must be
averred in the complaint. In order to maintain an action in a court of justice, the
plaintiff must have an actual legal existence, that is, he or she or it must be a person
in law and possessed of a legal entity as either a natural or an artificial person, and
no suit can lawfully be prosecuted in the name of that person. The party bringing suit
has the burden of proving the sufficiency of the representative character that he
claims. If a complaint is filed by one who claims to represent a party as plaintiff but
who, in fact, is not authorized to do so, such complaint is not deemed filed and the
court does not acquire jurisdiction over the complaint.
67. Chu v. Sps. Cunanan
G.R. No. 156185; 12 September 2011.

Facts of the Case:

5 registered lands were sold to Cunanan for P5m by Sps Chu. It was stipulated that
the ownership of the lots were still with the Chus. Such would be transferred to
Cunan only upon complete payment of the total consideration in the terms of the
deed of sale, A special power of attorny was issued authorizing Cunanan to borrow
P5, 161, 090 from any banking institution. The proceeds of the net balance would
then be delivered. But Cunanan transderred the title of the five lands to her name
without the knowledge of the spouses Chu, and then later to spouses Carlos.

Procedural History:

Sps. Chu commenced Civil Case No. G-1936 in the RTC to recover the unpaid
balance from Sps. Cunanan. The case was dismissed, and the parties entered into a
compromise agreement which was approved by the RTC. Sps. Chu later brought
another suit (Case No. 12251) against Sps. Carlos and Benelda Estate, seeking the
cancellation of their TCTs, and later impleaded Sps. Cunanan as additional
defendants. Defendants moved to dismiss the complaint based on res judicata. The
RTC denied the motions to dismiss as it found no identity of parties and subject
matter between the two cases. The CA reversed the RTC.

Statement of Issues:

Was the case barred by res judicata although the compromise agreement did not
expressly include Benelda Estate as a party and although the compromise
agreement made no reference to the lots now registered in Benelda Estate's name?

Holding:

A compromise agreement is a contract whereby the parties, by making reciprocal


concessions, avoid a litigation or put an end to one already commenced. The intent
of the parties to settle all their claims against each other is expressed. Apparently,
the petitioners were guilty of splitting their single cause of action to enforce or
rescind the deed of sale with assumption of mortgage. The petitioners were not at
liberty to split their demand to enforce or rescind the deed of sale with assumption of
mortgage and to prosecute piecemeal or present only a portion of the grounds upon
which a special relief was sought under the deed of sale with assumption of
mortgage, and then to leave the rest to be presented in another suit; otherwise, there
would be no end to litigation. Their splitting violated the policy against multiplicity of
suits, whose primary objective was to avoid unduly burdening the dockets of the
courts. Their contravention of the policy merited the dismissal of Civil Case.
68. Manila Bankers Life Insurance Corp. v. Aban
G.R. No. 175666; 29 July 2013.

Facts of the Case:

A life insurance policy was taken out by Sotero. In it, her niece respondent was
beneficiary. Petitioner issued the insurance policy. Sotero died. However, upon
investigation, petitioner found that Sotero did not personally apply for insurance, and
it was respondent who filed the insurance application.

Procedural History:

Petitioner filed a civil case for annulment of the policy before the RTC. Respondent
filed a Motion to Dismiss claiming that petitioner's cause of action was barred by
prescription pursuant to Section 48 of the Insurance Code. The RTC granted the
motion to dismiss, which was affirmed by the CA.

Statement of Issues:

Whether the courts below were correct in applying Sec. 48 of the Insurance Code?

Holding:

Section 48 serves a noble purpose, as it regulates the actions of both the insurer and
the insured. Under the provision, an insurer is given two years — from the effectivity
of a life insurance contract and while the insured is alive — to discover or prove that
the policy is void ab initio or is rescindible by reason of the fraudulent concealment or
misrepresentation of the insured or his agent. After the two-year period lapses, or
when the insured dies within the period, the insurer must make good on the policy,
even though the policy was obtained by fraud, concealment, or misrepresentation.
This is not to say that insurance fraud must be rewarded, but that insurers who
recklessly and indiscriminately solicit and obtain business must be penalized, for
such recklessness and lack of discrimination ultimately work to the detriment of bona
fide takers of insurance and the public in general. The self-regulating feature of
Section 48 lies in the fact that both the insurer and the insured are given the
assurance that any dishonest scheme to obtain life insurance would be exposed,
and attempts at unduly denying a claim would be struck down. Life insurance
policies that pass the statutory two-year period are essentially treated as legitimate
and beyond question, and the individuals who wield them are made secure by the
thought that they will be paid promptly upon claim. Section 48 contributes to the
stability of the insurance industry, and prevents a situation where the insurer
knowingly continues to accept annual premium payments on life insurance, only to
later on deny a claim on the policy on specious claims of fraudulent concealment and
misrepresentation, such as what obtains in the instant case.
69. ALICE VITANGCOL and NORBERTO VITANGCOL , petitioners, vs. NEW VISTA
PROPERTIES, INC., MARIA ALIPIT, REGISTER OF DEEDS OF CALAMBA, LAGUNA,
and the HONORABLE COURT OF APPEALS, respondents.

G.R. No. 176014. September 17, 2009

Facts of the Case


A land in Calamba, LAguna, was owned by both Maria and Clemente Alipit. They then
executed a special power of attorney making Milagros A De Guzman their attorney-in-fact to
sell their property. De Guzman then executed a deed of absolute sale to New Vista
Properties. When New Vista leanred that the parcel of land was claimed by petitioners
Vitangcol by a deed of sale between Maria and Vitangcol.
Procedural History

New Vista filed a notice of adverse claim over the lot and a suit for quieting of title before the
RTC of Calamba Court.. RTC ruled in the favor of Vitangcol. According to the RTC, what the
agent (De Guzman) sold to New Vista was a different lot which she was not authorized to
sell. The Court of Appeals reversed the decision.

Statement of the Issue

Whether or not the Amended Complaint of Vitangcol states a cause of action?

Holding

The trial court acted within its discretion in considering said SPA relative to the motion to
dismiss the amended complaint, as Vitangcol duly submitted that piece of document in court
in the course of the June 7, 2004 hearing on the motion to dismis The trial court, however,
erred in ruling that, taking said SPA into account, the amended complaint stated no cause of
action. Indeed, upon a consideration of the amended complaint, its annexes, with the June
18, 1989 SPA thus submitted, the Court is inclined, in the main, to agree with the appellate
court that the amended complaint sufficiently states a cause of action. In a motion to dismiss
for failure to state a cause of action, the focus is on the sufficiency, not the veracity, of the
material allegations. The test of sufficiency of facts alleged in the complaint constituting a
cause of action lies on whether or not the court, admitting the facts alleged, could render a
valid verdict in accordance with the prayer of the complaint. And to sustain a motion to
dismiss for lack of cause of action, it must be shown that the claim for relief in the complaint
does not exist, rather than that a claim has been defectively stated, or is ambiguous,
indefinite, or uncertain.
70. BENJAMIN CUA (CUA HIAN TEK), petitioner, vs. WALLEM PHILIPPINES SHIPPING,
INC. and ADVANCE SHIPPING CORPORATION, respondents.

[G.R. No. 171337. July 11, 2012.]

Facts of the Case

Cua sought payment of 2 million pesos as reparation for damage to his shipment of soybean
against respondent.

Procedural history:

Respondent filed a motion to dismiss the complaint, assailing the RTC's jurisdiction over
Cua's claim; it argued that Cua's claim should have first been brought to arbitration. Cua
opposed Advance Shipping's argument; he contended that he, as a consignee, was not
bound by the Charter Party Agreement, which was a contract between the ship owner
(Advance Shipping) and the charterers.

Statement of the Issue

Whether or not Cua's claim for payment of damages against the respondents has prescribed?

Holding

Cua’s claim for payment of damages has not prescribed due to the admitted fact that there
was indeed an extension of their agreement. Cua timely filed his claim before the trial court
and the CA failed to appreciate the admissions made by the respondents in their pleadings
that negate a finding of prescription of Cua's claim. It was ruled that the action has not
prescribed because jurisprudence recognized the validity of agreements extending such
period pursuant to COGSA. The allegation of an agreement extending the period to file an
action in Cua's complaint is a material averment that, under Section 11, Rule 8 of the Rules
of Court, must be specifically denied by the respondents; otherwise, the allegation is
deemed admitted. A review of the pleadings submitted by the respondents discloses that
they failed to specifically deny Cua's allegation of an agreement extending the period to file
an action to November 12, 1990. Wallem's motion to dismiss simply referred to the fact that
Cua's complaint was filed more than one year from the arrival of the vessel, but it did not
contain a denial of the extension. Since the COGSA is the applicable law, the respondents'
discussion to support their claim of prescription under Article 366 of the Code of Commerce
would, therefore, not constitute a refutation of Cua's allegation of extension.
71. SWEDISH MATCH, AB, JUAN ENRIQUEZ, et al. petitioners, vs. COURT OF
APPEALS, ALS MANAGEMENT & DEVELOPMENT CORPORATION and ANTONIO K.
LITONJUA, respondents.

G.R. No. 128120. October 20, 2004

Facts of the Case

Petitioner had three subsidiary corporations in The Philippines. Phimco and Phils are some
of them. Petitioner decided to sell them, and several corporations tried to get Phimco, one of
them being respondent. A case was filed against petitioners to sell the Phimco, but petitioner
alleged that there is no cause of action as there was no perfected contract between them.

Procedural History

RTC dismissed respondents' complaint. The Court of Appeals reversed the trial court's
decision. It ruled that the series of written communications between petitioners and
respondents collectively constitute a sufficient memorandum of their agreement.

Statement of the Issue

Whether or not the appellate court erred in reversing the trial court's decision?

Holding

The Court of Appeals erred procedurally in motu propio dismissing the complaint in its
entirety for “lack of a valid cause of action: without the benefit of a full blown trial and on the
mere motion to dismiss. Respondents' attempt to prove the alleged verbal acceptance of
their US$36 million bid becomes futile in the face of the overwhelming evidence on record
that there was in the first place no meeting of the minds with respect to the price.
72. CALIFORNIA AND HAWAIIAN SUGAR COMPANY; PACIFIC GULF MARINE, INC.
and C.F. SHARP & COMPANY , petitioners, vs. PIONEER INSURANCE AND SURETY
CORPORATION, respondent.

G.R. No. 139273. November 28, 2000.

Facts of the Case

A cargo of soyben meal arrived in manila via the vessel MV Sugarl Islander. Such was
consigned to several consignees, one of them being the Metro Manila Feed Millers
Association. The cargo from the vessel to barges were discharged on Nov 30. Respondent
says that when the cago was weighed, a shortage was discovered. Petitioners refused to
settle.

Procedural History

Respondent filed a complaint for damages against herein petitioners. Petitioners filed a
Motion to Dismiss the complaint on the ground that respondent's claim is premature, the
same being arbitrable. The RTC deferred the hearing on the motion to dismiss and directed
the petitioners to file an answer. The trial court denied petitioners' Motion for a Preliminary
Hearing. Affirming the trial court, the Court of Appeals ruled that a preliminary hearing on
affirmative defenses could not be allowed because a motion to dismiss had previously been
filed. Further, it ruled that the arbitration clause did not bind the respondent.

Statement of the Issue

Whether or not the trial court committed grave abuse of discretion when it denied petitioners'
Motion for a Preliminary Hearing?

Holding

Where a preliminary hearing appears to suffice, there is no reason to go on to trial.In the


present case, the trial court did not categorically resolve petitioners' Motion to Dismiss, but
merely deferred resolution thereof. Section 6, Rule 16 of the 1997 Rules, specifically
provides that a preliminary hearing on the affirmative defenses may be allowed only when no
motion to dismiss has been filed. Section 6, however, must be viewed in the light of Section
3 of the same Rule, which requires courts to resolve a motion to dismiss and prohibits them
from deferring its resolution on the ground of indubitability. The ground invoked may still be
pleaded as an affirmative defense even if the defendant's Motion to Dismiss has been filed
but not definitely resolved, or if it has been deferred as it could be under the pre-1997 Rules.
The trial court committed grave abuse of discretion when it denied a preliminary hearing on a
simple issue of fact that could have possibly settled the entire case. One reason why
dockets of trial courts are clogged is the unreasonable refusal to use a process or procedure,
like a motion to dismiss, which is designed to abbreviate the resolution of a case.
73. SPS. ENRIQUETA RASDAS, and TOMAS RASDAS, SPS. ESPERANZA A. VILLA,
and ERNESTO VILLA, and LOLITA GALLEN, petitioners, vs. JAIME ESTENOR,
respondent.

G.R. No. 157605. December 13, 2005.

Facts of the Case

A parcel of land in Isabela is the subject of this case. Respondent filed a complaint for
recovery of owndership of the land, asserting that he was the owner. The RTC ruled in favor
of petitioners. Respondent then appealed the RTC decision before the Court of Appeals,
which reversed the judgement of the RTC. Petitioners were ordered to vacate the land.

Procedural History

Petitioners filed a Complaint against respondent for just compensation and preliminary
injunction with temporary restraining order. Petitioners asserted therein that they were the
lawful owners of the subject property, although they ultimately conceded the efficacy of the
appellate court's final and executory decision. Still, they alleged that they were entitled to just
compensation relating to the value of the houses they had built on the property, owing to
their purported status as builders in good faith. Respondent as defendant countered with a
Motion to Dismiss, arguing that petitioners' complaint was barred by res judicata, owing to
the final and executory judgment of the Court of Appeals.

Statement of the Issue

Whether or not the complaint is barred by res judicata?

Holding

It was held that it is the second kind of res judicata, conclusiveness of judgment," that barred
the instant complaint. Any right, fact, or matter in issue directly adjudicated or necessarily
involved in the determination of an action before a competent court in which judgment is
rendered on the merits is conclusively settled by the judgment therein and cannot again be
litigated between the parties and their privies whether or not the claim, demand, purpose, or
subject matter of the two actions is the same. The Supreme Court observed that the causes
of action in the two cases involved were so glaringly similar that it had to affirm the dismissal
of the second case by virtue of the "bar of former judgment rule."
74. SUSIE CHAN-TAN, petitioner, vs. JESSE C. TAN, respondent.

G.R. No. 167139. February 25, 2010

Facts of the Case

Petitioner and respondent were married in Manila Cathedral. They were blessed with two
sons. Justin was born in Canada and Russel was born in The Philippines. Petitioner then
filed a case for annulment under Article 36. The trial court issued a partial judgement
approving the compromise and declared the marriage void. On 4 November 2004, petitioner
filed a motion to dismiss and a motion for reconsideration. She claimed she was no longer
interested in the suit. Petitioner stated that the circumstances in her life had led her to the
conclusion that withdrawing the petition was for the best interest of the children. She prayed
that an order be issued vacating all prior orders and leaving the parties at the status quo
ante the filing of the suit.

Procedural History

The trial court denied both the motion to dismiss and the motion for reconsideration filed by
petitioner. It held that the decision is final and executory upon the lapse of the 15-day
reglementary period without any timely appeal having been filed by either party. I

Statement of the Issue

Whether or not the annulment decided by the trial court attained finality?

Holding

It as decided that it was executory when the husband failed to appeal during the
reglementary period. The husband claimed that the decision of the trial court was null and
void for violation of his right to due process. He argued he was denied due process when,
after failing to appear on two scheduled hearings, the trial court deemed him to have waived
his right to present evidence and rendered judgment based solely on the evidence presented
by private respondent. We upheld the judgment of nullity of the marriage even if it was based
solely on evidence presented by therein private respondent. The alleged negligence of
counsel resulting in petitioner's loss of the right to appeal is not a ground for vacating the trial
court's judgments. Further, petitioner cannot claim that she was denied due process. While
she may have lost her right to present evidence due to the supposed negligence of her
counsel, she cannot say she was denied her day in court. When petitioner filed the motion to
dismiss on 4 November 2004, the 30 March 2004 decision and the 17 May 2004 resolution
of the trial court had long become final and executory upon the lapse of the 15-day
reglementary period without any timely appeal having been filed by either party. Nothing is
more settled in law than that when a judgment becomes final and executory, it becomes
immutable and unalterable.
75. ALDERSGATE COLLEGE, INC et al. petitioners, vs. JUNIFEN F. GAUUAN et al.
respondents

G.R. No. 192951. November 14, 2012.

Facts of the Case

A case was filed against respondents before the SEC. When the SEX was reorganized
pursuant to RA 8799, the case was transferred to the RTC of Nueva Vizcaya for further
proceedings. The case was sought to be dismissed on the complaint or the issuance of a
summary judgement. The RTC denied the motion on the ground that "there are several
issues raised which would still need the presentation of evidence to determine the rights of
the parties."

Procedural History

Respondents-intervenors also sought the dismissal of the complaint in their Answer-in-


Intervention with Motion to Dismiss. The RTC, did not allow. The respondents-intervenors
again filed in February 2010 a Motion to Withdraw and/or to Dismiss Case. The RTC granted
the motion despite the opposition of the petitioners, and dismissed the case on the basis of
the Resolution passed by the members of the Board of Trustees of petitioner Aldersgate
College dated December 14, 2009 recommending the dismissal of the case.

Statement of the Issue

Whether or not the Regional Trial Court erred in dismissing the case?

Holding

As this case involves an intra-corporate dispute, the motion to dismiss is undeniably a


prohibited pleading. Moreover, the Court finds no justification for the dismissal of the case
based on the mere issuance of a board resolution by the incumbent members of the Board
of Trustees of petitioner corporation recommending its dismissal, especially considering the
various issues raised by the parties before the court a quo. Hence, the RTC should not have
entertained, let alone have granted the subject motion to dismiss.
76. VILMA QUINTOS AND CATALINO IBARRA, petitioners, vs. PELAGIA NICOLAS et
al., respondents

G.R. No. 210252. June 16, 2014.

Facts of the Case

Petitioners and respondents are siblings. Their parents were owners of the subject property,
whcih was located in Quezon city. When the parents died, they left to their ten children the
ownership over the subject property. Respondent siblings brought an action against petition

Procedural History

Petitioners filed a complaint for Quieting of Title and Damages against respondents wherein
they alleged that during their parents' lifetime, the couple distributed their real and personal
properties in favor of their ten (10) children. They likewise averred that they have been in
adverse, open, continuous, and uninterrupted possession of the property for over four (4)
decades and are, thus, entitled to equitable title thereto. They also deny any participation in
the execution of the aforementioned Deed of Adjudication dated September 21, 2004 and
the Agreement of Subdivision. Respondents countered that petitioners' cause of action was
already barred by estoppel when sometime in 2006, one of petitioners offered to buy the
7/10 undivided share of the respondent siblings.

Statement of the Issue

Whether or not the respondents' counterclaim for partition is already barred by laches or res
judicata?

Holding

Dismissal with prejudice under Rule 17, Sec. 3 of the Rules of Court cannot defeat the right
of a co-owner to ask for partition at any time, provided that there is no actual adjudication of
ownership of shares yet. The Supreme Court held that Art. 494, as cited, is an exception to
Rule 17, Sec. 3 of the Rules of Court to the effect that even if the order of dismissal for
failure to prosecute is silent on whether or not it is with prejudice, it shall be deemed to be
without prejudice. This is not to say, however, that the action for partition will never be barred
by res judicata. There can still be res judicata in partition cases concerning the same parties
and the same subject matter once the respective shares of the co-owners have been
determined with finality by a competent court with jurisdiction or if the court determines that
partition is improper for co-ownership does not or no longer exists.
77. BENEDICTA M. SAMSON and MARCIAL M. SAMSON, petitioners, vs. HON. JUDGE
GERALDINE C. FIEL-MACARAIG, FAR EAST BANK AND TRUST CO et al.,
respondents.

G.R. No. 166356. February 2, 2010.

Facts of the Case

Benedicta and Marcial Samson obtained a loan of 10k pesos from Far East Bank and Trust
Co. The loan was secured by a real estate morgage over four parcels of land in Marikina.
When petitioners did not comply with the terms of the loan, FEBTC filed for extra judicial
foreclosure of the real estate mortgage at the RTC. These were sold to FEBTC.

Procedural History

Petitioners filed a case for annulment of the sale. Six months later the RTC issued an Order
dismissing the complaint. The CA dismissed. The appellate court ruled that a writ of
certiorari lies only where there is no appeal or plain, speedy, and adequate remedy in the
ordinary course of law. The RTC Order subject of the petition was a final judgment which
disposed of the case on the merits; hence, it was a subject for an ordinary appeal, not a
petition for certiorari. The Court of Appeals added that even assuming that the petitioners
availed of the proper remedy, they failed to show that public respondent gravely abused her
discretion by acting in a despotic or arbitrary manner, or that she was motivated by passion
or personal hostility when she issued the assailed Orders.

Statement of the Issue

Whether or not the case was dismissed with prejudice?

Holding

The RTC dismissed the case with prejudice for failure to prosecute for an unreasonable
length of time, pursuant to Section 3, Rule 17 of the Rules of Court. The RTC Order dated
20 June 2003 was a final judgment which disposed of the case on the merits. This was even
clarified in the subsequent RTC Order of 22 December 2003 which denied petitioners'
motion for reconsideration wherein the lower court stated that: "Therefore, the dismissal was
'with prejudice' or a dismissal that had the effect of adjudication upon the merits in
accordance with Section 3, Rule 17 of the Rules of Court." The remedy to obtain reversal or
modification of the judgment on the merits is appeal.The availability of the right to appeal
precludes recourse to the special civil action for certiorari. The RTC Order subject of the
petition was a final judgment which disposed of the case on the merits; hence, it was a
subject for an ordinary appeal, not a petition for certiorari.
78. ELOISA MERCHANDISING, INC. and TREBEL INTERNATIONAL, INC., petitioners,
vs. BANCO DE ORO UNIVERSAL BANK and ENGRACIO M. ESCASINAS, JR., ,
respondents

G.R. No. 192716. June 13, 2012

Facts of the Case

A real estate mortgage over properties located in Makati were executed by petitioner on
behalf of respondent. A continuing suretyship was executed in favor of BDO to secure the
credit accomondation. BDO inisiated foreclosure proceedings. Petitioners filed a complaint
for annulment of real estate mortgage. BDO filed a motion to dismiss on the ground of lack
of cause of action, which can be determined from the facts alleged in the complaint.

Procedural History

Petitioners were allowed to present evidence ex parte in view of the absence of BDO during
the pre-trial. The trial court reinstated the case and set the case again for pre-trial
conference pursuant to BDO’s reconsideration. For failure of the petitioners to appear
despite due notice at the scheduled pre-trial conference, case was ordered dismissed. The
trial court dismissed the case for failure of petitioners to prosecute their case. CA affirmed
the trial court's dismissal of the case.

Statement of the Issue

Whether or not the dismissal is proper?

Holding

Section 3, Rule 17 of the 1997 Rules of Civil Procedure stated that the failure on the part of
the plaintiff, without any justifiable cause, to comply with any order of the court or the Rules,
or to prosecute his action for an unreasonable length of time, may result in the dismissal of
the complaint either motu proprio or on motion by the defendant. However, despite the trial
court's leniency and admonition, petitioners continued to exhibit laxity and inattention in
attending to their case.
79. HEIRS OF DR. MARIANO FAVIS, SR., petitioners, vs. JUANA GONZALES, her son
MARIANO G. FAVIS, MA. THERESA JOANA D. FAVIS, JAMES MARK D. FAVIS,
respondents.

G.R. No. 185922. January 15, 2014

Facts of the Case

Dr. Favis was married to Captolina Aguilar. They had seven children. When Captilona died,
Dr Favis took a common law wife. They had one child. Finally, he married his common law
wife, and he acknowledged their son as legitimate. When Dr Favis died, he executed a deed
of fonation transferring and conveying properties in favor of his grandchildren with the
common law wife.

Procedural History

Dr. Favis' children with his first wife, petitioners herein, filed an action for annulment of the
Deed of Donation. The RTC nullified the Deed old Donation. The trial court found that Dr.
Favis, at the age of 92 and plagued with illnesses, could not have had full control of his
mental capacities to execute a valid Deed of Donation. The Court of Appeals motu proprio
ordered the dismissal of the complaint for failure of petitioners to make an averment that
earnest efforts toward a compromise have been made, as mandated by Article 151 of the
Family Code.

Statement of the Issue

Whether or not the appellate court may dismiss the order of dismissal of the complaint for
failure to allege therein that earnest efforts towards a compromise have been made?

Holding

The appellate court committed egregious error in dismissing the complaint. The facts of the
case show that compromise was never an option insofar as the respondents were
concerned. The impossibility of compromise instead of litigation was shown not alone by the
absence of a motion to dismiss but on the respondents' insistence on the validity of the
donation in their favor of the subject properties Respondents not only confined their
arguments within the pre-trial order; after losing their case, their appeal was based on the
proposition that it was error for the trial court to have relied on the ground of vitiated consent
on the part of Dr. Favis. The Court of Appeals ignored the facts of the case that clearly
demonstrated the refusal by the respondents to compromise. Instead it ordered the
dismissal of petitioner's complaint on the ground that it did not allege what in fact was shown
during the trial. The error of the Court of Appeals is patent.
80. RAMON CHING AND PO WING PROPERTIES, INC., petitioners, vs. JOSEPH
CHENG, JAIME CHENG, MERCEDES IGNE AND LUCINA SANTOS, respondents.

G.R. No. 175507. October 8, 2014

Facts of the Case

Antonio Ching owned P Wing Enterprises. Ching alleged that he was the only child of
Ramon with his common law wife. The wife disputed this. She states that even if Ramon’s
certificate alleges that he was Antonio’s illegitimage child, he was just adopted. Joseph and
Jaime, on the other hand, claim to be his illegitimage children with the maid. LAter, Ching
was murdered. Ramon induced the maid and her children to sign an agreement that waived
the estate.

Procedural History

A complaint for declaration of nullity of titles against Ching were filed by petitioners.
Petitioners filed a complaint for "Annulment of Agreement, Waiver, Extra-Judicial Settlement
of Estate and the Certificates” against Ramon Ching and Po Wing Properties. The dismissal
of the second case was made without prejudice. Respondents argue that the dismissal
should have been with prejudice under the "two-dismissal rule" of Rule 17, Section 1 of the
1997 Rules of Civil Procedure, in view of the previous dismissal of the first case.

Statement of the Issue

Whether or not the "two-dismissal rule” is applicable in this case?

Holding

Rule 17 of the Rules of Civil Procedure governs dismissals of actions at the instance of the
plaintiff. Hence, the "two-dismissal rule" under Rule 17, Section 1 of the Rules of Civil
Procedure will not apply if the prior dismissal was done at the instance of the defendant. The
dismissal of the first case was done at the instance of the defendant under Rule 16, Section
1 (b) of the Rules of Civil Procedure. Thus, when respondents filed the second case, they
were merely re-filing the same claim that had been previously dismissed on the basis of lack
of jurisdiction. When they moved to dismiss the second case, the motion to dismiss can be
considered as the first dismissal at the plaintiff's instance. The trial court dismissed the first
case by granting the motion to dismiss filed by the defendants.
81. PARA AQUE KINGS ENTERPRISES, INC., petitioner, vs. CATALINA L. SANTOS,
represented by her Attorney-in-Fact, LUZ B. PROTACIO, and DAVID R. RAYMUNDO,
respondents.

G.R. No. 194638. July 2, 2014.

Facts of the Case

Catalina Santos entered into a contract of lease with Chua. This lease was over land in
Paranaque. Chua was given the first option to buy. Chua then caused the construction of a 6
door commercial complex on the leased premise. However, by reason of business reverses,
he was constrained to assign his rights to Bing. Bing assumed all oblications under the lease
contract with Santos. Lee executed a deed of assignment over the leased premises.
Petitioner then filed a complaint before the RTC against Santos. Respondents moved to
dismiss.

Procedural History

On July 2, 1998, Petitioner filed a Motion to Cancel Pre-Trial, claiming that it was preparing a
petition for certiorari and prohibition which (a) was to be filed with the CA before the
scheduled pre-trial, and (b) was intended to challenge the validity of the RTC's Orders by
raising alleged prejudicial questions that must be resolved first before the pre-trial and trial
on the merits of the case could proceed. Incidentally, the petition for certiorari and prohibition
that was actually filed at 2:17 in the afternoon of July 7, 1998, contrary to petitioner's
assertion in its Motion to Cancel Pre-Trial that it was to be filed before the July 7, 1998 pre-
trial.

Statement of the Issue

Whether or not the CA correctly upheld the RTC's denial of petitioner's Motion to Cancel
Pre-Trial?

Holding

The trial court has the discretion on whether to grant or deny a motion to postpone and/or
reschedule the pre-trial conference in accordance with the circumstances obtaining in the
case. This must be so as it is the trial court which is able to witness firsthand the events as
they unfold during the trial of a case. Postponements, while permissible, must not be
countenanced except for clearly meritorious grounds and in light of the attendant
circumstances. In this case, the RTC was able to explain to the satisfaction of the Court that
the postponement of the pre-trial.
82. ROBERTO G. ALARCON, petitioner, vs. THE COURT OF APPEALS and
BIENVENIDO JUANI, respondents.

G.R. No. 126802. January 28, 2000

Facts of the Case

Roberto Alarcon filed a complaint for the annulment of a deed of sale agaisnt respondents.
Alacron says that before he went to Brunei to work, he left with his father a special power of
attorney to administer, mortgage, or sell his properties in Bulacan. Using the SPA, a cportion
of the land was sold to respondents.

Procedural History

The parties made admissions of facts during the pre-trial conference. The partial decision
declared that the deed of sale in favor of the defendants was void ab initio. For failure of the
defendants to interpose an appeal from the partial decision, the same became final and
executory. After several years, respondent Bienvenido Juani filed with the Court of Appeals
a petition for the annulment of the trial court's partial decision. After hearing the case, the
Court of Appeals granted the petition and set aside the partial decision. Its decision was
anchored on its finding that the partial decision was vitiated by extrinsic fraud.

Statement of the Issue

Whether or not there was intrinsic or collateral fraud attendant, which would justify the
setting aside of the partial decision?

Holding

From the foregoing, the admissions were clearly made during the pre-trial conference and,
therefore, conclusive upon the parties making it. The purpose of entering into a stipulation of
facts or admissions of facts is to expedite trial and to relieve the parties and the court, as
well, of the costs of proving facts which will not be disputed on trial and the truth of which
can be ascertained by reasonable inquiry. The parties themselves are required to attend or
their representative with written authority from them in order to arrive at a possible amicable
settlement, to submit to alternative modes of dispute resolution, and to enter into stipulations
or admissions of facts and documents. All of the matters taken up during the pre-trial,
including the stipulation of facts and the admissions made by the parties are required to be
recorded in a pre-trial order. Juani cannot now claim that he was denied his day in court
when judgment was rendered on the basis of their (Juani, Baluyot and Sulit) counsels'
admissions.
83. SILVESTRE TIU, petitioner, vs. DANIEL MIDDLETON and REMEDIOS P.
MIDDLETON, respondents.

G.R. No. 134998. July 19, 1999

Facts of the Case

A complaint for recovery of possession of real property was filed against petitioner. Before
the commencement of the trial, the court a quo sent a notice of Pre-trial Conference stating
in part: "The parties are warned that witnesses whose names and addresses are not
submitted at the pre-trial may not be allowed to testify at the trial . . . ." In his Pre-trial Brief,
petitioner averred that he would be presenting six witnesses, but he did not name them.

Procedural History

After the pre-trial conference, the court issued a Pre-trial Order. In such order, it was said
that the petitioner would present six witnesses and specifying the hearing dates for the said
purpose. Trial ensued, and herein respondents, as plaintiffs in the case below, presented
their witnesses in due course. When his turn came, petitioner called Antonia Tiu as his first
witness. Respondents objected, arguing that the witness could not be allowed to testify
because petitioner had failed to name her in his Pre-trial Brief citing Section 6, Rule 18 of the
1997 Rules of Court, . Sustaining respondents, the lower court then issued the assailed
orders.

Statement of the Issue

Whether or not a judge can exclude a witness whose name and synopsis of testimony were
not included in the pre-trial brief?

Holding

It was emphasized that pre-trial and its governing rules are not technicalities which the
parties may ignore. As earlier stated, pre-trial is essential in the simplification and the speedy
disposition of disputes. In light of the objectives of a pre-trial and the role of the trial court
therein, it is evident that judges have the discretion to exclude witnesses and other pieces of
evidence not listed in the pre-trial brief, provided the parties are given prior notice to this
effect. Indeed, the court and the parties must pay attention not only to the pre-trial briefs, but
also to the pre-trial order. Respondents' failure to challenge the pre-trial brief submitted by
petitioner before the trial nor invoke the power of the trial court to compel the petitioner to
submit the names of his witnesses and summaries of their testimonies is fatal to their case.
The provision in the Pre-trial Order allowing petitioner to present six witnesses "shall control
the subsequent course of action." The court proceeded with the trial without modifying the
Order. In the same vein, respondents did not challenge it before the trial. Neither did they
invoke the power of the trial court to compel the petitioner to submit the names of his
witnesses and summaries of their testimonies. By their silence, respondents acquiesced to
the Pre-trial Order allowing the presentation of petitioner's unnamed witnesses. Modifying a
pre-trial order during the trial or, worse, when the defendant is about to present witnesses
will indubitably result in manifest injustice. This could not have been the intention of the
Rules.
84. JACINTO SAGUID, petitioner, vs. HON. COURT OF APPEALS, THE REGIONAL
TRIAL COURT, BRANCH 94, BOAC, MARINDUQUE and GINA S. REY, respondents.

G.R. No. 150611. June 10, 2003

Facts of the Case

Gina was married. But then she petitioner and separated from her husband. They cohabited
as husband and wife. They had no children. When their relationship went bitter, she went to
work as an entertainer in Japan. The couple then separated. Respondent filed a complaint
for Partition and Recovery of Personal Property with Receivership against the petitioner with
the Regional Trial Court of Boac, Marinduque. She alleged that from her salary of $1,500.00
a month as entertainer in Japan, she was able to contribute P70,000.00 in the completion of
their unfinished house. She prayed that she be declared the sole owner of these personal
properties and that the amount of P70,000.00, representing her contribution to the
construction of their house, be reimbursed to her. petitioner claimed that the expenses for
the construction of their house were defrayed solely from his income as a captain of their
fishing vessel.

Procedural History

The trial court declared the petitioner as in default for failure to file a pre-trial brief. Petitioner
filed a motion for reconsideration which was denied, and private respondent was allowed to
present evidence ex parte. Petitioner filed another motion for reconsideration but the same
was also denied. A decision was rendered in favor of private respondent. CA affirmed.

Statement of the Issue

Whether or not the trial court erred in allowing private respondent to present evidence ex
parte?

Holding

Under Section 6, Rule 18 of the 1997 Rules of Civil Procedure, the failure of the defendant to
file a pre-trial brief shall have the same effect as failure to appear at the pre- trial, i.e., the
plaintiff may present his evidence ex parte and the court shall render judgment on the basis
thereof.
85. GEORGE S. TOLENTINO et al, petitioners vs. PACIFICO S. LAUREL et al,
respondents

G.R. No. 181368. February 22, 2012

Facts of the Case

Respondents allege that they are registered owners of a parcel of land in Quezon. However,
petitioners have been in actual possession of the Western portion of the said property which
they had tried to develop into fishponds.

Procedural History

Respondents filed a suit against petitioners to recover the property. Petitioners, as


defendants in the trial court, averred in their Answer that the subject property is owned by
the Republic and they are occupying the same by virtue of a Fishpond Lease Agreement
entered with the Department of Agriculture. Petitioners were declared in default, for failure to
appear at the pre-trial conference. However, the trial court set aside the default order and
reset the pre- trial conference. Despite several resetting of the pre-trial conference of which
petitioners were notified, petitioners failed to appear. Hence, the trial court issued an Order
allowing respondents to present their evidence ex parte, instead of declaring petitioners in
default. After the ex parte hearing for the reception of evidence, the RTC ruled in favor of
respondents. CA affirmed.

Statement of the Issue

Whether or not Petitioners were denied their day in court?

Holding

There was no lack of due process claimed by petitioners. On the contrary, petitioners were
given more than ample opportunity to be heard through counsel. From the foregoing, the
failure of a party to appear at the pre-trial has adverse consequences. If the absent party is
the plaintiff, then his case shall be dismissed. If it is the defendant who fails to appear, then
the plaintiff is allowed to present his evidence ex parte and the court shall render judgment
on the basis thereof. Thus, the plaintiff is given the privilege to present his evidence without
objection from the defendant, the likelihood being that the court will decide in favor of the
plaintiff, the defendant having forfeited the opportunity to rebut or present its own evidence.
86. Corpuz v. Corpuz

G.R. Nos. 175677 & 177133, July 31, 2009

Facts of the Case

Azucena Corpus had both a mastercard and VISA of Citibank, which she paid in full on
December 7, 1998. While she was in Italy, she dined at a restaurant. When she tried to
settle her bill, she presented her VISA which the restaurant did not honor, but then she
presented her MasterCard which was honored. On the same day she incurred a bill with a
store but both her VISA and Mastercard were not honored. Azucena’s husband inquired why
his wife’s credit cards were not honored, and Citibank explained that her check-payments
had not yet been cleared at the time. When she returned, she wrote letters to
Citibank.Citibank did not respond to the letter, however, drawing Azucena to write Citibank
for the cancellation of the cards. Only after Azucena's counsel informed Citibank of imminent
legal remedies on her part did Citibank indulge Azucena with a written explanation why her
credit cards were not honored in Italy.

Procedural History
Azucena later filed a complaint for damages. After an exchange of pleadings by the parties,
and the issues having been joined, the trial court set the case for pre-trial conference,
during which the spouses and their counsel failed to appear, despite notice. =

Statement of the Issue

Whether or not the excuse of the petitioners is a valid cause under Rule 18 of the Rules of
Court.

Holding

NO. The Court held that while Section 4 of Rule 18 of the Rules of Court allows as an
exception a valid cause for the non-appearance of a party at the pre-trial, the instances cited
by the spouses and their counsel hardly constitute compelling exigencies or situations which
warrant occasional flexibility of litigation rules. In Quelnan v. VHF Philippines where the
counsel for the therein petitioner failed to calendar a scheduled pre-trial in his diary, the
Court held that: The alleged failure of petitioner's counsel to record the scheduled pre-trial in
his 1997 diary to justify his absence at the pre-trial cannot amount to excusable
negligence. To constitute excusable negligence, the absence must be due to petitioner's
counsel's failure to take the proper steps at the proper time, not in consequence of his
carelessness, inattention or willful disregard of the process of the court, but in consequence
of some unexpected or unavoidable hindrance or accident. Petitioner's counsel's failure to
record the date of pre-trial in his 1997 diary reflects his carelessness, his failure to heed his
responsibility of not neglecting a legal matter entrusted to him, xxx. Petitioner's counsel must
know that pre-trial is mandatory. Being mandatory, the trial court has discretion to declare a
party non-suited. Absent a showing of grave abuse in the trial court's exercise thereof, as in
the case at bar, appellate courts will not interfere.
87. Chan Kent v. Micarez

G.R. No. 185758, March 9, 2011

Facts of the Case

Petitioner’s parents were looking for a residential lot for petitioner and her spouse. As she
was married to an American citizen, she arranged to pay for the purchase price of the
residential lot and register in the names of Spouses Micarez, under an implied trust. A deed
of absolute sale was executed between Spouses Micarez and the owner. Petitioner sent the
money which was used for the payment of the lot. Later, she learned that the Spouses
Micarez sold the lot to Dionesio.

Procedural History

Petitioner filed a Complaint for recovery of real property before the RTC. After filing their
respective pre-trial briefs, and the issues in the case had been joined, the RTC explored the
possibility of an amicable settlement among the parties by ordering the referral of the case to
the Philippine Mediation Center.

Statement of the Issue

Whether the RTC erred in dismissing Civil Case No. 13-2007 due to the failure of petitioner's
duly authorized representative, Manalang, and her counsel to attend the mediation
proceedings under the provisions of A.M. No. 01-10-5-SC-PHILJA and 1997 Rules on Civil
Procedure.

Holding

Yes. Since mediation is part of Pre-Trial, the trial court shall impose the appropriate
sanction including but not limited to censure, reprimand, contempt and such other
sanctions as are provided under the Rules of Court for failure to appear for pre-trial, in
case any or both of the parties absent himself/themselves, or for abusive conduct during
mediation proceedings. The Court found the sanction too severe to be imposed on the
petitioner where the records of the case is devoid of evidence of willful or flagrant
disregard of the rules on mediation proceedings. There is no clear demonstration that the
absence of petitioner's representative during mediation proceedings on March 1, 2008 was
intended to perpetuate delay in the litigation of the case. Neither is it indicative of lack of
interest on the part of petitioner to enter into a possible amicable settlement of the case.|||
88. Aguilar v. LIghtbringers Credit Cooperative

GR No. 209605, 12 January 2015.

Facts of the Case


Three complaints for sum of money were separately filed by respondent Lightbringers Credit
Cooperative against petitioners. Petitioners were members of the cooperative who borrowed
funds. Petitioners say there was a discrepancy between the principal amount of the loans
which show that they never borrowed. They also claim that no interest could be had because
there was no written agreement as to it’s imposition.

Procedural History

On the scheduled pre-trial conference, only respondent and its counsel appeared.

Statement of the Issue

Whether or not the petitioners can have their evidence admitted despite being declared in
default, due to their absence during the pre-trial.

Holding

NO. The case of Philippine American Life & General Insurance Company v. Joseph
Enario discussed the difference between non-appearance of a defendant in a pre-trial
conference and the declaration of a defendant in default in the present Rules of Civil
Procedure. If the absent party is the plaintiff, then his case shall be dismissed. If it is the
defendant who fails to appear, then the plaintiff is allowed to present his evidence ex
parte and the court shall render judgment on the basis thereof. Thus, the plaintiff is given the
privilege to present his evidence without objection from the defendant, the likelihood being
that the court will decide in favor of the plaintiff, the defendant having forfeited the
opportunity to rebut or present his own evidence. In the case at bar, the petitioners failed to
attend the pre-trial conference set on August 25, 2009. They did not even give any excuse
for their non-appearance, manifestly ignoring the importance of the pre-trial stage. Thus, the
MCTC properly issued the Order, allowing respondent to present evidence ex parte.
89. Sps. Salvador v. Sps. Rabaja

GR No. 199990, 4 February 2015

Facts of the Case


There was a dispute wherein petitioner spouses as the sellers, the sellers’ agent, and the
buyers, respondents, were fighting over a parcel of land. What happened was that
respondent heard that petitioners were looking for buyers, so respondent made the initial
payment to the agent in front of the seller. The seller had a special power of attorney. The
next day, there was a contract to sell. Respondents made several payments. However,
petitioners said they did not receive any payment, and respondents received a notice to
vacate.

Procedural History

An action for ejectment, the complaint was filed before the MeTC. Ruled in favor of
petitioners. Brought to RTC. The pre-trial conference began but attempts to amicably settle
the case were unsuccessful. It was formally reset to February 4, 2005, but Spouses
Salvador and their counsel failed to attend. Consequently, the RTC issued the pre-trial
order declaring Spouses Salvador in default and allowing Spouses Rabaja to present their
evidence ex parte against Spouses Salvador and Gonzales to present evidence in her favor.

Statement of the Issue

Whether or not the Court of Appeals erred in not finding that the lower court gravely abused
its discretion in declaring petitioners in default and in depriving them of the opportunity to
cross-examine respondents Sps. Rabaja.

Holding

NO. Pre-trial is an answer to the clarion call for the speedy disposition of cases.
Although it was discretionary under the 1940 Rules of Court, it was made mandatory under
the 1964 Rules and the subsequent amendments in 1997. "The importance of pre-trial in
civil actions cannot be overemphasized." There is no dispute that Spouses Salvador and
their counsel failed to attend the pre-trial conference set on February 4, 2005 despite proper
notice. Spouses Salvador aver that their non-attendance was due to the fault of their counsel
as he forgot to update his calendar. This excuse smacks of carelessness, and indifference
to the pre-trial stage. It simply cannot be considered as a justifiable excuse by the Court. As
a result of their inattentiveness, Spouses Salvador could no longer present any evidence in
their favor. Spouses Rabaja, as plaintiffs, were properly allowed by the RTC to present
evidence ex parte against Spouses Salvador as defendants. Considering that Gonzales as
co-defendant was able to attend the pre-trial conference, she was allowed to present her
evidence. The RTC could only render judgment based on the evidence presented during the
trial.
90. Pagtalunan v. Tamayo

GR No. 54281, 19 March 1990

Facts of the Case


A case was filed with the Court of the First Instance of Bulacan by the respondent, who is
the Republic of The Philippines, for expropriation of a parcel of land. The CFI gave a writ of
posession placing the Republic in possession of the land,
Procedural History

On June 8, 1978, petitioners herein filed a supplemental motion for leave to intervene, with
complaint in intervention attached thereto, alleging that petitioner Celso Pagtalunan has
been the bona fide agricultural tenant of a portion of the land.

Statement of the Issue

Whether or not petitioners have the right to intervene in the expropriation proceedings
instituted by the State against private respondents as registered owner of the subject
property

Holding

NO. Intervention is not a matter of right but may be permitted by the courts when the
applicant shows facts which satisfy the requirements of the law authorizing intervention.
Under Section 2, Rule 12 of the Revised Rules of Court, what qualifies a person to intervene
is his possession of a legal interest in the matter in litigation, or in the success of either of the
parties, or an interest against both, or when he is so situated as to be adversely affected by
a distribution or other disposition of property in the custody of the court or an officer thereof.
In the present case, the State in the exercise of its sovereign power of eminent domain has
decided to expropriate the subject property for public use as a permanent site for the
Bulacan Area Shop of the Department of Public Works and Highways. On the other hand,
petitioners have not been issued an emancipation patent. |Petitioners cannot now
successfully argue that Celso Pagtalunan is legally entitled to a portion of the proceeds from
the expropriation proceedings corresponding to the value of the landholding.
91. Pulgar v. RT of Mauban, Quezon

GR No. 157583, 10 September 2014

Facts of the Case

34 tax declarations on the building and machinery in Mauban Plant were issued and
assessedfor a certain amount. Later, the QPL tried to pay first quarter installment but such
was rejected. Thus, QPL filed a complaint for consignation and damages before the RTC
against the province of Quezon.

Procedural History

Pulgar filed a Motion for Leave to Admit Answer-in-Intervention, Pulgar's motion was
initially granted and his Answer-in-Intervention was admitted. Declaring that QPL's
complaint essentially challenged the amount of the taxes assessed, the RTC ruled that it is
the Local Board of Assessment Appeals that had jurisdiction over the complaint.
Consequently, it also dismissed Pulgar's motion for intervention since with the dismissal of
the main case, the same had no leg to stand on.

Statement of the Issue

Whether or not the RTC erred in dismissing Pulgar's motion for intervention as a
consequence of the dismissal of the main case.

Holding

NO. Jurisdiction over an intervention is governed by jurisdiction over the main


action. Accordingly, an intervention presupposes the pendency of a suit in a court of
competent jurisdiction. The cessation of the principal litigation on jurisdictional grounds at
that means that Pulgar had, as a matter of course, lost his right to intervene.
92. Ang Kek Chen v. Bello

GR Nos. L-76344-46, 30 June 1988

Facts of the Case

Petitioner Ang was charged before the then Manila City, with the crimes of "Maltreatment,"
"Threats," and "Slight Physical Injuries," committed on December 1977.

Procedural History

After the prosecution had presented its evidence, Ang filed a Demurrer to Evidence which
was denied by the respondent court. Ang elevated the incident to the RTC. On appeal, the
Court of Appeals affirmed Regional Trial Court. Meanwhile, the then presiding judge of
MTC, where the cases were pending, was promoted to the RTC of Manila. As a
consequence, respondent judge, as officer-in-charge of the MTC, directed the return of the
case records to the Clerk of Court for "re-raffle." Petitioner, however, alleged that he
received the corresponding order after the cases had already been actually "re-raffled" and
assigned to respondent judge. Ang filed a motion to re-raffle the cases, which was denied.
The subsequent motion for reconsideration was likewise denied. Hence, the present
petition, alleging that Respondent judge committed grave abuse of discretion.
|||

Statement of the Issue

Whether or not the issue of alleged grave abuse of discretion in violation of Circular No. 7 of
this Court, regarding the manner of raffle of cases, not denied or explained by public
respondent, is a trivial one.||.

Holding

It is not. A violation or disregard of the Court's circular on how the raffle of cases should
be conducted is not to be countenanced. A party has the right to be heard by an impartial
and unbiased tribunal. LThe raffle of cases is of vital importance to the administration of
justice because it is intended to insure impartial adjudication of cases. By raffling the cases
public suspicion regarding assignment of cases to predetermined judges is obviated.
When the respondent judge conducted the raffle of the three criminal cases in question,
apparently in violation of the Court's Circular No. 7, he did not only arouse the suspicion
that he had some ulterior motive for doing so, but he violated the cardinal rule that all
judicial processes must be done above board.
|||
93. Universal Rubber Products, Inc. v. CA

GR No. L-30266, 29 June 1984

Facts of the Case


Petitioner was sued by two respondent corporations who claim unfair competition.

Procedural History

Private respondents made a request to the respondent Judge to issue a


subpoena duces tecum against the treasurer of herein petitioner. Acting favorably on
that request, said respondent Judge issued a subpoena duces tecum,Petitioner filed
a motion in the court below praying that the subpoena duces tecum dated be
quashed on the grounds that: (1) the said subpoena is both unreasonable and
oppressive as the books and documents called for are numerous and voluminous; (2)
there is no good cause shown for the issuance thereof; and (3) the books and
documents are not relevant to the case pending below. Acting on the said motion and
on the opposition thereto, respondent Judge denied the motion to quash the
subpoena duces tecum. ||

Statement of the Issue

Whether the issuance of the "subpoena duces tecum" is proper in a suit for unfair
competition.

Holding

YES. Based on jurisprudence, in order to entitle a party to the issuance of a


"subpoena duces tecum", it must appear, by clear and unequivocal proof, that the book or
document sought to be produced contains evidence relevant and material to the issue before
the court, and that the precise book, paper or document containing such evidence has been
so designated or described that it may be identified. A "subpoena duces tecum" once issued
by the court may be quashed upon motion if the issuance thereof is unreasonable and
oppressive, or the relevancy of the books, documents or things does not appear, or if the
persons in whose behalf the subpoena is issued fails to advance the reasonable cost of
production thereof. The argument that the petitioner should first be found guilty of unfair
competition before an accounting for purposes of ascertaining the amount of damages
recoverable can proceed, stands without merit. The complaint for unfair competition is
basically a suit for "injunction and damages. Hence, the collection of the complainant for the
accounting of petitioner's gross sales as damages per R.A. 166, appears most relevant. For
Us, to determine the amount of damages allowable after the final determination of the unfair
labor case would not only render nugatory the rights of complainant under Sec. 23 of R.A.
166, but would be a repetitious process causing only unnecessary delay.
94. People v. Montejo

GR No. L-24154, 31 October 1967

Facts of the Case


A petition was alleged by the City Fiscal of Zamboanga against Felix Wee Sit for double
homicide and serious physical injuries through reckless imprudence. It was claimed that
Ernesto Salvador, a resident of Rizal, was a material and important witness.

Procedural History
It was alleged that at the time accused was called for trial in the Court of First Instance of
Zamboanga City, pursuant to a formal request of the City Fiscal, respondent Judge issued
a subpoena to patrolman Uaje. But when the case was called for continuation, he did not
appear "and forthwith the undersigned City Fiscal formally moved for an order of arrest" or
in the alternative "to cite him for contempt for willful failure to appear at the trial of the case
as a material witness.” Such a motion was presented and denied on the same day by the
respondent Judge in the Order sought to be annulled in this petition. In their answer of the
Respondents, they denied that of the allegation that Uaje was "a material, much less an
important, witness,".

Statement of the Issue

Whether or not a Court of First Instance possesses authority on a criminal case "to compel
by subpoena the attendance of the witness who, as in this case, resides hundreds of miles
away from the place of trial."|||

Holding

Yes. It was argued that under the Rules of Court, a witness is not bound to attend a
hearing if held outside the province he resides unless the distance be less than 50
kilometers from his residence to the place of trial. Such contention did not command the
assent of this Court. Thus: "It is loathe to clip what undoubtedly is the inherent power of the
Court to compel the attendance of persons to testify in a case pending therein. Section 9 of
Rule 23 is thus interpreted to apply solely to civil cases. A recognition of such power in a
court of first instance conducting the trial of an accused may be gleaned from principle that
justifies it when satisfied 'by proof or oath, that there is reason to believe that a material
witness for the prosecution will not appear and testify when required,' to order that he 'give
bail in sum as may deem proper for such appearance. Upon refusal to give bail, the court
must commit him to prison until he complies or is legally discharged.
|||
95. Genorga v. Quintain

Adm. Matter No. 981-CFI (78 SCRA 94), 29 July 1977

Facts of the Case

Dr Gil Genorga, the former municipal health officer of Masbate had a warrant of arrest
against him as he did not appear as medico-legal witness in a pending murder case.
Accused says that he sent a letter to respondent judge asking if he would be reimbursed but
got nor reply. Thus, he suffered humiliation from being arrested.

Procedural History

He filed an administrative complaint for grave abuse of authority and conduct unbecoming a
judge.against Judge Quintain..

Statement of the Issue

Whether a court of first instance hearing a criminal case may compel by subpoena the
attendance of a witness in his sala. ||.

Holding

It was argued that under the Rules of Court, a witness is not bound to attend a hearing if
held outside the province he resides unless the distance be less than 50 kilometers from his
residence to the place of trial. Such contention did not command the assent of this Court. It
would follow then that respondent Judge should decide said motion without taking into
consideration Section 9 of Rule 23. What was done by Judge Quitain was, therefore, within
his discretion.|||
96. Luz v. National Amnesty Commission

GR No. 159708, 24 September 2004

Facts of the Case

The petitioner was charged with violation of illegal possession of firearms in the
Regional Trial Court of Makati City.
|

Procedural History

On March 22, 2000, the petitioner filed an application for amnesty with the Local
Amnesty Board for Metro Manila. In due course, the board denied the said application.
Under Administrative Order No. 2, the petitioner had until December 7, 2002, a Saturday,
within which to file a petition for review of the said resolution with the Court of Appeals. On
December 9, 2002, the petitioner filed a motion in the appellate court for an extension of
fifteen (15) days from December 9, 2002, or until December 24, 2002 within which to file his
petition. The petitioner alleged therein that he had just engaged the services of counsel who
needed additional time to study the case and draft the petition.

Statement of the Issue

Whether or not the petitioner timely filed his second motion for extension of time to file his
petition for review|.|

Holding

YES.
The Court Resolves, for the guidance of the Bench and the Bar, to declare
that Section 1, Rule 22 speaks only of “the last day of the period” so that when a
party seeks an extension and the same is granted, the due date ceases to be the last
day and hence, the provision no longer applies. Any extension of time to file the
required pleading should therefore be counted from the expiration of the period
regardless of the fact that said due date is a Saturday, Sunday or legal holiday.
The extension granted by the Court of Appeals should be tacked to the original
period and commences immediately after the expiration of such period. Under the
Resolution of this Court in A.M. No. 00-2-14-SC, the CA has no discretion to reckon the
commencement of the extension it granted from a date later than the expiration of such
period, regardless of the fact that said due date is a Saturday, Sunday, or a legal holiday.
97. Neypes v. CA

GR No. 141524, 14 September 2005

Facts of the Case

An action for annulment of judgement and titles of land were filed before the RTC of Oriental
Mindoro against the Bureau of Forest Development. In the course of the proceedings, the
parties filed various motions with the trial court. Among these were: (1) the motion filed by
petitioners to declare the respondent heirs, the Bureau of Lands and the Bureau of Forest
Development in default and (2) the motions to dismiss filed by the respondent heirs and the
Land Bank of the Philippines, respectively.
|||

Procedural History

The trial court, presided by public respondent Judge Antonio N. Rosales, resolved
that the petitioners' motion to declare respondents Bureau of Lands and Bureau of Forest
Development in default was granted for their failure to file an answer, but denied as against
the respondent heirs of del Mundo because the substituted service of summons on them
was improper.

Statement of the Issue

Whether or not the aggrieved party is given a fresh reglementary period of 15 days to file an
appeal counted from the receipt of the denial of the motion for reconsideration or new trial.

Holding

Yes.

Henceforth, this "fresh period rule" shall also apply to Rule 40 governing appeals from the
Municipal Trial Courts to the Regional Trial Courts; Rule 42 on petitions for review from the
Regional Trial Courts to the Court of Appeals; Rule 43 on appeals from quasi-judicial
agencies to the Court of Appeals and Rule 45 governing appeals by certiorari to the
Supreme Court.

The new rule aims to regiment or make the appeal period uniform, to be counted from
receipt of the order denying the motion for new trial, motion for reconsideration (whether full
or partial) or any final order or resolution. The Court thus hold that petitioners seasonably
filed their notice of appeal within the fresh period of 15 days, counted from July 22, 1998, the
date of the receipt of Motion for Reconsideration.
99. Vda. de Manguerra v. Risos

GR No. 152643, 28 August 2008

Facts of the Case

Respondents were charged with Estafa through Falsification of public document. The
Criminal case arose from the falsification of a deed of real estate mortgage allegedly
committed by respondents where they made it appear that Concepcion, the owner of the
mortgaged property known as the Gorordo property, affixed her signature to the document.
Hence, the criminal case. Respondents filed a Motion for Suspension of the Proceedings
in Criminal Case on the ground of prejudicial question. The RTC granted the aforesaid
motion. Concepcion's motion for reconsideration was denied on June 5, 2000.
|||

Procedural History
Concepcion instituted a special civil action for certiorari before the CA seeking the
nullification of RTC orders. The counsel of Concepcion filed a motion to take the latter's
deposition. RTC granted the motion. The respondents' motion for reconsideration was
denied by the trial court on November 3, 2000. The court ratiocinated that procedural
technicalities should be brushed aside because of the urgency of the situation, since
Concepcion was already of advanced age. After several motions for change of venue of
the deposition-taking, Concepcion's deposition was finally taken on March 9, 2001 at her
residence.

Statement of the Issue

Whether or not Rule 23 of the 1997 Rules of Civil Procedure applies to the Deposition of
Petitioner.

Holding

NO. In criminal cases, the accused may be afforded the opportunity to cross-examine
the witnesses pursuant to his constitutional right to confront the witnesses face to face. It
also gives the parties and their counsel the chance to propound such questions as they
deem material and necessary to support their position or to test the credibility of said
witnesses. Lastly, this rule enables the judge to observe the witnesses' demeanor. As
exceptions, Rules 23 to 28 of the Rules of Court provide for the different modes of
discovery that may be resorted to by a party to an action. These rules are adopted either to
perpetuate the testimonies of witnesses or as modes of discovery. In the case at bench, in
issue is the examination of a prosecution witness, who, according to the petitioners, was
too sick to travel and appear before the trial court.
100. AM No. 15-06-10-SC, RE: Adopting the Guidelines for Continuous Trial of
Criminal Cases in Pilot Courts, 30 June 2015.

101. Lee v. Romillo, Jr.,

GR No. L-60937, 28 May 1988.

Facts of the Case


Private respondent filed a complaint for accounting with damages. There was a contract for
arrastre services in Manila between respondent and William Lines. In the year 1973, plaintiff
Mendioro conducted arrastre operations as per arrastre contract with William Lines, Inc, that
all the income from the arrastre operations during the said period were received and held in
trust for Mendioro by defendants Lee, Tan and Chiongbian who were also tasked with the
disbursement of the said income for operational expenses. Defendants Lee, Tan and
Chiongbian were duty-bound to turn over to plaintiff Mendioro the income they held in trust
for him; and they failed to do so despite Mendioro's repeated demands.|||

Procedural History

Defendants filed a motion to dismiss the complaint. The motion was denied. The trial court
granted the motion to include .At the pre-trial stage of the proceedings, the parties filed a
partial stipulation of facts.During the hearing on where the counsel for the petitioners failed
to show up, Mendioro was allowed to present evidence ex-parte. An amended order of the
trial court granted the petitioners an opportunity to adduce their evidence. Lengthy trials
were conducted and The Trial Court decided against the Petitioners.

Statement of the Issue


Whether or not the delegation of the reception of the evidence ex-parte to the lower court's
legal researcher is illegal.|||

Holding

In the case of The National Housing Authority v.Court of Appeals, the Court categorically
stated that: "The contention that the Trial Court cannot delegate the reception of evidence
to its Clerk of Court, citing the case of Lim Tanhu v. Ramolete is not well taken. Suffice it to
say, for purposes of this suit, that the said case referred to reception of evidence by a
Clerk of Court after declaration of defendant's default. No default is involved herein. As
held in the case of Laluan v. Manalo, no provision of law or principle of public policy
prohibits a Court from authorizing its Clerk of Court to receive the evidence of a party
litigant.||
102. Metropolitan Bank and Trust Company v. Sandoval

GR No. 169677, 18 February 2013

Facts of the Case


A complaint was filed against the Marcoses for ill-gotten wealth. Later, the Republic moved
for the amendment of the complaint to implead Asian Bank as an additional defendant. This
was granted.
|||

Procedural History

When the Republic was about to terminate its presentation of evidence against the original
defendants in Civil Case No. 0004, it moved to hold a separate trial against Asian Bank.
Asian Bank sought the deferment of any action on the motion until it was first given the
opportunity to test and assail the testimonial and documentary evidence the Republic had
already presented against the original defendants. In its reply to Asian Bank's comment, the
Republic maintained that a separate trial for Asian Bank was proper.
Statement of the Issue

Whether or not holding of a separate trial would deny Metro bank due process, because
Asian Bank was entitled to contest the evidence of the Republic against the original
defendants prior to Asian Bank's inclusion as an additional defendant.|||

Holding

The Trial Court held the Separate Trials to be improper. The Court held that
Sandiganbayan committed grave abuse of its discretion in ordering a separate trial as to
Asian Bank (Metrobank) on the ground that the issue against Asian Bank was distinct and
separate from that against the original defendants. Thereby, the Sandiganbayan veered
away from the general rule of having all the issues in every case tried at one time,
unreasonably shunting aside the dictum in Corrigan, supra, that a "single trial will generally
lessen the delay, expense, and inconvenience to the parties and the courts." Exceptions to
the general rule are permitted only when there are extraordinary grounds for conducting
separate trials on different issues raised in the same case, or when separate trials of the
issues will avoid prejudice, or when separate trials of the issues will further convenience, or
when separate trials of the issues will promote justice, or when separate trials of the issues
will give a fair trial to all parties. Otherwise, the general rule must apply.

|||
103. Casent Realty Development Corp. v. Philbanking Corp., GR No. 150731,
14 September 2007.

Facts of the Case:

Two promissory notes were issued by petitioner to Rare reality, which were assigned to
respondent by a Deed of Assignment. Petitioner did not pay, despite demand.

Procedural History:

A complaint before the Makati City RTC was filed by respondent. The parties failed
to reach an amicable settlement during the pre-trial conference. The respondent
presented its evidence and exhibits. Petitioner then filed a Motion for Judgment on
Demurrer to the Evidence, claiming that the plaintiff’s failure to file a Reply to the
Answer which raised the Dacion and Confirmation Statement constituted an
admission of the genuiness and execution of said documents. Trial court ruled in
favor of the petitioner. Appellate court ruled that the trial court erred when it
considered the Answer.

Statement of Issue:

Did CA err in excluding the petitioner’s affirmative defenses in its answer in resolving
a demurrer to evidence.

Held:

Rule 33, Section 1 of the 1997 Rules of Civil Procedure: “the defendant may move
for dismissal on the ground that upon the facts and the law the plaintiff has shown no
right to relief. The evidence contemplated by the rule on demurrer is that which
pertains to the merits of the case, excluding technical aspects such as capacity to
sue. The “facts” referred to in Section 8 should include all the means sanctioned by
the Rules of Court in ascertaining matters in judicial proceedings. Petitioner, by way
of defense, should have presented evidence to show that the Dacion includes the
promissory notes.
104. Uy v. Chua, GR No. 183965, 18 September 2009.

Facts of the Case:

Respondent had an illegal relationship with Irene and sired two children. The birth
certificates of the children were filed out with the names or Irene’s father and her
mother’s maiden name. Respondent financially supported petitioner

Procedural History:

Petitioner filed before the RTC a petition for the issuance of a decree of illegitimate
filiation. Hearings then ensued, petitioner presented documentary evidence to prove
her claim of illegitimate filiation wherein the respondent filed a Demurrer to Evidence
on the ground that the Decision in Spec. Proceeding No. 8830-CEB, a prior
proceeding where petitioner had already filed a similar Petition for the issuance of a
decree of illegitimate affiliation against respondent which a Compromise Agreement
was filed by the parties, had already been barred by res judicata. The trial court
allowed respondent’s Demurrer and denied petitioner’s Motion for Reconsideration.

Statement of Issue:

Was the trial court correct in granting the demurrer to evidence?

Held:

Demurrer to Evidence is governed by Rule 33 of the Rules of Court, Section 1 provides that:
“After the plaintiff has completed the presentation of his evidence, the defendant may move
for dismissal on the ground that upon the facts and the law the plaintiff has shown no right to
relief. If his motion is denied, he shall have the right to present evidence. If the motion is
granted but on appeal the order of dismissal is reversed he shall be deemed to have waived
the right to present evidence.” Demurrer is an aid or instrument for the expeditious
termination of an action, similar to a motion to dismiss, which the court or tribunal may either
grant or deny. Necessarily, the last line of Section 1, Rule 33 should not apply herein and
respondent should still be allowed to present evidence before the RTC for Special
Proceedings No. 12562-CEB. Thus, it was incumbent upon RTC to make the determination.
It did not take into consideration any of the evidence presented by petitioner.
105. Wood Technology Corp. v. Equitable Bank, GR No. 153867, 17
February 2005.

Facts of the Case:


WTC got a loan from respondent, signed by Cordova and Young, which bound
themselves as surities. When demand was made by respondent, petitioners did not
pay. They claim that only one demand letter was made. Plus, the loan had not
matured yet, as the maturity date was blank.

Procedural History:

A complaint for sum of money was then filed. Petitioners prayed for the reformation
of the promissory note and surety agreement while the respondent moved for a
judgment on the pleadings wherein the RTC rendered judgment against the
petitioners. Petitioner appealed but the CA affirmed the RTC’s judgment.

Statement of Issue:

Whether or not the appellate court made a mistake when it allowed RTCs judgement
on the pleadings

Held:

“In a proper case for judgment on the pleadings, there is no ostensible issue at all
because of the failure of the defending party’s answer to raise an issue. On the other
hand, in the case of a summary judgment, issues apparently exist.”
The judgment rendered by the trial court is valid as a summary judgment. A
judgment on the pleadings is proper when an answer fails to tender an issue, or
otherwise admits the material allegation of the adverse party’s pleadings. We agree
with both the trial and the appellate courts that this matter proferred as a defense
could be resolved judiciously by plain resort to the stipulations in the promissory note
which was already before the trial court.
106. Municipality of Tiwi v. Betito, GR No. 171873, 9 July 2010.

Facts of the Case:

The NPC was found liable for unpaid real estate taxes. This included geothermal
plants in the Municipality of Tiwi with substations in Albay (which were sold in an
auction sale). In the sale, the Municipality obtained ownership over the properties.
An agreement was entered into between BPC and Albay where NPC agreed to settle
it’s tax liabilities.The mayor requested to remit the rightful tax share of Tiwi and its
barangays. The governor stated that the request cannot be granted. So NPC
requested clarification from the OP.

Procedural History:

Respondent filed a motion for partial judgement on the pleadings. Trial court
rendered such, as petitioners answer to the complaint did not tender an issue. The
Court of Appeals affirmed the decision of the trial court.

Statement of Issue:

If the application of the rule of judgment on the pleadings and/or summary judgment
is baseless, improper and unwarranted

Held:

a review of the records reveal that respondent and petitioners set-up multiple levels of claims
and defenses, respectively with some failing to tender an issue while others requiring the
presentation of evidence for resolution. A motion for judgment on the pleadings admits the
truth of all material and relevant allegations of the opposing party. The judgment must rest
on those allegations taken together with such other allegations as are admitted in the
pleadings. It is proper when an answer fails to tender an issue, or otherwise admits the
material allegation of the adverse party’s pleading. If it appears that not all the material
allegations of the complaint were admitted in the answer for some of them were either
denied or disputed, then the defendant has set up certain special defenses. Those, if proven,
would have the effect of nullifying plaintiff's main cause of action, judgment on the pleadings
cannot be rendered.
107. Tan v. Dela Vega, GR No. 168809, 10 March 2006.

Facts of the Case:

A complaint for quieting of title and the declaration of nullity of free patent was filed
by respondents against the heirs of Mencias and the Secretary of DENR. Petitioner
was later impleaded as purchasers of the disputed lot in Rizal.

Procedural History:

For faliure to file their answer petitioners were declared in default. Respondents filed a
motion for judgment on the pleadings which was granted by the trial court. Hence, the titles
derived or issued on the basis of said Free Patent are void because the Public Land Act
applies only to public lands and not private lands. On the theory that the spring cannot rise
higher that its sources, the trial court concluded that petitioners cannot be purchases in good
faith considering that their title was derived from Macario who acquired the property by virtue
of a void title. It further ruled that petitioners' defense of good faith must fail because they
were forewarned of the notice indicating that the questioned lot is inside. Petitioners
appealed to the CA which affirmed the assailed order of the trial court. They filed a motion
for reconsideration but was denied in a resolution. Hence, this petition.

Statement of Issue:

Whether or not a judgment on the pleadings is proper in the instant case.

Held:

In the case at bar, we find that the trial court erred in rendering judgment on the
pleadings because the pleadings filed by the parties generated ostensible issues that
necessitate the presentation of evidence in order to determine the validity of the titles
from which the respondent’s title was derived. In a proper case for judgment on the
pleadings, there is no ostensible issue at all because of the failure of the defending
party’s answer to raise an issue.
108. Adolfo v. Adolfo, GR No. 201427, 18 March 2015.

Facts of the Case:


A petitioner for judicial separation of property was filed in the RTC by Adolfo against
his wife. Allegedly, the parties were married and they had one child. The acquired a
lot in Mandaue during marriage. The parties then separated because of
irreconcilable differences. Petitioner then suggested separation of conjugal property,
but respondent refused. Respondent contended that she is the sole owner of the subject
property which she inherited from her mother; that petitioner is a lazy bum, gambler,
drunkard, wife abuser, and neglectful father; that respondent found all means to support the
family. She caused to be built on a portion of her mother's land a house even while petitioner
was bumming around; that one day, petitioner destroyed the roof of the house that was then
being built; that petitioner subsequently abandoned her and their child in 1968, and
transferred to Davao City where he took a mistress and begot four children by her.

Procedural History:

The trial court rendered in its decision that the subject property constituted conjugal property.
An appeal was filed with the CA. meanwhile, during the pre-trial conference, petitioner filed a
Request for Admission of 1) the genuineness of the duly marked certified true copies of the
Complaint, Answer; 2) respondent's declaration in said Answer that the subject property
constituted conjugal property of the marriage; and 3) the trial court's pronouncement in said
case that the subject property forms part of the conjugal estate. A Motion for Judgment
Based on the Pleadings was filed, stating that since respondent failed to answer his request
for admission, the matters contained in the request are deemed admitted pursuant to Rule
26, Section 2 of the 1997 Rules of Civil Procedure (1997 Rules); that as a consequence of
the application of the rule, respondent is in effect considered to have admitted that the
subject property is a conjugal asset of their subsisting marriage . Branch 55 issued an Order
granting petitioner's motion for judgment on the pleadings. It held that plaintiff is entitled to
the relief demanded. The CA rendered its decision, which reversed the Decision of the trial
court and declared that, among others, that the subject property was respondent’s
paraphernal property.

Statement of Issue:

Whether or not the CA erred in deciding the case on a question of substance not in
accord with law, Rule 26 of the 1997 Rules, and applicable jurisprudence.

Held:

Instead of resolving petitioner's motion for judgment on the pleadings, the trial court should
have denied it or held it in abeyance. Judgment on the pleadings is proper "where an answer
fails to tender an issue, or otherwise admits the material allegations of the adverse party's
pleading." Summary judgment, on the other hand, will be granted "if the pleadings,
supporting affidavits, depositions, and admissions on file, show that, except as to the amount
of damages, there is no genuine issue as to any material fact and that the moving party is
entitled to a judgment as a matter of law." If the answer does in fact specifically deny the
material averments of the complaint and/or asserts affirmative defenses, a judgment on the
pleadings would naturally be improper. In a case for judgment on the pleadings, the Answer
is such that no issue is raised at all. The essential question in such a case is whether there
are issues generated by the pleadings. The trial court however disregarded the fact that its
decision was then the subject of a pending appeal. One of the issues raised in the appeal is
precisely whether the subject property is conjugal, or a paraphernal asset of the respondent.
109. Calubaquib v. Republic, GR No. 170658, 22 June 2011.

Facts of the Case:

A landholding in Cagayan was declared by President Manuel L Quezon as a military


reservation site, under Proclamation No. 80. It is subject to private rights, if any there be.
Supposedly, petitioners unlawfully entered the military reservation and demands were made
for them to leave. Petitioners claim that the land is theirs.

Procedural History:

Respondent filed before the RTC of Tuguegarao, Cagayan a complaint for recovery of
possession against petitioners. Without any trial, the trial court rendered a decision
dismissing petitioners' claim of possession of the subject property in the concept of owner.
The trial court held that while Proclamation No. 80 recognized and respected the existence
of private rights on the military reservation, petitioners' position could "not be sustained, as
there was no right of [petitioners] to speak of that was recognized by the government." The
CA affirmed the RTC Decision.

Statement of Issue:

Whether or not a propriety of rendering a summary judgment is proper in this case.

Held:

The guidelines for summary judgement was ignored by the trial court. Summary judgment is
proper when, upon motion of the plaintiff or the defendant, the court finds that the answer
filed by the defendant does not tender a genuine issue as to any material fact and that one
party is entitled to a judgment as a matter of law. For this case, the trial court proceeded to
render summary judgment with neither of the parties filing a motion therefor it was improper
for the trial court to have persisted in rendering summary judgment.
110. YKR Corp. v. Philippine Agri-Business Center Corp., GR No. 191838, 20
October 2014.

Facts of the Case:

Yulo died. After his death, his late wife Teresa J Yulo and their six children became
substitutes. His wife then died. The petitioners have three other siblings who claim
that they "have gone their own separate way[s] when Luis A. Yulo died. [p]etitioners have
no knowledge or information sufficient to form a belief if their siblings are aware of the
proceedings in (Sandiganbayan), including respondent's move for a summary judgment and
the assailed resolutions of the Sandiganbayan."

Procedural History:

Respondent PABC, a domestic corporation, is a plaintiff-in-intervention which sought, among


others, that the Sandiganbayan call it as the true and lawful owner of a real property known
as the "Yulo King Ranch" located in Busuanga, Palawan, and order petitioner Republic of
the Philippines (Republic) to lift the sequestration and return possession of the subject
property to said respondent. The Sandiganbayan issued the assailed Resolutions in its favor.
Petitioners YKR Corporation and then seven out of the ten Yulo heirs responded to the
Request for Admissions by making no categorical admission or denial of the matters set
forth in the Request for Admissions allegedly because all the records of YKR
Corporation have been taken by the PCGG when they were sequestered. Court held that
the matters requested for admission “ought to be within the “personal knowledge” of YKR
Corporation and seven out of the ten Yulo Heirs.

Statement of Issue:

Whether or not the Sandiganbayan erred in law when it granted the respondent’s
motion for summary judgment even if the same is not supported by deposition,
affidavit or admission on the records.

Held:

A prudent examination of the evidence on record yields to no other conclusion that there
exists a genuine issue of fact as raised in both petitions. In ruling on the issue of whether a
genuine issue of fact exists, there was no mention of any circumstance or situation upon
which the court a quo derived its conclusion that the matters requested for admission "ought
to be within the personal knowledge" of YKR Corporation and seven out of the ten Yulo
Heirs. We cannot thus properly ascertain whether the facts which the latter could not make
any truthful admission or denial are so plainly and necessarily within their knowledge.
111. Sumbilla v. Matrix Finance Corp., GR No. 197582, 29 June 2015.

Facts of the Case:

Petitioner issued Philippine Business Bank Checks as payment for her loan.
However, the checks were dishonored having been drawn against a closed account.

Procedural History:

MeTC found petitioner criminally and civilly liable for the issuance of the six rubber checks.
Instead of filing a Notice of Appeal, petitioner filed a Motion for Reconsideration. This was
denied being a pleading barred under the Revised Rules on Summary Procedure. With the
denial of her Motion for Reconsideration, petitioner filed a petition for certiorari under Rule 65
of the Rules. The RTC dismissed petition of certiorari and its motion for reconsideration,
ruling that the MeTC did not act with grave abuse of discretion in denying the Notice of
Appeal filedl by petitioner. Petitioner elevated the case to the CA.

Statement of Issue:

Whether or not the penalty imposed in the MeTC Decision, which is already final and
executory, may still be modified.

Held:

Unfortunately, in the present case, the MeTC Decision is already final and executory after
petitioner failed to timely file a Notice of Appeal. Under the doctrine of finality and
immutability of judgments, a decision that has acquired finality becomes immutable and
unalterable and may no longer be modified in any respect, even if the modification is meant
to correct erroneous conclusions of fact or law, and whether it will be made by the court that
rendered it or by the highest court of the land. Upon finality of the judgment, the Court loses
its jurisdiction to amend, modify or alter the same. Nonetheless, the Court has the power and
prerogative to suspend its own rules and to exempt a case from their operation if and when
justice requires it. Substantial justice dictates that the penalty meted on the petitioner be
accordingly corrected within the maximum limits prescribed under Section 1 of BP 22.
112. Delfino v. Anasao, GR No. 197486, 10 September 2014.

Facts of the Case:

Delfino owned several parcels of agricultural land in Laguna. He sold some coconut
land, leaving him with rice land. The tenanted portion was tilled by respondents, and
placed under Operation land transfer.

Procedural History:

Delfino filed an Application for Retention. What was issued was an order denying the
retention of the 9.8597 has but granting the 4.8120 has which was not covered by OLT.
Delfino appealed to the DAR Secretary Respondents appealed to the OP but later withdrew
the appeal and instead filed a petition for review in the CA. CA reversed the OP's ruling and
reinstated the Orders of Secretary Pangandaman. DAR Secretary is not precluded from
making the necessary amendments/clarifications thereof so that the fallo would at least
conform with the body of said order and so that the same could readily be executed with
dispatch.

Statement of Issue:

Whether or not the Order of Secretary Pangandaman violated the rule on


immutability of final judgments.

Held:

The doctrine of finality and immutability of judgments is grounded on fundamental


considerations of public policy and sound practice to the effect that, at the risk of occasional
error, the judgments of the courts must become final at some definite date set by law. There
are exceptions to the general rule, namely: (1) the correction of clerical errors; (2) the so-
called nunc pro tunc entries which cause no prejudice to any party; (3) void judgments; and
(4) whenever circumstances transpire after the finality of the decision rendering its execution
unjust and inequitable. The clarification made by Secretary Pangandaman falls under the
fourth exception.
113. Club Filipino, Inc. v. Bautista, GR No. 168406, 14 January 2015.

Facts of the Case:

The employees of Club Filipino Inc were represented by a union, CLUFEA. There
were bargaining agreements between the two, and CLUFEA made several demands
for a new agreement, which was denied. CLUFEA filed before the NCMB a request for
preventive mediation. The meeting ended with the parties' respective panels declaring a
deadlock in negotiation. CLUFEA filed with the NCMB a Notice of Strike on the ground of
bargaining deadlock. CLUFEA conducted a strike vote under the Department of Labor and
Employment's supervision with the majority of CLUFEA's total union membership voting to
strike. CLUFEA staged a strike on the ground of bargaining deadlock.

Procedural History:

Club Filipino, Inc. filed before the NLRC a Petition to Declare CLUFEA's Strike illegal. Labor
Arbiter declared CLUFEA's strike "procedurally infirm for CLUFEA's failure to comply with
the procedural requirements for staging a strike. The Labor Arbiter declared the strike illegal
and considered "all the officers of the union . . . terminated from service." CLUFEA appealed
the Labor Arbiter's Decision before the NLRC and ruled that CLUFEA's Appeal was filed by
persons "[having] no legal standing to question the [Labor Arbiter's] decision."

Statement of Issue:

Whether or not Club Filipino, Inc.’s filing of the Supplemental Motion for
Reconsideration prevented the Resolution from becoming final and executory.

Held:

As a general rule, the filing of second Motions for Reconsideration of a judgment or final
resolution is prohibited as provided by Rule 52, Section 2 of the Rules of Court. The
petitioner’s Supplemental Motion for Reconsideration is in the nature of a second Motion for
Reconsideration. For this court to entertain second Motions for Reconsideration, the second
Motions must present "extraordinarily persuasive reasons and only upon express leave first
obtained." Once leave to file is granted, the second Motion for Reconsideration is no longer
prohibited This case falls under the exception, . In the present case, this court granted leave
to petitioner Club Filipino, Inc. to file the Supplemental Motion for Reconsideration in the
Resolution. The Supplemental Motion for Reconsideration, therefore, is no longer prohibited.
114. NTC v. Aplhaomega Integrated Corp., GR No. 184295, 30 July 2014.

Facts of the Case:

Public biddings were conducted by TRANSCO. AIC was licensed transmission line
contractor, and was part of the biddings. They were awarded 6 government
construction projects. However, they encountered difficulties and incurred losses
because TRANSCO breached contract, so they gave up the projects to TRANSCO.
AIC, a duly licensed transmission line contractor, participated in the public biddings
conducted by TRANSCO and was awarded six (6) government construction projects and
encountered difficulties and incurred losses allegedly due to TRANSCO's breach of their
contracts, prompting it to surrender the projects to TRANSCO under protest. Charged
against TRANSCO: breached the contracts by its failure to: (a) furnish the required Detailed
Engineering; (b) arrange a well-established right-of-way to the project areas; (c) secure the
necessary permits and clearances from the concerned local government units (LGUs); (d)
ensure a continuous supply of construction materials; and (e) carry out AIC's requests for
power shut down.

Procedural History:

AIC submitted a request for arbitration before the CIAC. Amended Complaint against
TRANSCO CIAC Arbitral Tribunal rendered its Final Award was filed. CA affirmed the
Arbitral Tribunal's factual findings that TRANSCO failed to exercise due diligence in
resolving the problems regarding the right-of-way and the lack of materials before
undertaking the bidding process and entering into the contracts with AIC.

Statement of Issue:

Whether or not the CA erred in affirming the CIAC Arbitral Tribunal's findings that AIC was
entitled to its claims for damages as a result of project delays.

Held:

The Court finds no reason to disturb the factual findings of the CIAC Arbitral Tribunal on the
matter of AIC's entitlement to damages which the CA affirmed as being well supported by
evidence and properly referred to in the record. Section 1, Rule 45 of the Rules of Court
provides that a petition for review on certiorari under the said rule, as in this case, "shall
raise only questions of law which must be distinctly set forth.” It is well-settled that findings of
fact of quasi-judicial bodies, which have acquired expertise because their jurisdiction is
confined to specific matters, are generally accorded not only respect, but also finality,
especially when affirmed by the CA. The CIAC possesses that required expertise in the field
of construction arbitration and the factual findings of its construction arbitrators are final and
conclusive, not reviewable by this Court on appeal. Indeed, the rule is well entrenched:
Specialis derogat generali. When two rules apply to a particular case, that which was
specially designed for the said case must prevail over the other.
115. DBP v. Guarina Agricultural and Realty Development Corp., GR No.
160758, 15 January 2014.

Facts of the Case:

DBP gave a loan to GC to finance the development of its resort complex in Iloilo, so
GC executed a promissory note and real estate mortgage over several real
properties as security, plus a chattel mortgage. The loan was released in several
installments. GC used to proceeds to defray costs of improvements, and demanded
balance on the loan but DBP refused. DBP paid some supplies of GC without it’s
approval. It was found by DBP that GC had not completed the construction works, so
DBP demanded that GC expedite the completion of the project. Not happy with the
non-action, DBP initiated extrajudicial foreclosure proceedings. A notice of
foreclosure sale was sent to GC, which was published leading the clients and
patrons of GC to think that the resort had closed.

Procedural History:

GC brought the case in the RTC to demand specific performance and to stop the
foreclosure of the mortgages. DBP moved for the dismissal of the complaint. GC
amended the complaint to seek the nullification of the foreclosure proceedings. DBP
filed its answer and trial followed upon the termination of the pre-trial without any
agreement being reached by the parties. In the meantime, DBP applied for the
issuance of a writ of possession by the RTC. Aggrieved, GC assailed the granting of
the application before the CA on certiorair. After the CA dismissed the petition for
certiorari, DBP sought the implementation of the order for the issuance of the writ of
possession. RTC issued the writ of possession and rendered its judgment.The CA
sustained the RTC’s judgment. DBP timely filed a motion for reconsideration, but the
CA denied its motion. Hence, this appeal by DBP.

Statement of Issue:

Whether or not the Decision of the CA and its resolution denying petitioner’s Motion
for Reconsideration were issued in accordance with law.

Held:

DBP's actuations were legally unfounded. It is true that loans are often secured by a
mortgage constituted on real or personal property to protect the creditor's interest in case of
the default of the debtor. In other words, GC would not incur in delay before DBP fully
performed its reciprocal obligation. Considering that it had yet to release the entire proceeds
of the loan, DBP could not yet make an effective demand for payment upon GC to perform
its obligation under the loan. Hence, GC would not be in default without the demand. Under
the circumstances, DBP's foreclosure of the mortgage and the sale of the mortgaged
properties at its instance were premature, and, therefore, void and ineffectual.
116. Gonzales v. Solid Cement Corporation, GR No. 198423, 23 October
2012.

Facts of the Case:


Gonzales employment was terminated by his employer, respondent. However, the
LA held that it was an illegal dismissal. So Gonzales filed a motion for execution of
reinstatement. Solid reinstated Gonzales in the payroll. But the NLRC modified the
decision be reduing the amount of damages.

Procedural history

Solid Cement filed a petition for review which was denied by the Court of Appeals. Thus, the
judgement became final and entry of judgement was recorded .

Statement of Issue:

Whether or not the second motion for reconsideration is allowed.

Held:

As a rule, a second motion for reconsideration is a prohibited pleading under the


Rules of Court, and this reason alone is sufficient basis for us to dismiss the present
second motion for reconsideration. The ruling in the original case, as affirmed by the
Court, has been expressly declared final. The immutability principle, rather than
being absolute, is subject to well-settled exceptions, among which is its
inapplicability when a decision claimed to be final is not only erroneous, but null and
void. The matter before us — in the Rule 45 petition questioning the CA's Rule 65
determination — is the scope of the benefits awarded by the LA, as modified on
appeal and ultimately affirmed by this Court, which ruling has become final and
which now must be implemented as a matter of law. Given these considerations, to
reopen this case on second motion for reconsideration would not actually embroil the
Court with changes in the decision on the merits of the case, but would confine itself
solely to the issue of the CA's actions in the course of determining lack or excess of
jurisdiction or the presence of grave abuse of discretion in reviewing the NLRC's
ruling on the execution aspect of the case. Additionally, while continued
consideration of a case on second motion for reconsideration very strongly remains
an exception, our action in doing so in this case is not without sound legal
justification. An order of execution that varies the tenor of a final and executory
judgment is null and void. This was what the CA effectively did — it varied the final
and executory judgment of the LA, as modified on appeal and ultimately affirmed by
the Court.
117. Tadeja v. People, GR No. 145336, 20 February 2013.

Facts of the Case:

Ruben Bernardo had died, and the witness. Elena and Jacinta, were watching a
public dance during a fiesta. There they saw Ruben being hacked to death by
petitioners. Petitioners claim that Ruben and his son went to the barangay in a
drunken state and was waving a knife and cursing at the crowd.

Procedural History:

An Information for homicide for the death of Ruben was filed against petitioners. The RTC
acquitted Russel and Robenson except for Plaridel who absconded, all the other accused
appealed to the CA. The CA issued a Decision in affirming the findings and Decision of the
RTC. This Court issued a decision affirming the Resolution of the CA and denied the
reconsideration. Petitioners filed a Supplemental Motion to Motion with leave of Court to
Vacate Judgment Due to Supervening Event alleging that the police force finally arrested
Plaridel. The Court treated the motion as a second motion for reconsideration and denied it
with finality on the ground that it was a prohibited pleading. Ferdinand prayed for the
reopening of the case on the basis of the confession of Plaridel.

Statement of Issue:

Whether or not to grant the petitioner’s motion to reopen the case for reception of
further evidence in the trial court.

Held:

Newly discovered evidence refers to that which (a) is discovered after trial; (b) could
not have been discovered and produced at the trial even with the exercise of
reasonable diligence; (c) is material, not merely cumulative, corroborative or
impeaching; and (d) is of such weight that it would probably change the judgment if
admitted. The most important requisite is that the evidence could not have been
discovered and produced at the trial even with reasonable diligence; hence, the
term "newly discovered." The confession of Plaridel does not meet this requisite. He
participated in the trial before the RTC and even gave testimony as to his defense. It
was only after he and petitioners had been convicted by the trial court that he
absconded.
118. SJS Officers v. Lim, GR No. 187836, 10 March 2015.

Facts of the Case:

A comprehensive plan and relocation plan was submitted to the RTC of Manila by
Chevron, Pilipinas Shell Petroleum, and Petron.The presiding judge would monitor
the strict enforcement of the decision.

Procedural History:

Now before us are the following submissions of the intervenor oil companies, to wit:
(1) Motion for Reconsideration[4] of the Decision dated 25 November 2014 filed by
intervenor Pilipinas Shell Petroleum Corporation (Shell); (2) Motion for
Clarification[5] filed by intervenor Chevron Philippines, Inc. (Chevron); and (3)
Manifestation of Understanding of the Dispositive Portion of the Decision of 15
December 2014[6] (the correct date of promulgation is 25 November 2014) filed by
intervenor Petron Corporation (Petron).

Statement of Issue:

Whether or not there is interference or encroachment with the policy making powers
of the political departments of the government.

Held:

The allegation of encroachment on the policy making power of the political departments of
the government is bereft of merit. The prayer that the submission of an updated
comprehensive plan and relocation schedule, including the period for relocation, be deferred
until after the Motion is resolved with finality is denied. The compliance period prescribed in
the Decision shall remain. These cases being a mere sequel to the earlier petition, we so
hold that the relocation and transfer contemplated therein include the removal of the facilities,
especially so when the city plans on building commercial establishments to replace the
Pandacan terminals and provide a source of employment for displaced employees.
Accordingly, the comprehensive plan to be submitted within forty-five (45) days from receipt
of the Decision shall also include the removal of the facilities. On the matter of the
enforcement of the assailed Decision in these cases, Petron further posits that its first theory,
that is, that the removal of the facilities is excluded from the comprehensive plan to be
submitted to the Regional Trial Court, would be in accord with its "Manifestation". The denial
of a motion for reconsideration signifies that the grounds relied upon have been found, upon
due deliberation, to be without merit, as not being of sufficient weight to warrant a
modification of the judgment or final order. It means not only that the grounds relied upon are
lacking in merit but also that any other, not so raised, is deemed waived and may no longer
be set up in a subsequent motion or application to overturn the judgment; and this is true,
whatever may be the title given to such motion or application, whether it be "second motion
for reconsideration" or "motion for clarification" or "plea for due process" or "prayer for a
second look," or "motion to defer, or set aside, entry of judgment," or . . ., etc . This
Resolution is final. Under pain of contempt, no further pleadings, motions or papers in the
guise of the above-enumerated submissions shall, thus, be entertained in these cases.
119. ABUBAKAR A. ADFAL and FATIMA A. ADFAL vs. ROMEO CARLOS
G.R. No. 173379
Facts of the Case
A complaint for unlawful detainer against petitioners Gulbajar and Joh Does was filed by
Carlos. It was alleged that they were occupying a parcel of land in respondents name, and
that respondents had allowed them to stay there by mere tolerance. It was claimed that
petitioner sold the property to respondents but allowed petitioners to stay in the property. So
respondent demanded that petitioners turn over the property to him, as he needed such for
his personal use. There were several attempts to serve summons and complaint on
petitioners, but they still failed to answer.

Procedural History

The MTC ruled in favor of respondent. Petitioners then filed the petition for relief before the
RTC, which the court dismissed. It ruled that the same should have been filed before the
MTC in accordance with Sec 1 of Rule 38 which provides that a petition for relief should be
filed "in such court and in the same case praying that the judgment, order or proceeding be
set aside."

Statement of the Issue

Whether the RTC erred in dismissing their petition for relief from judgment.

Holding

A petition for relief from judgment in forcible entry and unlawful detainer cases, as in the
present case, is a prohibited pleading. The legal justification is that Section 13(4) of Rule 70
of the Rules of Court provides petitions for relief from judgment are prohibited. The same is
also provided in the Rules of Summary Procedure. The reason for this is to achieve an
expeditious and inexpensive determination of the cases subject of summary procedure.
Moreover, Section 1, Rule 38 of the Rules of Court provides that when a judgment or final
order is entered, or any other proceeding is thereafter taken against a party in any court
through fraud, accident, mistake or excusable negligence, he may file a petition in such
court and in the same case praying that the judgment, order or proceeding be set
aside. A petition for relief from judgment, if allowed by the Rules and not a prohibited
pleading, should be filed with and resolved by the court in the same case from which the
petition arose.
120. CITY OF DAGUPAN vs. ESTER F. MARAMBA

G. R. No. 174411

Facts of the Case

The City of Dagpuan caused the demolition of a commercial fish center, without giveing
notice to Maramba. There was also included a threat of taking over the property. This made
Maramba file a complaint for injunction with damages and a restraining order. According to
Maramba, the demolition was unlawful and that the "complete demolition and destruction of
the previously existing commercial fish center of plaintiff is valued at Five Million
(₱10,000,000.00) pesos." The word, "ten," was handwritten on top of the word, "five."
Maramba asked for a judgment "ordering defendant corporation to pay plaintiff the amount of
Ten Thousand (₱10,000.00) pesos for the actual and present value of the commercial fish
center completely demolished by public defendant." The word, "million," was handwritten on
top of the word, "thousand," and an additional zero was handwritten at the end of the
numerical figure. The handwritten intercalation was notexplained in any part of the records
and in the proceedings.

Procedural History

The RTC ruled in favor of plaintiff. A petition for relief with prayer for preliminary injunction
was filed. The court denied the petition for relief. However, acting through another judge, it
granted such and modified its earlier decision, reducing the amount to P75k, the amount
representing that which the plaintiff proved. Maramba filed a petition for certiorari with the CA,
which the court granted, ruling that the city’s motion for reconsideration lacked notice and
hearing and did not toll the period to appeal. Petitioner city argues that its petition for relief
was filed on time.

Statement of the Issue

Whether the lack of notice of hearing in a motion for reconsideration is excusable negligence
that allows the filing of a petition for relief of judgment.

Holding

This Court has indeed held time and time again that, under Sections 4 and 5 of Rule 15 of
the Rules of Court, mandatory is the notice requirement in a motion, which is rendered
defective by failure to comply with the requirement. As a rule, a motion without a notice of
hearing is considered pro forma and does not affect the reglementary period for the appeal
or the filing of the requisite pleading. As an integral component of procedural due process,
the three-day notice required by the Rules is not intended for the benefit of the movant.
Rather, the requirement is for the purpose of avoiding surprises that may be sprung upon the
adverse party, who must be given time to study and meet the arguments in the motion
before a resolution by the court. The test is the presence of the opportunity to be heard, as
well as to have time to study the motion and meaningfully oppose or controvert the grounds
upon which it is based. Considering the circumstances of the present case, we believe that
the requirements of procedural due process were substantially complied with, and that the
compliance justified a departure from a literal application of the rule on notice of hearing.
This court held that "when the adverse party has actually had the opportunity to be heard,
and has indeed been heard through pleadings filed in opposition to the motion, the purpose
behind the rule is deemed duly served.”
121. ATTY. RICO PAOLO R. QUICHO, representing BANK OF COMMERCE vs.
BIENVENIDO S. REYES, JR., Sheriff IV, Branch 98, Regional Trial Court, Quezon City
A. M. No. P-14-3246

Facts of the Case

Petitioner alleged that the procedure observed by Reyes in implementing the alias writ
violated the 2002 Revised Manual for Clerks of Court. He cited the manual: "if the judgment
obligor cannot pay all or part of the obligation in cash, certified bank check or other mode of
payment acceptable to the judgment obligee, the officer shall levy upon the properties of the
judgment obligor of every kind and nature whatsoever which may be disposed of for value
and not otherwise exempt from execution giving the latter the option to immediately choose
which property or part thereof may be levied upon, sufficient to satisfy the judgment." He
states that as the holder of the assets and properties of Traders Royal Bank (TRB), BOC
was given the option to choose which property to be surrendered to satisfy the judgment. It
was only when BOC was unable to exercise the option that Reyes could levy on other
properties. BOC was forced to surrender under protest a real estate property to satisfy the
judgment and preserve its other properties from being wrongfully levied by Reyes. It was
argued that Reyes did not give BOC a chance to exercise that option. What happened was
that Reyes blow-torched the locked grill door of BOC’s cash vault and forcibly took the
money deposits of its clients as well as its computers. Reyes sowed terror by bringing with
him agents of the NBI who were in full-battle gear and carrying high-powered firearms. Atty.
Quicho concluded that these illegal acts of Reyes warranted his relief as sheriff of the RTC.
Reyes denied the charges against him. According to him, he did not violate any law when he
refused to accept BOC’s offer of a property located in Paranaque City to satisfy the judgment
debt. Atty. Quicho refuted Reyes’ argument that BOC was not entitled to exercise the option
to choose the properties to be levied; he explained that under Section 9, Rule 39 of the
Rules of Court, it was clear that if the judgment obligor could not pay the judgment debt in
cash, certified bank check or other mode of payment acceptable to the judgment oblige, he
still had the option to choose which of his properties he could offer to satisfy the obligation.

Procedural History

The OCA found sufficient grounds to hold Reyes administratively liable for his
overzealousness in implementing the alias writ of execution. The OCA opined that when
BOC offered its real estate properties in Paranaque to answer for the judgment debt, a legal
issue arose as to whether the offer was acceptable under the law. Thus, according to OCA,
Reyes should have brought the matter to the attention of the Court, instead of resolving it
himself. The OCA recommended that the case be redocketed as a regular administrative
matter against Reyes, and that he be found guilty of Grave Abuse of Authority and fined.

Statement of the Issue

Whether or not Reyes gravely abused his authority relative to the implementation of the writ
of execution

Holding

Section 9, Rule 39 of the Rules of Court provides for the procedure as to how execution of
judgments for money is enforced. The duties of a sheriff are: (1) to first make a demand from
the obligor for the immediate payment of the full amount stated in the writ of execution and of
all lawful fees; (2) to receive payment in the form of cash, certified bank check payable to the
obligee, or any other form of payment acceptable to the latter; (3) to levy upon the properties
of the obligor, not exempt from execution, if the latter cannot pay all or part of the obligation;
(4) give the obligor the opportunity to exercise the option to choose which property may be
levied upon; (5) in case the option is not exercised, to first levy on the personal properties of
the obligor, including the garnishment of debts due the obligor and other credits, i.e., bank
deposits, financial interests, royalties, commissions and other personal properties not
capable of manual delivery or in the possession or control of third parties; and (6) to levy on
real properties if the personal properties are insufficient to answer for the judgment.
From the aforecited provisions, it is clear that the sheriff shall demand from the judgment
obligor the immediate payment in cash, certified bank check or any other mode of payment
acceptable to the judgment obligee. If the judgment obligor cannot pay by these methods
immediately or at once, he can exercise his option to choose which of his property can be
levied upon.
122. HEIRS OF CRISTETA DE LA ROSA vs. HON. ADELINA CALDERON-BARGAS et al.
H. R. No. 147939

Facts of the Case

There was a previous case, wherein petitioners were said to illegally enter a parcel of land in
Rizal, and they were held to vacate the premises and pay damages.

Procedural History

A notice of appeal to the RTC of Morong, Rizal was filed. Private respondents filed a Motion
for Dismissal. Petitioners filed an Opposition to the Motion for Dismissal of Appeal. The RTC
issued an order dismissing the case for failure of the petitioners to file the required
memorandum. Petitioners filed a petition for certiorari before the CA, questioning the
jurisdiction of the respondent judge to hear and try the case. The CA denied the petition and
affirmed in toto the decision of the RTC. The court decreed that failure to comply with
Section 6, Rule 40 of the Rules of Court is not jurisdictional.

Statement of the Issue

Whether or not the Court of Appeals erred in denying the petitioners' Petition and Motion for
Reconsideration based on its erroneous interpretation of Section[s] 6 and 7 of Rule 40 of the
1997 Rules of Civil Procedure.

Holding

Section 9, Rule 41 of the Revised Rules of Court, states, in part, that a party's appeal by
notice of appeal is deemed perfected as to him upon the filing of the notice of appeal in due
time. The RTC acting as an appellate court acquired jurisdiction over the case of the
petitioners upon their filing of the notice of appeal on October 5, 1998. The filing of the notice
of appeal in due time and the payment of the appropriate fees by the petitioners perfected
their appeal in the RTC. As a necessary consequence thereof, the MTC was divested of
jurisdiction over their case. From the filing of the written notice of appeal, petitioners' appeal
was perfected without need of any further act, and, consequently, the trial court lost
jurisdiction over the case, both over the record and over the subject of the case. The alleged
failure of the clerks of court to furnish petitioners copies of the letter of transmittal and Notice
of Appealed Case that resulted in their inability to file their memorandum on time can no
longer be reviewed by this Court.
123. VICTORIANO M. ENCARNACION vs. NIEVES AMIGO
G. R. No. 169793

Facts of the Case

Two lots in Isabela covered by TCTs were registered under the name of Encarnacion. The
lots formed part of a single track of land owned by Valiente. But Valiente sold the same to
Mallapitan, who sold it to Magpantay. Magpantay died, and his widow executed an affidavit
of waiver. In this waiver, she waived her right over the property in favor of her son-in-law,
Encarnacion. Respondent allegedly entered the premises and took possession of a portion
without the permission of Magpantay. Despite demands, respondent did not vacate.

Procedural History

Petitioner filed a complaint for ejectment, damages with injunction and prayer for restraining
order with the Municipal Trial Court in Cities of Isabela. The MTC ruled that respondent
vacate the premises and pay damages. On appeal, the RTC ruled that the case be
dismissed on the ground that the MTC had no jurisdiction over the case. Aggrieved,
petitioner filed a petition for review under Rule 42 of the Rules of Court before the CA, which
remanded the case to the RTC for further proceedings.

Statement of the Issue

Was accion publiciana and not unlawful detainer proper

Holding

In this jurisdiction, the three kinds of actions for the recovery of possession of real property
are a.) Accion interdictal, or an ejectment proceeding which may be either that for forcible
entry or unlawful detainer, which is a summary action for recovery of physical possession
where the dispossession has not lasted for more than one year, and should be brought in
the proper inferior court b.) Accion publiciana or the plenary action for the recovery of the
real right of possession, which should be brought in the proper Regional Trial Court, and
c.) Accion reinvindicatoria, which is an action for the recovery of ownership which must be
brought in the proper Regional Trial Court. After a careful evaluation of the evidence on
record of this case, we find that the Court of Appeals committed no reversible error in
holding that the proper action in this case is accion publiciana; and in ordering the remand of
the case to the RTC for further proceedings. The length of time that the petitioner was
dispossessed of his property made his cause of action beyond the ambit of an accion
interdictal and effectively made it one for accion publiciana. After the lapse of the one-year
period, the suit must be commenced in the Regional Trial Court via an accion
publiciana which is a suit for recovery of the right to possess.
124. RODOLFO “RUDY” CANLAS et al vs. ILUMINADA TUBIL
H. R. No. 184285

Facts of the Case

Tubil together with her husband, claim that they are the owners of a lot. As proof, she says
that before the lot was titled, it was declared for tax purposes in her name. Respondents
erected a residential house on the land on which they are presently occupying.

Procedural History

A complaint for unlawful detainer was filed by Tubil against petitioners before the MTC.
Petitioners filed a motion to dismiss alleging that the MTC is without jurisdiction over the
subject matter. The MTC denied the respondents’ motion because the grounds relied upon
were evidentiary in nature which needed to be litigated. RTC affirmed. CA reversed.

Statement of the Issue

Which court (MTC or RTC) has jurisdiction over the matter?

Holding

The allegations in the complaint are what determines the nature of the action as well as the
court which has jurisdiction over the case. In the instant case, respondent’s allegations in the
complaint clearly make a case for an unlawful detainer, essential to confer jurisdiction on the
MTC over the subject matter. She sent on January 12, 2004 a letter demanding that
petitioners vacate the property but they failed and refused to do so. The complaint for
unlawful detainer was filed on June 9, 2004, or within one year from the time the last
demand to vacate was made.
125. RICARDO S. SILVERIO, JR. vs. COURT OF APPEALS and NELIA S. SILVERIO-
DEE
G. R. No. 178933

Facts of the Case

An intestate proceeding was filed by Ricardo Silberio for the settlement of his deceased
wife’s estate. While the case was pending, petitioner filed a petition to remove the
adminstrator, Ricardo C Silvero sr. Ricardo Silberio Sr. filed an intestate proceeding for the
settlement of his deceased wife’s estate. During the pendancy of the case, petitioner Ricardo
Silberio, Jr. filed a petition to remove Ricardo C. Silverio, Sr. as the administrator of the
subject estate.

Procedural History

The RTC issued an Order granting the petitin. The RTC then issued an Omnibus Order
affirming its order and denying private respondent’s motion for reconsideration. The
Omnibus Order also directed Nelia S. Silverio-Dee to vacate the property within 15 days
from receipt. Ricardo Silverio, Jr. filed a motion for reconsideration which was denied by the
RTC. Meanwhile, Nelia Silverio-Dee filed a Notice of Appeal/ Ricardo Silverio, Jr. filed a
Motion to Dismiss Appeal and for Issuance of a Writ of Execution against the appeal of Nelia
Silverio-Dee on the ground that the Record on Appeal was filed ten (10) days beyond the
reglementary period pursuant to Section 3, Rule 41 of the Rules of Court. The RTC issued
an Order denying the appeal on the ground that it was not perfected within the reglementary
period. CA ruled that the Notice of Appeal was filed within the reglementary period provided
by the Rules of Court applying the "fresh rule period" enunciated by this Court in the Neypes
case

Statement of the Issue

Whether or not the Omnibus orders are interlocutory orders which are not subject to appeal
under Sec. 1 of Rule 41

Holding

Sec. 1(a), Rule 41 of the Rules of Court provides that no appeal may be taken from an order
denying a motion for reconsideration. In all the above instances where the judgment or final
order is not appealable, the aggrieved party may file an appropriate special civil action under
Rule 65. The rationale behind the rule proscribing the remedy of appeal from an interlocutory
order is to prevent undue delay, useless appeals and undue inconvenience to the appealing
party by having to assail orders as they are promulgated by the court, when they can be
contested in a single appeal. The appropriate remedy is thus for the party to wait for the final
judgment or order and assign such interlocutory order as an error of the court on appeal.
The denial of the motion for reconsideration of an order of dismissal of a complaint is not an
interlocutory order, however, but a final order as it puts an end to the particular matter
resolved, or settles definitely the matter therein disposed of, and nothing is left for the trial
court to do other than to execute the order.
126. SPOUSES ELBE LEBIN and ERLINDA LEBIN vs. VILMA S. MIRASOL
H. R. No. 164255

Facts of the Case

The administrator of the Estate of L.J. Hodges was relayed an offer by petitioners to
purchase an asset of the estate. A deposit of 20% of the offer was made. The administrator
stated that Erlinda Lebin was the actual occupant of Lot. 18. Atty. Tabares was
commissioned by the RTC to do an ocular inspection of Lot 18 to see if it was true. In the
report, it was confirmed, and the RTC granted the administrator’s motion for the approval of
the offer. Mirasol offered to purchase the lot, but then found out that her house was actually
standing on Lot 18. Mirasol then filed a petition for relief from the order. While the resolution
was pending, the petitioners made the last payment for installment, but the motion for
execution of the deed of sale was not acted upon by the RTC.

Procedural History
RTC ruled that there shall be equal partition of the subject lot. An appeal was filed. Mirasol
filed a motion to dismiss the appeal. The RTC granted the motion to dismiss the appeal.
Hence, the petitioners appealed via petition for review on certiorari before this court.
Statement of the Issue
Whether or not the RTC erred in dismissing the petitioners’ appeal for their failure to timely
file a record on appeal
Holding

The elimination of the record on appeal in most cases, retaining the record on appeal only
for appeals in special proceedings and in other cases in which the Rules of Court allows
multiple appeals was among the innovations introduced by Batas Pambansa Blg. 12. No
record on appeal shall be required to take an appeal. In lieu thereof, the entire record shall
be transmitted with all the pages prominently numbered consecutively, together with an
index of the contents thereof. The petition for review must be filed with the Court of Appeals
within 15 days from notice of the judgment, and as already stated, shall point out the error of
fact or law that will warrant a reversal or modification of the decision or judgment sought to
be reviewed. An ordinary appeal is taken by merely filing a notice of appeal within 15 days
from notice of the judgment, except in special proceedings or cases where multiple appeals
are allowed in which event the period of appeal is 30 days and a record on appeal is
necessary. There is therefore no longer any common method of appeal in civil cases to the
Supreme Court and the Court of Appeals.
127. ATTY. GEORGE S. BRIONES vs. LILIA J. HENSON-CRUZ, RUBY J. HENSON, and
ANTONIO J. HENSON
G.R. No. 159130

Facts of the Case

Ruby J Henson filed a petitioner to allow the will of her late mother with the RTC of Manila.
One of the deceased’s daughters, Lilia Henson-Cruz, opposed the petition of Ruby.
According to her, Ruby had understated the value of their late mother’s estate and acted
with unconscionable bad faith. Lilia asked that her mother’s holographic will be disallowed
and that she be appointed as the intestate administratrix. Lilia subsequently moved for the
appointment of an interim special administrator of the estate of her late mother. The trial
court then made petitioner Briones the special administrator of the estate. Atty Briones
submitted the special administrator’s final report, and prayed that he be paid a commission
of around 10m. The respondents opposed the approval of the final report. Respondents filed
an audit request with the trial court, which the court granted

Procedural History

The RTC ruled that it reiterates its designation of the accounting firm and suspends the
approval of the report of the special administrator. Respondents filed with the Court of
Appeals (CA) a Petition for Certiorari, Prohibition, and Mandamus, which the trial court
denied. They then filed a Petition for Mandamus. The CA granted the respondent’s petition
for Mandamus, holding that the trial court had neither the power nor the authority to deny the
appeal on the ground of forum shopping. It pointed out that under Section 13, Rule 41 of the
1997 Rules of Civil Procedure, as amended, the authority of the trial court to dismiss an
appeal, either motu proprio or on motion, may be exercised only if the appeal was taken out
of time or if the appellate court docket and other fees were not paid within the reglementary
period.

Statement of the Issue

Whether or not the CA erred in not dismissing the respondent’s petition for mandamus

Holding

From an estate proceeding perspective, the Special Administrator's commission is no less a


claim against the estate than a claim that third parties may make. Section 8, Rule 86 of the
Rules recognizes this when it provides for "Claim of Executor or Administrator Against an
Estate." Hence, by the express terms of the Rules, the ruling on the extent of the
Special Administrator's commission - effectively, a claim by the special administrator
against the estate - is the lower court's last word on the matter and one that is
appealable.
128. BPI FAMILY SAVINGS BANK, INC. vs. PRYCE GASES, INC. et al
G. R. No. 188365

Facts of the Case

Pryce Gases is a debtor of the IFC and the FMO. The IFC and the FMO filed a petitioner for
rehabilitation with the RTC of Makati as Pryce was unable to service it’s debts.

Procedural History

The RTC approved the rehabilitation plan. BFB filed a notice of appeal. PGI filed a motion to
dismiss the appeal. RTC dismissed BFB’s appeal, ruling that the law clearly states that in
special proceedings, record on appeal is required to perfect the appeal. BFB filed a petition
for certiorari before the CA, which the court dismissed. It ruled that corporate rehabilitations
are special proceedings and as such, appeals from the final order or decision therein should
be by record on appeal in accordance with Section 2, Rule 41 of the 1997 Rules of Civil
Procedure.

Statement of the Issue

The issue in this case is whether the Court of Appeals committed a reversible error in
sustaining the RTC decision in dismissing BFB’s appeal.

Holding

A petition for corporate rehabilitation is considered a special proceeding. Thus, the period of
appeal provided in paragraph 19(b) of the Interim Rules Relative to the Implementation
of Batas Pambansa Blg. 129 for special proceedings shall apply, that is, the period of appeal
shall be 30 days since a record of appeal is required. In this case, BFB filed a notice of
appeal on 3 November 2003, before the effectivity of the AM. Hence, at the time of filing of
BFB’s appeal, the applicable mode of appeal is Section 2, Rule 41 of the 1997 Rules of Civil
Procedure which provides that a party’s appeal by record on appeal is deemed perfected as
to him with respect to the subject matter thereof upon approval of the record on appeal filed
in due time. In this case, BFB did not perfect the appeal when it failed to file the record on
appeal. The filing of the notice of appeal was not sufficient because at the time of its filing,
the Rules required the filing of the record on appeal and not merely a notice of appeal.
Appeal is not a matter of right but a mere statutory privilege. The party who seeks to
exercise the right to appeal must comply with the requirements of the rules, failing in which
the right to appeal is lost.
129. BARANGAY SANGALANG vs. BARANGAY MAGUIHAN
H. R. No. 159792

Facts of the Case

A case was lodged before the Sangguinang Bayan. In this case, petitioner claims that the
lots are within their territorial jurisdiction, wheras respondent claims that it is within their
territorial boundary. The case was then referred to a hearing committee. The committee said
that the properties belonged to petitioner.

Procedural History

Respondent appealed the decision to the Regional Trial Court (RTC). RTC rendered a
Decision ruling in favor of respondent. Petitioner filed an MR, which was denied. Aggrieved,
petitioner filed a Notice of Appeal, which the CA dismissed, ruling that petitioner had availed
itself of the wrong remedy in filing a notice of appeal instead of filing a petition for review
under Rule 42 of the Rules of Court.

Statement of the Issue

Whether or not the Court should entertain petitioner’s appeal.

Holding

As correctly observed by the CA, under Section 118 of the Local Government Code, the
jurisdictional responsibility for settlement of boundary disputes between and among local
government units is to be lodged before the proper Sangguniang Panlungsod or
Sangguniang Bayan concerned. In the case at bar, it is clear that when the case was
appealed to the RTC, the latter took cognizance of the case in the exercise of its appellate
jurisdiction, not its original jurisdiction. Hence, any further appeal from the RTC Decision
must conform to the provisions of the Rules of Court dealing with said matter. On this score,
Section 2, Rule 41 of the Rules of Court provides that the appeal to the Court of Appeals in
cases decided by the Regional Trial Court in the exercise of its appellate jurisdiction shall be
by petition for review in accordance with Rule 42. Based on the foregoing, it is apparent that
petitioner has availed itself of the wrong remedy. Since the RTC tried the case in the
exercise of its appellate jurisdiction, petitioner should have filed a petition for review under
Rule 42 of the Rules of Court, instead of an ordinary appeal under Rule 41.
130. ATTY. JESUS F. FERNANDEZ vs. HON. COURT OF APPEALS and CONCEPCION
OLIVARES
G. R. No. 131094

Facts of the Case

Private respondent Concepcion Olivares filed a complaint against petitioner for unlawful
entry.

Procedural History

The MeTC dismissed the Complaint for lack of sufficient cause of action. Olivares appealed
to the RTC of Manila and the latter reversed the MeTC’s decision. 14 days after receipt of
the decision, he filed a Motion for Reconsideration, which was denied. Fernandez filed a
motion for extension of time with the CA. Fernandez asked for a motion for new trial. This
was denied by the RTC. The court ruled that when Fernandez went to the Court of Appeals
and filed a Motion for Extension of Time to File Petition for Review, and the Court of Appeals
accordingly acted on the same by granting the extension sought, jurisdiction of the Court of
Appeals over the parties and the subject matter had already attached. This prompted
Fernandez to file a Petition for Certiorari, Prohibition and Mandamus with prayer for the
issuance of a writ of preliminary injunction and temporary restraining order. Fernandez filed
a Motion for Reconsideration which the Court of Appeals denied.

Statement of the Issue

Whether or not mere filing of a motion for extension of time to file the petition for review
automatically divested the RTC of its jurisdiction over the case

Holding

The Court of Appeals has not yet acquired jurisdiction over the case because Fernandez
merely filed a motion for extension of time to file petition but not the petition itself. In general,
in order for a Court to have authority to dispose of the case on the merits, it must acquire
jurisdiction over the subject matter and over the parties. Jurisdiction over the person is
acquired by service of summons or by voluntary appearance. At first glance and mindful of
the rule that the filing of motions seeking affirmative relief, such as the motion for extension
of time to file petition for review filed by Fernandez in this case, is considered voluntary
submission to the jurisdiction of the court it may seem at once apparent that the Court of
Appeals had in fact acquired jurisdiction over his person. As we are dealing here with the
jurisdiction of an appellate court, additional rules are required for jurisdiction to attach therein,
to wit: (1) the petitioner must have invoked the jurisdiction of the Court of Appeals within the
time for doing so; (2) he must have filed his petition for review likewise within the time for
doing so; (3) he must have paid the necessary docket fees; and (4) the other parties must
have perfected their appeals in due time. The Rule requires that in an appeal by way of
Petition For Review, the appeal is deemed perfected as to the petitioner upon the timely
filing of the petition and the payment of docket and other lawful fees. In the discussion of the
Committee on the revision of the Rules of Court, it was emphasized that to perfect the
appeal, the party has to file the petition for review and to pay the docket fees within the
prescribed period.
131. ST. MARTIN FUNERAL HOME vs. NATIONAL LABOR RELATIONS COMMISSION
and BIENVENIDO ARICAYOS
H. R. No. 130866

Facts of the Case

Alleged is that private respondent worked as an operations manager of petitioner. However,


there was no contract of employment executed between him and petitioner. His name was
also not included in the monthly payroll. He was then dismissed from his employment for
misappropriating 38k. Petitioner claims that respondent was not his employee.

Procedural History

The LA rendered a decision in favor of petitioner declaring that no employer-employee


relationship existed between the parties. Thus his office had no jurisdiction over the
case. Private respondent appealed to the NLRC, and the court set aside the decision,
remanding the case back to LA. Petitioner then filed a motion for reconsideration which was
denied by the NLRC for lack of merit, hence the present petition alleging that the NLRC
committed grave abuse of discretion.

Statement of the Issue

Whether or not appeals from the NLRC can be filed directly with the SC

Holding

The Court is of the considered opinion that ever since appeals from the NLRC to the
Supreme Court were eliminated, the legislative intendment was that the special civil action
of certiorari was and still is the proper vehicle for judicial review of decisions of the NLRC.
Therefore, all references in the amended Section 9 of B.P. No. 129 to supposed appeals
from the NLRC to the Supreme Court are interpreted and hereby declared to mean and refer
to petitions for certiorari under Rule 65. Consequently, all such petitions should hence forth
be initially filed in the Court of Appeals in strict observance of the doctrine on the hierarchy of
courts as the appropriate forum for the relief desired.
132. PHILLIPS SEAFOOD (PHILIPPINES) CORPORATION vs. THE BOARD OF
INVESTMENTS
G. R. No. 175787

Facts of the Case

Philips Seafood Philippines Corporation was granted a tax income holiday for six years for
locating in a less developed area. Petitioner used to supply semi-processed raw materials to
Philips Seafood, Inc, an affiliate corporation, before the latter closed due to financial
difficulties. As they were unable to recover from it’s financial reverses. PSPI stopped
operations. Petitioner acquired title to the plant and informed respondent of transfer.
Respondent infromed petitioner that it’s ITH would extend.

Procedural History

Petitioner received by fax BOI’s letter denying its motion for reconsideration. Petitioner
elevated the matter to the Office of the President, which dismissed petitioner’s appeal on the
ground of lack of jurisdiction. Petitioner filed a petition for review before the Court of Appeals,
questioning the dismissal of its appeal before the Office of the President. The appellate court
denied petitioner’s omnibus motion on the ground that the same was filed with intent to delay
the case. Simultaneously, the appellate court dismissed the petition for review for having
been filed out of time as petitioner opted to appeal to the Office of the President instead of
filing a Rule 43 petition to the Court of Appeals within the reglementary period.

Statement of the Issue

Whether or not the review by the Office of the President of the decisions of respondent BOI
is allowed

Holding

E.O. No. 226 contains no provision specifically governing the remedy of a party whose
application for an ITH has been denied by the BOI. The right to appeal is not a constitutional,
natural or inherent right – it is a statutory privilege and of statutory origin and, therefore,
available only if granted or provided by statute. The BOI was created by virtue of E.O. No.
226 at the time when then President Corazon Aquino was exercising legislative powers
under the Freedom Constitution. E.O. No. 226 apparently allows two avenues of appeal from
an action or decision of the BOI, depending on the nature of the controversy. One mode is to
elevate an appeal to the Office of the President when the action or decision pertains to either
of these two instances: first, in the decisions of the BOI over controversies concerning the
implementation of the relevant provisions of E.O No. 226 that may arise between registered
enterprises or investors and government agencies under Article 7; and second, in an action
of the BOI over applications for registration under the investment priorities plan under Article
36.
133. PADILLA MERCADO et al vs. SPOUSES AGUEDO ESPINA and LOURDES ESPINA
H. R. No. 173987

Facts of the Case

Alleged in the complaint of petitioners is that they are the heirs of the late spouses Santiago
and Sofronia Mercado. Both were owners of the subject parcel of land. After the death of
Santiago and Sofronia, petitioners inherited the lot. Respondents then claimed that the lot
was theirs, as gthey had bought it from Josefa, who in turn bought it from Genivera, who
bought it from Mercado. Petitioners say that Josefa, through fraudulent machinations,
obtained title over the subject property in her name. A declaration of nullity of the deed of
sale was prayed for.

Procedural History

Respondents filed a Motion to Dismiss on grounds that the RTC has no jurisdiction over the
case. The RTC denied respondents' Motion to Dismiss. Respondents then filed a motion for
reconsideration, but the same was denied by the RTC. Respondents then filed a special civil
action for certiorari with the CA, which the court also denied. Meanwhile, petitioners, by
leave of court, filed an Amended Complaint to include the assessed value of the subject
property. Respondents filed a Motion to Dismiss Amended Complaint, which was denied.
Respondents filed a special civil action for certiorari with the CA praying that both orders be
set aside and the petition dismissed, which the CA finally granted. Petitioners filed a motion
for reconsideration, but the CA denied it

Statement of the Issue

Was giving due course to respondents’ second motion to dismiss proper

Holding

Under Section 8, Rule 10 of the Rules of Court, an amended complaint supersedes an


original one. As a consequence, the original complaint is deemed withdrawn and no longer
considered part of the record. In the present case, the Amended Complaint is, thus, treated
as an entirely new complaint. As such, respondents had every right to move for the dismissal
of the said Amended Complaint.
134. MARICALUM MINING CORPORATION vs. REMINGTON INDUSTRIAL SALES
CORPORATION
G. R. No. 158332

Facts of the Case

Marinduque Mining and Industrial Corporation was sued by Remington Industrial Sales
Corporation for payment of around 900 thousand pesos worth of construction materials and
other merchandise. The complaint was amended 4 times, and impleaded other parties
because they were assigness/transferees of the real and personal properties, chattels,
machineries, equipment, and other assets of Marinduque Mining.

Procedural History

The RTC ruled in favor of the plaintiff, ordering the defendants Marinduque Mining &
Industrial Corporation et al. to pay the sum demanded. Petitioner and its co-defendants PNB,
DBP, Nonoc Mining, ICC and APT filed with the CA an appeal, which the court dismissed.
On its own, petitioner also attempted to institute an appeal with the Court. However, for lack
of an affidavit of service the Court denied its motion. Thus, private respondent filed with the
RTC a Motion for Execution solely against petitioner. Over petitioner's objection, the RTC
granted the Motion for Execution, dismissing the original complaint filed in the RTC. This
became final.

Statement of the Issue

Whether the Court's Decisions in DBP v. CA and PNB v. CA inured to the benefit of
petitioner which was not a party to either case

Holding

The general rule is that one party's appeal from a judgment will not inure to the benefit of a
co-party who failed to appeal; and as against the latter, the judgment will continue to run its
course until it becomes final and executory. To this general rule, however, one exception
stands out: where both parties have a commonality of interests, the appeal of one is deemed
to be the vicarious appeal of the other. The adjudication rendered in DBP v. CA and PNB v.
CA is plain: private respondent has no cause of action against DBP, PNB and their
transferees, including petitioner, for they are corporate entities separate and distinct from
Marinduque Mining, and cannot be held liable for the latter's obligations to private
respondent.
135. Corazon Cruz v. MIAA, GR No. 184732, 9 September 2013.
G. R. No. 184732

Facts of the Case:

Petitioner Corazon Cruz entered into a contract with MIAA for lease of a 1400 square meter
property which petitioner would use as a commercial arcade for lease to other businesses.
However, according to her, respondent did not inform her that part of the leased premises
was subject to public use (easement), and as a result she suffered losses such as being
unable to obtain a building permit and certificate of electrical inspection from the Manila
Electric Company, tenants not paying rent, etc. Since her tenants could not pay, she was
unable to pay her own rent Dec 2004 onwards. And as some of Cruz’s stalls were located in
the easement area, MMDA demolished them, causing her damages. She tool this into
consideration when computing her rent, however MIAA terminated the lease contract.

Procedural History
Cruz then filed a case before the RTC of Pasic city for breach of contract, consignation, and
damages. MIAA then filed a motion to dismiss, and the RTC dismissed the complaint due to
forum shopping, as the Pasig and Manila cases are founded on the same actionable
document. Petitioner filed a motion for reconsideration which was denied, thus Cruz filed a
notice of appeal. The CA held that the dismissal by the RTC was proper, but that Cruz did
not commit forum shopping but that venue was improper as she admitted that her residence
was actually in Manila and not San Juan.

Statement of the Issue:


Was the dismissal of the case proper?

Holding

In the case at bar, the Court finds that the CA committed a reversible error in sustaining the
dismissal of the Pasig case on the ground of improper venue because the same was not an
error raised by Cruz who was the appellant before it. Jurisprudence dictates that the
appellee’s role in the appeal process is confined only to the task of refuting the assigned
errors interposed by the appellant. Since the appellee is not the party who instituted the
appeal and accordingly has not complied with the procedure prescribed therefor, he merely
assumes a defensive stance and his interest solely relegated to the affirmance of the
judgment appealed from. Keeping in mind that the right to appeal is essentially statutory in
character, it is highly erroneous for the appellee to either assign any error or seek any
affirmative relief or modification of the lower court’s judgment without interposing its own
appeal. As held in the case of Medida v. CA:An appellee who has not himself appealed
cannot obtain from the appellate court any affirmative relief other than the ones granted in
the decision of the court below. He cannot impugn the correctness of a judgment not
appealed from by him. He cannot assign such errors as are designed to have the judgment
modified. All that said appellee can do is to make a counter-assignment of errors or to argue
on issues raised at the trial only for the purpose of sustaining the judgment in his favor, even
on grounds not included in the decision of the court a quo nor raised in the appellant's
assignment of errors or arguments.
136. DAR v. Berenguer

GR No. 154094, 9 March 2010.

Facts of the case:

Respondents were registered owners of several residential and industrial lands in Barangay
Bibincahan, Sorsogon. The respondents received from the Department of Agrarian Reform
that their land fell under the CARP. This was protested, and respondents filed in the office of
the DAR an application for exclusion of landholdings from the CARP coverage. Without
acting on the application, their titles and certificates were canceled.

Procedural history:

Later, Regional Director Dalugdug denied the respondents application for exclusion, and
thus they appealed to DAR secretary. Pending the resolution of respondents appeal, Baribag
filed in the DAR office a petition seeking to implement the order of the regional director, and
he was placed in possession of respondent’s land. Respondents appealed before the DAR
adjudication board, which was noted but a write of possession was given to Baribag. CA
reversed DAR’s order

Statement of Issue:

Whether or not the CA erred in not dismissing respondent’s petition for certiorari

Held

The CA did not err in treating the petition for certiorari as a petition for review. There are
precedents in that regard. In Department of Education v. Cuanan,11 this Court ruled that the
petition for certiorari filed by therein respondent Cuanan with the CA within the 15-day
reglementary period for filing a petition for review could be treated as a petition for review,
for that would be in accord with the liberal spirit pervading the Rules of Court and in the
interest of substantial justice. The Court had occasion to expound on the exceptions to the
rule that a recourse to a petition for certiorari under Rule 65 rendered the petition dismissible
for being the wrong remedy, thus: The remedy of an aggrieved party from a resolution
issued by the CSC is to file a petition for review thereof under Rule 43 of the Rules of Court
within fifteen days from notice of the resolution. Recourse to a petition for certiorari under
Rule 65 renders the petition dismissible for being the wrong remedy. Nonetheless, there are
exceptions to this rule, to wit: (a) when public welfare and the advancement of public policy
dictates; (b) when the broader interest of justice so requires; (c) when the writs issued are
null and void; or (d) when the questioned order amounts to an oppressive exercise of
judicial authority. As will be shown forthwith, exception (c) applies to the present
case.Furthermore, while a motion for reconsideration is a condition precedent to the filing of
a petition for certiorari, immediate recourse to the extraordinary remedy of certiorari is
warranted where the order is a patent nullity, as where the court a quo has no jurisdiction;
where petitioner was deprived of due process and there is extreme urgency for relief; where
the proceedings in the lower court are a nullity for lack of due process; where the proceeding
was ex parte or one in which the petitioner had no opportunity to object.
137. Panganiban v. Tara Trading Ship Management Inc.

GR No. 187302, 18 October 2010.

Facts of the case

Petitioner was hired by Tara Trading Shipmanagement on behalf of respondent Shiling SDB
BHD to work as an oiler. Aroung 2006, petitioner started exhibiting signs of mental instability
and was diagnosed with brief psychotic disorder. DEspite demands for compensation,
respondents failed and refued to comply with their contractual obligations.

Procedural history

Petitioners filed a complaint against respondents praying for the payment of 60,000 as total
and permanent disability benefits. The Labor arbiter ruled in favor of petitioners. The case
was appealed to the NLRC, who affirmed the decision of the Labor Arbiter. The appeal of
respondents was dismissed for lack of merit. Respondents filed a Petition for Certiorari with
prayer for the issuance of a write of preliminary injunction and temporary restraining order
with the CA. The CA reversed the decision of the NLRC.

Statement of issue:

Whether or not the petitioner can file both a petition for review under Rule 45 and a petition
for certiorari under Rule 65

Held:

Preliminarily, considering the grounds raised by petitioner, it appears that he denominated


this petition as one under Rule 45, but he filed it as both a petition for review under Rule
45 and a petition for certiorari under Rule 65 of the Rules of Court. The applicable rule is
Rule 45, which clearly provides that decisions, final orders or resolutions of the CA in any
case, regardless of the nature of the action or proceeding involved, may be appealed to this
Court through a petition for review. This remedy is a continuation of the appellate process
over the original case. Recourse under Rule 65 cannot be allowed either as an add-on or as
a substitute for appeal.The procedural infirmity notwithstanding, the Court shall treat this
petition as one filed under Rule 45 only and shall consider the alleged grave abuse of
discretion on the part of the CA as an allegation of reversible error
138. Llamas v. CA

GR No. 149588, 29 September 2009

Facts of the case

On November 1978, petitioners conspired and, despite knowing that their parcel of land in
Paranaque was mortgaged to Rural Bank of Imus, seel such to Conrado P Avilla.
Petitioners were doung duilty and sentenced to two months imprisonment.

Procedural history

On appeal, the Court of Appeals affirmed the decision of the trial court. Petitioners then
filed a petition for review, which was denied for failure to state material dates. Since it
denied petitioners motion for reconsideration, the judgement became final and executory.
Petitioner Francisico moved for the lifting of the warrant of arrest, raising for the first time
the issue that the trial court had no jurisdiction over the offence charged. There being no
actions taken by the trial court on the said motion, petitioners instituted on September 13
the instant proceedings for the annumlment of the trial and the appellate courts’ decision.

Statement of issue:

Whether or not Rule 47 may be applied to criminal cases

Held

In People v. Bitanga,13 the Court explained that the remedy of annulment of judgment cannot
be availed of in criminal cases, thus —

Section 1, Rule 47 of the Rules of Court, limits the scope of the remedy of annulment of
judgment to the following:

Section 1. Coverage. — This Rule shall govern the annulment by the Court of Appeals of
judgments or final orders and resolutions in civil actions of Regional Trial Courts for which
the ordinary remedies of new trial, appeal, petition for relief or other appropriate remedies
are no longer available through no fault of the petitioner.a1f

The remedy cannot be resorted to when the RTC judgment being questioned was rendered
in a criminal case. The 2000 Revised Rules of Criminal Procedure itself does not permit
such recourse, for it excluded Rule 47 from the enumeration of the provisions of the 1997
Revised Rules of Civil Procedure which have suppletory application to criminal cases.
Section 18, Rule 124 thereof, provides:

Sec. 18. Application of certain rules in civil procedure to criminal cases. – The provisions of
Rules 42, 44 to 46 and 48 to 56 relating to procedure in the Court of Appeals and in the
Supreme Court in original and appealed civil cases shall be applied to criminal cases insofar
as they are applicable and not inconsistent with the provisions of this Rule.

There is no basis in law or the rules, therefore, to extend the scope of Rule 47 to criminal
cases.
139. Sps. Arcenas v. Queen City Development Bank

GR No. 166819, 16 June 2010.

Facts of the case

Petitioners filed with the Regional Trial Court of Roxas City an action for declaratory relief
against respondent, praying for the declaration of their rights as lessors under the
contract of lease.

Procedural history

Respondent filed an answer contending that the action for declaraotry releif was not
proper since the contract of lease had already been violated. RTC dismissed the action
for declaratory relief. Petitioners then filed with the RTC another case, this time for breach
of the same contract of lease, which the RTC set for pre trial. Petitioners filed a pre trial
brief, pre trial was reset as respondent’s counsel wanted to know the exact amount for
settlement. On the date of pre-trial, only respondent was present. On the date scheduled
for the hearing, petitioner and counsel failed to appear thus respondent bank presented
evidence for it’s counterclaim. Petitioner filed a manifestiation with motion, alleging that
the faulure to file a motion to reconsider the order and failure to attend hearing was due to
their mistaken belief that respondent bank was earnestly seeking a settllement.
Respondent filed an opposition to such manifestation and motion. RTC denied the
manifestation and motion to reconsider. Petitioners then filed with the CA a petition for
annulment of order, which was dismissed as there was no appropriate remedies without
sufficient justification before reporting to the petition for annulment of order. CA also
denied motion for reconsideration filed by petitioners. On July 14, Oscar Arcenas, one of
the petitioners died, leaving Dolores the only petitioner, who filed this petition for review.

Statement of issue:

Was the CA correct in dismissing the petition for annulment of judgement?

Held

Section 1, Rule 47 provides that it does not allow a direct recourse to a petition for
annulment of judgment if other appropriate remedies are available, such as a petition for
new trial, appeal or a petition for relief.17 If petitioner fails to avail of these remedies without
sufficient justification, she cannot resort to the action for annulment of judgment under Rule
47, for otherwise, she would benefit from her inaction or negligence Petitioner tries to justify
her failure to avail of the appropriate remedies on a promise of settlement. However, such
promise was not an excuse for petitioner's counsel not to lift the order of non-suit and to file
a petition for relief.Petitioner's claim that she was present when respondent bank's counsel
moved for the issuance of the order of non-suit against her was not proven by any
evidence.There was indeed a failure to show, to our satisfaction, that petitioner could not
have availed of the ordinary and appropriate remedies under the Rules. Thus, she cannot
resort to the remedy under Rule 47 of the Rules; otherwise, she would benefit from her
inaction or negligence.
140. Genato Investments Inc. v. Barrientos

GR No. 207443, 23 July 2014

Facts of the case:

A lot situated in Caloocan city was registered under the name of the petitioner. Due to an
alleged deficiency in real property taxes, the office of the city treasurer of Caloocan city
sold the land at public auction, and respondent was the highest bidder and the office of
the city treasurer issued a certificate of sale of delinquent property. Petitioner was not
aware of any of the proceedings before the office of the city treasurer. REspondent
prayed for the consolidation of ownership of the proper, despite the fact that the
delinquency sale only involved one lot of the property.

Procedural history

RTC issued an order granting respondent’s petition. Petitioner filed with the CA a petition
for Certiorari under Rule 65 but withdrew, CA granted prayer to withdraw. Petitioner then
filed with the CA a petitioner for annulment of judgement. CA dismissed claiming that the
petition for annulment of judgement is not the proper remedy

Statement of issue:

Whether or not the CA was correct in dismissing on the grounds that the petition for
annulment of judgement is not the proper remedy

Held

We disagree with the reasoning of the CA and respondents that petitioner in this particular
case should have filed either an action for reconveyance or annulment of the auction sale,
because to do so would have required the court hearing the action to modify or interfere with
the judgment or order of another co-equal court, especially in this case where the said
judgment ororder had attained finality. Wellentrenched in our jurisdiction is the doctrine that
a court has no power to do so, as that action may lead to confusion and seriously hinder the
administration of justice.We have repeatedly ruled that a Petition for Annulment of Judgment
under Rule 47 of the Rules of Court is a remedy granted only under exceptional
circumstances where a party,without fault on his part, has failed to avail of the ordinary
remedies of new trial, appeal, petition for relief or other appropriate remedies. The same
petition is not available as a substitute for a remedy which was lost due to the party’s own
neglect in promptly availing of the same.31 There is here no attempted substitution;
annulment of judgment is the only remedy available to petitioner.
141. Natalia Realty v. CA

GR No. 126462, 5 February 2003

Facts of the case

Petitioner alleged that respondents were occupying and illegally squatting on certain
portions of the subject property. Private respondents claim that they are the wners of their
respoective houses and lots that have been in their possession before the outbreak of
World War II

Procedural history

Natalia Realty filed with the Regional Trial Court an action for recovery of possession of
the two parcels of land. The case was dismissed for petitioner’s failure to prosecute.
However, the trial court issued another order by a different judge granting the motions of
private respondents to order petitioner to surrender possession of portions of the subject
property. The orders then attained finality and petitioner moved to set aside the orders
claiming excusable oversight on the part of its counsel. This was denied by the trial court.
An entry of judgement was issued claiming that the resolution became final and executory.
A writ of execution was filed by Navarro who claimed to be the original counsel of
respondents, but was denied, as it was ruled that it was not a class suit. Navarro
appealed to the Court of Appeals. REpsondents then filed for execution, but petitioner
opposed the motion as the trial court had denied because of a supervening event. Judge
Queribin wrote a letter instead of acting on the motion, and inquired what final orders and
decisions he should enforce. The resoultion of the motion for execution was then allowed.
Petitioner thhen filed with the Court of Appeals a motion to set aside as the personal letter
of the judge is not authorized by the Rules of Court.

Statement of issue

Whether or not the writ of execution to should have been allowed

Held

The general rule is when a court's judgment or order becomes final and executory, it is the
ministerial duty of the trial court to issue a writ of execution to enforce the judgment or
order.29 In this case, because of the innumerable delays, the enforcement of the final orders
and decision has been long overdue. The delays are partly attributable to the fact that the
case went through a number of presiding judges. The filing of unauthorized motions and
appeal by Navarro complicated what was otherwise just a simple case of enforcement of
final judgment. But it was petitioner’s intolerable strategy of filing motions, some belatedly
filed, to thwart execution that caused this case to drag for more than a decade.Judge
Querubin could have ended the delays had he readily complied with the February 6, 1995
Resolution of the Court of Appeals ordering him to execute the final orders and decision in
Civil Case No. 359-A. Instead, Judge Querubin wrote a letter to the Court of Appeals asking
it to specify the final orders and decision that should be enforced. In response to the query,
the Court of Appeals issued another resolution identifying these final orders and
decision.While we agree with petitioner that Judge Querubin’s query is not provided for in
the Rules of Court, it is nevertheless not prohibited.
142. Philippine Hawk Corp. v. Lee
GR No. 166869, 16 February 2010

Facts of the case

A vehicular accident occurred in Quezon which killed respondent’s husband and caused
respondent physical injuries. The accident involved a motorcycle, a passenger jeek, and
a buy. The buy was owned by petitioner and was bieng driven by Margarito Avila

Procedural history

Respondent then filed before the RTC of Quezon City a complaint against petitoner
Philippine Hawk corporation. The trial court issed a pre-trial order, and the parties agreed
to stipulate on certain facts and issues. Avila was found guilty of simple negligence. On
appeal, the Court of appeals affirmed the decision of the trial court.

Statement of issue:

Whether or not the court of appeals erred in awarding other kinds of damages in favor of
respondent, when this was not appealed from the trial courts decision

Held:

Rule 51 SEC. 8. Questions that may be decided. -- No error which does not affect the
jurisdiction over the subject matter or the validity of the judgment appealed from or the
proceedings therein will be considered unless stated in the assignment of errors, or closely
related to or dependent on an assigned error and properly argued in the brief, save as the
court pass upon plain errors and clerical errors.1. Sec. 8, which is an amendment of the
former Sec. 7 of this Rule, now includes some substantial changes in the rules on
assignment of errors. The basic procedural rule is that only errors claimed and assigned by a
party will be considered by the court, except errors affecting its jurisdiction over the subject
matter. To this exception has now been added errors affecting the validity of the judgment
appealed from or the proceedings therein.Also, even if the error complained of by a party is
not expressly stated in his assignment of errors but the same is closely related to or
dependent on an assigned error and properly argued in his brief, such error may now be
considered by the court. These changes are of jurisprudential origin. The procedure in the
Supreme Court being generally the same as that in the Court of Appeals, unless otherwise
indicated (see Secs. 2 and 4, Rule 56), it has been held that the latter is clothed with ample
authority to review matters, even if they are not assigned as errors on appeal, if it finds that
their consideration is necessary in arriving at a just decision of the case. Also, an
unassigned error closely related to an error properly assigned, or upon which the
determination of the question raised by error properly assigned is dependent, will be
considered by the appellate court notwithstanding the failure to assign it as error.It may also
be observed that under Sec. 8 of this Rule, the appellate court is authorized to consider a
plain error, although it was not specifically assigned by the appellant, otherwise it would be
sacrificing substance for technicalities
143. Gonzales III v. Office of the President

GR No. 196231-32, 28 January 2014

Statement of the facts:

Christian Kalaw filed separate charges with the PNP IAS against Manila Police District
Senior Inspector Mendoza and four others for robbery, grave threat, robbery extortion, and
physical injury. Police Senior Superintendent Atty. Clarence Guinto filed an administrative
charge for grave isconduct with the National Police Commission against Mendoza based on
the allegations of Kalaw.After preparing a draft decision on MEdoza’s case, the deputy
obudsman, who found Mendoza guilty, forwarded the records to the office of the
ombudsman. This was approved. Pending action by ombudsman, Mendoza hijacked a
toursit bus and held 21 foreign tourists and four Filipino tour assistants hostages. Mendoza
died. Ombudsman was found guilty of gross misconduct in handling the case against
Mendoza. Gonzales, the deputy pmbudsman, was charged for gross neglect of duty.

Procedural history

Gonzales posits that OP has no administrative disciplinary jurisdiction over a Deputy


Ombudsman.

Statement of issue

Whether on not the Office of the president’s ruling that Gonzales had been grossly negligent
for taking nine days instead of five days to review the motion for reconsideration was
baseless

Held

Gonzales cannot be guilty of gross neglect of duty and/or inefficiency since he acted on the
case forwarded to him within nine days. In finding Gonzales guilty, the OP72 relied on
Section 8, Rule III of Administrative Order No. 7 (or the Rules of Procedure of the Office of
the Ombudsman, series of 1990, as amended) in ruling that Gonzales should have acted on
Mendoza’s Motion for Reconsideration within five days. Even if we consider this provision to
be mandatory, the period it requires cannot apply to Gonzales since he is a Deputy
Ombudsman whose obligation is to review the case; he is not simply a Hearing Officer
tasked with the initial resolution of the motion. In Section 6 of Administrative Order No. 7 on
the resolution of the case and submission of the proposed decision, the period for resolving
the case does not cover the period within which it should be reviewed
144. Vivares v. Reyes

GR No. 155408, 13 February 2008

Facts of the case

Severino Reyes was the father of respondent Jose Reyes and Torcuato Reyes. Upon his
death, the two came upon their inheriance and had an oral partition of properties. When
Torcuato dies, his will was up for probate. Petitioner, the executor of the will when Ignaling
(also petitioner ) was declared a lawful heir to Torcuato. Believing that Torcuato did not
receive his full share in the estate of Severino, petitioners instituted an action for partition
and recovery of real estate.

Procedural history

A commission was formed and the disputed properties were annotated with notices of lis
pendens. Petitioners filed a motion to place properties in litigation under recievership before
the trial court, alleging that to their prejudice respondent had, without prior court approval
and without petitioner’s knowledge, sold to third parties and transferred in his own name
several common properties.

Statement of issue

Whether or not receivership was justified

Held

We sustain the CA ruling that the trial court acted arbitrarily in granting the petition for
appointment of a receiver as "there was no sufficient cause or reason to justify placing the
disputed properties under receivership." Petitioners cannot now impugn the oral partition
entered into by Torcuato and respondent and hence cannot also assail the transfers made
by respondent of the lots which were subject of said agreement, considering that Torcuato
also sold properties based on said verbal arrangement. Indeed, the parties agreed that the
civil action does not encompass the properties covered by the oral partition. In this factual
setting, petitioners cannot convince the Court that the alleged fraudulent transfers of the lots
made by respondent, which purportedly form part of his share in Severino’s estate based on
the partition, can provide a strong basis to grant the receivership. petitioner is willing to post
a counterbond in the amount to be fixed by the court based on Sec. 3, Rule 59 of the 1997
Rules of Civil Procedure.Anchored on this rule, the trial court should have dispensed with the
services of the receiver, more so considering that the alleged fraud put forward to justify the
receivership was not at all established.Petitioners advance the issue that the receivership
should not be recalled simply because the adverse party offers to post a counterbond. At the
outset, we find that this issue is barred by estoppel.10 While the CA made a statement that
the trial court should have discharged the appointed receiver on the basis of the proposed
counterbond, such opinion does not jibe with the import of Sec. 3, Rule 59. The rule states
that the "application may be denied or the receiver discharged." In statutory construction, the
word "may" has always been construed as permissive. If the intent is to make it mandatory
or ministerial for the trial court to order the recall of the receiver upon the offer to post a
counterbond, then the court should have used the word "shall." Thus, the trial court has to
consider the posting of the counterbond in addition to other reasons presented by the offeror
why the receivership has to be set aside.
145. Smart Communications v. Astorga

GR No. 148132, 28 January 2008

Facts of the case

Astorga was employed by Smart Communications as a Distric Sales manager of the


Corporate SAles Marketing Group. SMART launghed an organizational realignment to
achieve more efficent operations. Part of the reorganization was the outsourcing of the
marketing and sales force. To soften the blow, SNMI agreed to absorb the personnel who
would be recommended by SMART, but Astorga landed last in the performance
evaluation. SMART offered her a supervisory position in the customer care department
but she refused due to a lower salary rank and rate. DEspite the abolition of her office,
she continued reporting for work, so SMART terminated her. Astorga then filed a
complaint for illegal dismissal. In the meantime, SMART sent a letter to Astorga
demanding that she pay the current market value of the Honda which was given to her
under the company’s car plan program. She did not do this, and SMART filed a replevin.

Procedural history

The labor arbiter ruled in her favor regarding employment. Astorga filed a motion to
dismiss the replevin case. The RTC fenied Astorga’s motion regarding the replevin case.
She brought it to the CA, who reversed the RTC ruling, claiming that it was intertwined
with Astorga’s complaint for illegal dismissal. However, the NLRC ordered that Astorga
return the vehicle. She filed then a motuon for reconsideration but it was denied.

Statement of issue

Whether or not the replevin was proper

Held

Contrary to the CA’s ratiocination, the RTC rightfully assumed jurisdiction over the suit
and acted well within its discretion in denying Astorga’s motion to dismiss. SMART’s
demand for payment of the market value of the car or, in the alternative, the surrender of
the car, is not a labor, but a civil, dispute. It involves the relationship of debtor and creditor
rather than employee-employer relations.33 As such, the dispute falls within the
jurisdiction of the regular courts.
146. Monetary Board v. Philippine Veterans Bank

GR No. 189571, 21 January 2015

Statement of fact

Respondent established a pension loan product for bona fide veterans or their surviving
spouses, as well as salary loan product for teachers and low salaried employees to provide
financial assitance to veterans and teachers. Respondent devised a program by charging a
premium in the form of a higher fee known as credit Redemption Fund from said borrowers.
An exampination was conducted by the Supervision and Examination Department of the
Banko Sentral ng Pilipinas, which found, among other things, that respondent’s collection of
premiums from the proceeds of the various salary and pension loans violated Section 54 of
RA 8791, whichs tates that banks shall not directly engage in the insurance business as
insurer. Respondent wrote a letter justifying the existence of the CRF. Petitioners issued a
resolution directing respondent’s trust and investment department to return to the borrowers
all the balances of the CRF.

Procedural history

The RTC dismissed respondent’s petition for declaratory relief. Almost a year later,
respondent filed a Motion to Admit it’s Motion for Reconsideration. RTC allowed the motion
and required petitioners to file their answer. RTC then granted respondents petition for
declaratory relief.

Statement of issue

Whether or not the petition for declaratory relief is proper

Held

Declaratory relief is defined as an action by any person interested in a deed, will, contract or
other written instrument, executive order or resolution, to determine any question of
construction or validity arising from the instrument, executive order or regulation, or statute;
and for a declaration of his rights and duties thereunder. The only issue that may be raised
in such a petition is the question of construction or validity of provisions in an instrument or
statute.9 Ergo, the Court, in CJH Development Corporation v. Bureau of Internal
Revenue,10 held that in the same manner that court decisions cannot be the proper subjects
of a petition for declaratory relief, decisions of quasijudicial agencies cannot be subjects of a
petition for declaratory relief for the simple reason that if a party is not agreeable to a
decision either on questions of law or of fact, it may avail of the various remedies provided
by the Rules of Court. In view of the foregoing, the decision of the BSP Monetary Board
cannot be a proper subject matter for a petition for declaratory relief since it was issued by
the BSP Monetary Board inthe exercise of its quasi-judicial powers or functions.
147. Dacudao v. Sec. of Justice,

GR No. 188056, 8 January 2013

Statement of facts

Petitioners were among the investors whom Angeles and his associates in the Legacy
Group allegedly defrauded through the Legacy Group’s buy back agreement that earned
them check payments that were dishonored. After their written demands went unheeded,
they initiated a number of charges for syndicated estafa against Angeles.

Procedural history

The Secretary of Justice directed all Regional State prosecutors to forward all cases
already filed against Delos Angeles to the secretariat of the DOJ. An order was then
issued directing the cases to be fowarded wither to the Secretariat of the department of
justice or the secretariat of the special panel for distribution. Petitioners went to the
Supreme Court on petition for certiorari, prohibition, and mandamus, ascribing to the
respondent Secretary of Justice grave abuse of discretion.

Statement of issue

Did petitioners property bring their petition for certiorari, prohibition, and mandamus
directly to the court?

Held:

The petition for certiorari, prohibition and mandamus, being bereft of substance and merit, is
dismissed.Firstly, petitioners have unduly disregarded the hierarchy of courts by coming
directly to the Court with their petition for certiorari, prohibition and mandamus without
tendering therein any special, important or compelling reason to justify the direct filing of the
petition.We emphasize that the concurrence of jurisdiction among the Supreme Court, Court
of Appeals and the Regional Trial Courts to issue the writs of certiorari, prohibition,
mandamus, quo warranto, habeas corpus and injunction did not give petitioners the
unrestricted freedom of choice of court forum. An undue disregard of this policy against
direct resort to the Court will cause the dismissal of the recourse. In Bañez, Jr. v.
Concepcion, we explained why, to wit:The Court must enjoin the observance of the policy on
the hierarchy of courts, and now affirms that the policy is not to be ignored without serious
consequences. The strictness of the policy is designed to shield the Court from having to
deal with causes that are also well within the competence of the lower courts, and thus leave
time to the Court to deal with the more fundamental and more essential tasks that the
Constitution has assigned to it. The Court may act on petitions for the extraordinary writs of
certiorari, prohibition and mandamus only when absolutely necessary or when serious and
important reasons exist to justify an exception to the policy.
148. Bank of Commerce v. Radio Philippines Network

GR No. 195615, 21 April 2014.

Statement of facts

Traders Royal Bank propsoed to sell to petitioner Bank of Commerce it’s banking
business consisting of specified assets and liabilities. Bancommerce agreed subject to
Banko Sentral ng Pilipinas’ approval of their purchase, and it was approved, subject to
Banccocommerce annd TRB setting up an escrow fund. To comply, TRB placed 50 m in
escrow with Metrobank. BSP approved. Shortly after, TRB was ordered to pay
respondents damages for a previous case. REspondents filed a motion for execution, but
ather than pursue a levy, they filed a supplemental motion for execution where they
described TRB now as the Bank of Commerce.

Procedural history

Bancommerce filed it’s special appearance with opposition to the same questioning the
jurisdiction of the RTC. This prompted Bancommerce to file a petition for certiorari with
the Court of Appeals. The Court of Appeals denied the petition. Bancommerce sought
reconsideration of the RTC order, but RTC denied. Bancommerce elevated the RTC
order to the CA via a petition for certiorari under Rule 65.

Statement of issue

Whether or not the direct filing of the petition for ceritorari by Bancommerce was valid

Held

Section 1, Rule 65 of the Rules of Court provides that a petition for certiorari may only be
filed when there is no plain, speedy, and adequate remedy in the course of law. Since a
motion for reconsideration is generally regarded as a plain, speedy, and adequate remedy,
the failure to first take recourse to is usually regarded as fatal omission.But Bancommerce
invoked certain recognized exceptions to the rule.12 It had to forego the filing of the required
motion for reconsideration of the assailed RTC Order because a) there was an urgent
necessity for the CA to resolve the questions it raised and any further delay would prejudice
its interests; b) under the circumstances, a motion for reconsideration would have been
useless; c) Bancommerce had been deprived of its right to due process when the RTC
issued the challenged order ex parte, depriving it of an opportunity to object; and d) the
issues raised were purely of law.In this case, the records amply show that Bancommerce’s
action fell within the recognized exceptions to the need to file a motion for reconsideration
before filing a petition for certiorari.
149. Yusay v. CA

GR No. 156684, 6 April 2011

Statement of facts

Petitioners owned a parcel of land in Mandaluyong. Half was used as residence, the rest
rented out to nine other families.The land was their only source of income. The mayor tried
to expropriate the land to develop it for cost housing for priveleged but deserving city
inhabitants.

Procedural history

Petitioners filed a petition for certiorari and prohibition in the RTC. The RTC ruled in favor of
the city. Petitioner filed a motion for reconsideration, and the RTC set aside its decision/ The
city appealed to the CA. The CA ruled in favor of respondents

Statement of issue

Does prohibition lie against expropriation

Held

The rule and relevant jurisprudence indicate that prohibition was not available to the
petitioners as a remedy against the adoption of Resolution No. 552, for the Sangguniang
Panglungsod, by such adoption, was not exercising judicial, quasi-judicial or ministerial
functions, but only expressing its collective sentiment or opinion. Verily, there can be no
prohibition against a procedure whereby the immediate possession of the land under
expropriation proceedings may be taken, provided always that due provision is made to
secure the prompt adjudication and payment of just compensation to the owner. This bar
against prohibition comes from the nature of the power of eminent domain as necessitating
the taking of private land intended for public use, and the interest of the affected landowner
is thus made subordinate to the power of the State. Once the State decides to exercise its
power of eminent domain, the power of judicial review becomes limited in scope, and the
courts will be left to determine the appropriate amount of just compensation to be paid to the
affected landowners. Only when the landowners are not given their just compensation for the
taking of their property or when there has been no agreement on the amount of just
compensation may the remedy of prohibition become available.Here, however, the remedy
of prohibition was not called for, considering that only a resolution expressing the desire of
the Sangguniang Panglungsod to expropriate the petitioners’ property was issued. As of
then, it was premature for the petitioners to mount any judicial challenge, for thepower of
eminent domain could be exercised by the City only through the filing of a verified complaint
in the proper court. Before the City as the expropriating authority filed such verified complaint,
no expropriation proceeding could be said to exist. Until then, the petitioners as the owners
could not also be deprived of their property under the power of eminent domain.
150. Star Special Watchman and Detective Agency v. Puerto Princesa City

GR No. 181792, 21 April 2014

Statement of facts

Petitioners were the owners of two parcels of land in Puerto Princesa. Before Puerto
Princesa became a city, the national governmnet established a military camp, known as
Western Command. Petitioners property was used as a road right-of-way leading to the
camp, called Wescom road. Petitioners filed an action for payment of just compensation
against Puerto Princessa. The RTC ruled in favor of petitioners. The write of execution was
issued directing respondents to satisfy the money judgement.

Procedural history

Petitioners diled a motion to declare respondents in indirect contempt of court for their failure
to comply with the decision despite the issuance of a write of execution against them.
Petitioners wrote a letter to the Commission on Audit requesting that it order respondents to
pay petitioners the amount adjudged. The Commision on Audit informed petitioners that they
could not act upon the request to order respondents to pay the amount adjudged as they
had no jurisdcition over the matter as the case was already in the execution stage.
Petitioners filed a mandamus.

Statement of issue

Whether or not the remedy of mandamus is proper to compel respondents to comply with
the decision

Held

It has been held, however, that a resort to the remedy of mandamus is improper if the
standard modes of procedure and forms of remedy are still available and capable of
affording relief. Considering that the COA still retained its primary jurisdiction to adjudicate
money claim, petitioners should have filed a petition for certiorari with this Court pursuant to
Section 50 of P.D. No. 1445. Hence, the COA's refusal to act did not leave the petitioners
without any remedy at all. Thus, denied.

Вам также может понравиться